Neuro EXAM MASTER

Réussis tes devoirs et examens dès maintenant avec Quizwiz!

Case A 65-year-old man was diagnosed with mild dementia 2 months ago. The laboratory evaluation, together with a several-year history of symptoms, is consistent with this diagnosis. Although this patient can no longer work as a physicist, he is still able to visit family and friends, play tennis and golf, enjoy church, and travel with his wife. He has lost the ability to keep track of the family finances, and his wife has taken over that responsibility. After obtaining a baseline Mini-Mental State examination (MMSE) score, you start him on donepezil 5 mg once daily, which you subsequently increase to 10 mg daily after a period of 4 weeks without side effects. You also prescribed vitamin E, 400 IU, twice daily. The family does not note improvement despite 8 weeks of therapy, and they are encouraging the patient to stop taking donepezil; however, there is no further deterioration of the patient's condition. Question Concerning the efficacy of the medications, what are the most appropriate recommendations for you to make to the patient and his family?

Correct answer: 10 mg of Donepezil has shown cognition improvement; however, attenuation of further decline is also considered a positive treatment outcome Explanation In Alzheimer's disease, the cognitive deficits are thought to be primarily due to degeneration of cholinergic neurons in the cortical and limbic region, which results in deficits of cholinergic neurotransmission. Therefore, restoration of cholinergic function is undertaken primarily for symptomatic treatment for Alzheimer's disease. Cholinesterase inhibitors help in the restoration of cholinergic function. Cholinesterase inhibitors have shown to be effective in improving the cognitive symptoms of Alzheimer's disease and in slowing or attenuating further functional decline. Donepezil works in the brain as a cholinesterase inhibitor at the neuronal synapse. By inhibiting the cholinesterase enzyme, it increases the level of acetylcholine neurotransmitter and aids in maintaining mental function. It does not stop the degeneration of cholinergic neurons that produce acetylcholine. Initial dosing begins at 5mg. This may then be increased to 10mg after 4 to 6 weeks of use if there is no change in the Mini-Mental State exam and if there are no side effects. The goal of donepezil is to keep an Alzheimer's patient from deteriorating further for as long as possible. This can be monitored using the Mini-Mental State exam. Later on, if there is no change or improvement in the Mini-Mental State exam, then continue with the donepezil at the same dosage. You should explain to the caregivers that optimal clinical benefit from cholinesterase inhibitors, such as donepezil, occurs by treatment that is continued without interruption and is maintained long-term. You should stress that attenuation of further functional decline is also considered as a positive treatment outcome; however, improvement in cognition also occurs. Patients and caregivers should be educated about the benefits of donepezil as well as the importance of establishing realistic expectations regarding treatment outcomes. If there is deterioration in the Mini-Mental State exam (e.g., by 2 points), then you should then consider stopping the donepezil. Once stopped, a precipitous drop in the patient's mental function should be anticipated. Increasing the dosage of donepezil beyond 10 mg to 20 mg has not been shown to cause improvement in the patient's mental function. Plasma levels of donepezil are not used to determine the dosage of the drug. If the patient's cognitive function does not deteriorate and if no side effects are noted, then the drug should be continued at the same dosage as before. Tapering Donepezil is not required, and the drug may be stopped safely when there is documented deterioration in cognitive function while donepezil is being used. The mechanism by which vitamin E induces beneficial effect in Alzheimer's disease is not known for sure, but it is believed that it may be related to the vitamin's ability to protect nerve cell membranes from oxidative damage. Because it is less expensive and readily available, high dose vitamin E is often recommended. No evidence of interactions with donepezil has been noted.

Question Ethosuximide is a primary treatment for what type of epilepsy?

Correct answer: Absence seizures Explanation The term epilepsy refers to a group of central nervous system disorders. These disorders are characterized by repeated occurrence of sudden and transitory seizures, which may be of motor, sensory, autonomic, or psychic origin. Seizures are almost always accompanied by abnormal and excessive discharges in the brain. The many types of seizures are classified into 10 different categories. Partial seizures, also known as focal or local seizures, are broken down into 3 groups: simple partial seizures, complex partial seizures, and partial seizures secondarily generalized. The generalized seizures (convulsive and nonconvulsive) are grouped into absence seizures, atypical absence seizures, myoclonic seizures, clonic seizures, tonic seizures, tonic-clonic seizures, and atonic seizures. Ethosuximide and valproic acid are the primary agents for the treatment of absence seizures. These drugs are generally ineffective against generalized tonic-clonic seizures and simple partial seizures, which are best treated with carbamazepine or phenytoin. Febrile seizures, if treated at all, are best treated with phenobarbital. Status epilepticus is a life-threatening disorder that must be treated promptly. Intravenous diazepam and/or phenytoin is indicated for control of this disorder.

Case A 51-year-old woman presents with chronic right lower leg pain. She notes swelling, burning, and extreme pain in the right calf through ankle. She has found nothing that helps. The patient believes her symptoms all started with what she believed to be an uncomplicated ankle fracture, following a fall, that occurred approximately 8 months ago. After casting and bracing, symptoms continued, and she now limits walking to avoid aggravating her right lower leg. The burning and pain keep her awake at night, stating even the bed sheets on the leg trigger pain. She notes that there is less hair on her right leg when she shaves. She denies the need to move the leg at night, fever and symptoms in other body regions. The remainder of her review of systems and past medical history are unremarkable. On physical exam, the patient is pleasant and cooperative, with normal vitals. She struggles a bit to step onto the exam table. The lower right extremity is mottled, diaphoretic and mildly edematous. Sensation testing is altered over the entire involved area. Range of motion is limited at the right ankle. Tenderness is noted over the lower extremity soft tissue, bony prominences and joint lines. The orthopedic notes are obtained and shown. History: 51 year-old woman here for 8 wk closed FX f/u. Complains of much pain, wants more prescription pain meds. She doesn't think her ankle healed. Reports she's been compliant with wearing brace since cast removed 2 wk ago. Exam: Expected tissue atrophy, but otherwise no edema, erythema or skin breakdown. X-ray: Well-healed distal tibia fracture Plan: Advised patient fracture healed. Ambulate as tolerated. No medications recommended. Question In addition to non-medication approaches and assuming no contraindications, what therapy would be most appropriate for this patient's condition?

Correct answer: Amitriptyline Explanation This patient's diagnosis is most likely a complex regional pain syndrome (CRPS). CRPS most often develops after a minor trauma and classic characteristics include pain out of proportion with findings and history, allodynia (pain sensation with normally non-painful stimuli) and motor and sensory disturbances in the affected extremity. The mechanism for CRPS development is not well understood. Prolonged immobilization following injury is a risk factor for development of CRPS. Treatment is multi-modal and primarily consists of physical therapy, focusing on mobilization and desensitization. Other treatments are often off-label and targeted at chronic pain relief, and include corticosteroids, bisphosphonates, tricyclic antidepressants, anticonvulsants and topical anesthetics. Of the choices listed, amitriptyline, a tricyclic antidepressant, is the most appropriate. It (or other tricyclics) seem to provide some reduction in neuropathic pain. Allopurinol decreases uric acid and is used to prevent gout, a painful inflammatory condition which may cause acute pain, erythema and swelling in the affected joint(s). Gout tends to be a more acute pain and affect joints only (not the calf, as with this patient). Cephalexin is a cephalosporin antibiotic, which may be considered if this patient's symptoms were attributed to cellulitis. Cellulitis, which commonly presents with erythema, edema and tenderness in an extremity, may be considered on the differential for this patient. Cellulitis tends to occur more acutely. This patient had a normal 8 week exam with symptoms of CRPS but no findings of cellulitis at that time. Ropinirole is a dopamine agonist and is used for restless legs syndrome (RLS). RLS is characterized by abnormal sensations (more often described as crawling, tingling, aching) with an urge to move the legs. Stretching seems to help and RLS symptoms are typically not bothersome during the daytime. Ropinirole has no role in treating CRPS. Warfarin inhibits vitamin-K-depended coagulation factors and may be used for treatment of deep venous thrombosis (DVT). However, DVT is not associated with the hyperesthesia and allodynia shown in this patient, nor would we expect DVT symptoms to last 8 months.

Case A 72-year-old man presents with low-grade fever, nausea, confusion, and lethargy. His past medical history is significant for hypertension, hypercholesterolemia, and diabetes. He had sinusitis approximately 1 week prior to presentation; otherwise, he has been healthy. Laboratory workup shows a CSF with elevated opening pressure and low blood glucose levels. A Gram stain on the CSF shows Gram-positive coccus. CBC shows an elevation of PMNs, but it is otherwise normal. Question What is the most appropriate treatment in this case?

Correct answer: Antibiotic therapy Explanation The correct response is antibiotic therapy. This patient is suffering from meningitis, which is an infection of the meninges that causes inflammation. It may be acute or chronic, and many different agents can cause it. Acute meningitis develops in a few hours to a few days, and chronic meningitis may develop weeks to months after exposure. Symptoms may be different in children or in older adults with underlying health conditions, such as diabetes or other immunocompromised states, IV drug users, those on a shunt for hydrocephalus, and extremes of age. Instead of presenting with headache and nuchal rigidity, they may present with lethargy and confusion. Both infectious and non-infectious agents can cause this disease. There are many infectious agents, including bacteria, viruses, fungi, and parasites. The Gram-positive cocci in the patient's CSF points to Streptococcus pneumoniae as the organism responsible. Bacterial meningitis is treated mainly with antibiotic therapy. The treatment usually consists of ampicillin plus ceftriaxone or cefotaxime plus vancomycin when meningitis is suspected and no other data is available. Vancomycin is added to cover penicillin-resistant S. pneumoniae, which may be present. If a Gram stain has been done and shows Gram-positive cocci, the treatment of choice is penicillin G or ceftriaxone. Meropenem is the drug of choice for cephalosporin-resistant isolates. When deciding on an antibiotic regimen, it is important to keep in mind the person's age and health status. Steroids are not considered first-line treatment. However, they can be used as an adjuvant therapy. They are thought to help lessen the severity of symptoms. Dexamethasone may be considered to reduce cerebral edema. Supportive care is important, especially if the patient's disease is advanced. However, antibiotic therapy should be initiated as soon as possible to avoid complications or death. On rare occasions, anticonvulsants may be required. Transfusion is not considered the treatment of choice for bacterial meningitis. If the patient has an underlying anemia or lacks certain clotting factors, transfusions would be supplied as part of his care. However, there is no indication of this in this case. Since he has a Gram stain showing Gram-positive cocci in his CSF, antiviral therapy is not indicated in this patient. Antibiotic therapy should be initiated immediately, and supportive care should be given as needed.

Case A 31-year-old female nurse who works at a local hospital presents with a purpuric rash covering her arms, legs, and abdomen, as well as fever, chills, nausea, abdominal tenderness, tachycardia, and generalized myalgias. Prior to the development of the rash, the patient noted that she had a headache, cough, and sore throat. Laboratory studies were positive for gram-negative diplococci in the blood, along with thrombocytopenia and an elevation in PMNs. Urinalysis showed blood, protein, and casts. Vital signs are as follows: BP 92/66, P 96, RR 14, T 39. The patient denies any foreign travel and does not have any sick contacts. She is admitted to the hospital and placed in respiratory isolation. Question What major course of therapy should this patient receive?

Correct answer: Antibiotics Explanation Antibiotics are the treatment of choice for meningococcemia. The preferred drug for active infection is a third generation cephalosporin. Patients will also receive supportive care, but antibiotic therapy must be initiated quickly if the patient is to survive. Intensive care placement may be necessary if organ failure is imminent. Ventilatory support, inotropic support, and IV fluids are necessary in some. If adrenal insufficiency occurs, corticosteroid replacement may be considered. A central venous line helps to provide large amounts of volume expanders and inotropic medications for adequate tissue perfusion. Steroids have not been shown to play a major role in the treatment of meningococcemia. However, they have been used in addition to antibiotic therapy. In the case of adrenal insufficiency, for example, steroid replacement has been shown to be beneficial. Transfusion does not generally play a major role in treatment. If the patient suffers from a devastating coagulopathy, blood or blood products may be replaced as necessary. Bactericidal/permeability-increasing protein is a protein stored in the granules of neutrophils. It binds to endotoxin in vitro and neutralizes it. This technique is experimental, and it is not used in everyday treatment of meningococcemia.

Case A 44-year-old man starts to notice that his eyelids are drooping. Some time afterwards, his jaw becomes weak. He has difficulty swallowing and also experiences weakness in his limbs. He is quite embarrassed when he eats because he must use his hand to help support his jaw. His weakness gets progressively worse. Finally, he seeks medical attention. His physical examination demonstrates the weakness in his limbs; however, no sensory defects are present. A Tensilon test is done and is positive. His doctor is concerned about an associated malignancy. Question What is the most likely underlying pathology of these symptoms?

Correct answer: Antibodies to the acetylcholine receptor Explanation Antibodies directed towards the acetylcholine receptor at the neuromuscular junction are seen with myasthenia gravis. This patient has myasthenia gravis. Ocular muscle weakness, ptosis, dysphagia, and limb weakness can all be seen with myasthenia gravis. When the initial symptom is ocular weakness, Eaton Lambert Syndrome is extremely unlikely. Eaton Lambert Syndrome tends to not involve the extra-ocular muscles or the muscles involving chewing, swallowing, or speech. The Tensilon test is used in the diagnosis of myasthenia gravis. The Tensilon test consists of the administration of edrophonium. Edrophonium is a quick acting anticholinesterase. Thymic tumors are associated with myasthenia gravis. Thymic tumors are also referred to as thymomas. Approximately 10 - 15% of patients with myasthenia gravis have an associated thymoma. The majority of patients with myasthenia gravis have hyperplasia of their thymus. Botulinum toxin inhibits acetylcholine release. The site of action is at the neuromuscular junction. Botulinum toxin is an enterotoxin produced by Clostridium botulinum. Botulism can result from incorrectly canned foods. Tetrodotoxin is a toxin produced by puffer fish. The sodium channels are blocked by tetrodotoxin. The blockage of the sodium channels interferes with the inflow of sodium. As a result, the propagation of nerve and muscle action potentials is affected. Demyelination refers to the loss of myelin around the axon. Several disorders result in demyelination. An example of a demyelinating disease is multiple sclerosis. Subacute combined degeneration of the spinal cord is also called combined systems disease. Subacute combined degeneration of the spinal cord is a neuropathy secondary to B12 deficiency. It is seen in patients with pernicious anemia, especially pernicious anemia that has been present for quite some time. Symptoms include paresthesias and a loss of proprioception.

Case A 35-year-old man presents with headaches. He describes them as only occurring on one side of his head; there is a "sharp stabbing pain just above his left eye". Each headache lasts for about 2 hours at a time; they occur daily for spans of 2 -3 weeks, and they have had the same pattern every 3 - 4 months for the past couple years. During these periods, the patient's nose feels congested and his left eye appears red and waters excessively. The patient has deduced that aggravating factors include alcohol and stressful situations. He denies any known family history of headaches similar to these; there is also no family history of migraines. The physical examination is within normal limits. Question Based on the given history and physical examination, what is an effective preventative medication that might prevent these headaches in the future?

Correct answer: Calcium channel blockers Explanation The correct response is calcium channel blockers. The patient in the above scenario is experiencing a cluster headache. Cluster headaches are headaches that occur more commonly in middle-aged men. Patients will complain of severe unilateral periorbital pain that occurs daily for several weeks and potentially are associated with 1 or more of the following symptoms: ipsilateral nasal congestion, rhinorrhea, lacrimation, or redness of the eye. Patients are restless and agitated during the acute phase of these headaches. Episodes vary in length from around 15 minutes up to 3 hours; at the same time, these occurrences may last for several weeks and then resolve for many weeks at a time. Any time a healthcare provider or a patient approaches a headache of any type, treatments for the acute signs/symptoms as well as preventative treatments should be considered. Fast acting treatments to give the patient immediate relief from the signs and symptoms of a cluster headache include oxygen, triptans, octreotide, local anesthetics, and dihydroergotamine (intravenous only). Various NSAIDs may also be attempted for symptoms, but not for prevention of cluster headaches. Preventative therapies are used at the onset of the cluster episode with the overall goal of suppressing the severity of the headaches. Various options may be attempted as preventative therapy; corticosteroids, lithium carbonate, nerve blocks, ergots (sublingual), calcium channel blockers, and even melatonin usage have been effective. At times, the anti-seizure medications divalproex and topiramate may also be considered. The calcium channel blocker verapamil is very often the first choice for preventing cluster headaches.

Case A 45-year-old man goes to a party and enjoys several glasses of an alcoholic cocktail. His past medical history is significant for headaches. The drinks trigger a unilateral right-sided headache. The headache is behind his right eye and spreads to his forehead. He also notices that his right nostril has a watery discharge and his right eye is tearing. He describes the pain as if he were "being stabbed in my eye." Question What kind of headache does he have?

Correct answer: Cluster headache Explanation This patient has a cluster headache. During an attack, there can be discharge from the nose and eye on the same side of the face as the headache, a manifestation of autonomic dysfunction. Other symptoms include flushing, sweating, and Horner's syndrome. The pain of a cluster headache begins rapidly and is described as stabbing or knife-like. The headaches last from only a few minutes to several hours. Tension headaches are associated with stress. This kind of headache would not be associated with watery discharge from the nose and eye. A hangover headache would not occur while the man is still drinking. This kind of headache would not be associated with watery discharge from the nose and eye. A common migraine and classic migraine would not be associated with watery discharge from the nose and eye.

Case A 33-year-old woman presents with a headache. She also has symptoms of hemicranial pulsatile cephalic discomfort, nausea, vomiting, anorexia, photophobia, and sonophobia. The patient experiences these headaches 2 or more times a month, and she is unable to pinpoint a trigger. Her mother also had headaches. Her vital signs are: temperature 97.0, heart rate 80, respiration 16, and a blood pressure of 126/20 mm Hg. Question What is the most likely diagnosis?

Correct answer: Common migraine Explanation A common migraine is an intermittent syndrome characterized by generalized or hemicranial pulsatile cephalic discomfort. Symptoms include nausea, vomiting, photophobia, anorexia, and sonophobia. Classic migraine presents with an aura, nausea, vomiting, photophobia, anorexia, sonophobia, and a sense of just wanting to hide in a quiet dark room. Stress headaches and hatband headaches generally encompass the head. They are not 1-sided. They do not have the photophobia, nausea, vomiting, aura, or sonophobia. Cluster headaches occur on 1 side of the head, and they often cause severe tearing from the affect side eye. Cluster headaches come in groups, and months or years may go by between attacks.

Case A 45-year-old secretary presents with a 1-month history of paresthesias on the medial aspect of her right hand. She has no neck pain or shoulder pain, but she has had some difficulty typing with the right hand. Neurological exam reveals diminished sensation to pain and light touch on the medial aspect of the palmar and dorsal surfaces of the hand to the wrist, the 5th finger, and the medial aspect of the ring finger. Mild weakness of the right abductor minimi digiti is present, but the other intrinsic hand muscles are of normal strength. The remainder of the neurological exam is normal. Question What condition does patient have?

Correct answer: Compressive ulnar neuropathy at the elbow Explanation This patient has a right ulnar neuropathy, which is most likely due to nerve compression at the elbow. The abnormalities on neurologic exam are limited to weakness of the right abductor digiti minimi, which is innervated by the ulnar nerve, and a sensory disturbance in the distribution of the ulnar nerve, which includes splitting of the 4th finger. The ulnar nerve exits the brachial plexus as the terminal branch of the medial cord. The nerve then descends through the axilla and upper arm, after which it traverses the groove ("ulnar groove") between the medial epicondyle of the lower humerus and the olecranon of the ulnar bone. The nerve runs through the forearm deep to the flexor carpi ulnaris muscle; it exits the wrist through an anatomic space distinct from the carpal tunnel, which is where the median nerve is located. Ulnar nerve compression can occur in the axilla, elbow, or wrist. Occupational injury is most often located in the elbow because of the nerve's proximity to the bony surfaces; therefore, it is susceptible to external compression (e.g., in persons who lean their elbow against a hard surface). In addition, work-related repetitive flexion and extension of the elbow causes narrowing of the ulnar groove, and it results in ulnar nerve entrapment. The diagnosis can be made by radiographs of the elbow, neck, chest, and wrist. Definitive diagnosis is made by electromyography and nerve conduction tests. Conservative treatment can be tried; it should consist of elbow rest and padding, non-steroidal anti-inflammatory drugs (NSAIDs), and vitamin B-6 supplements. If the paresthesias worsen, surgical decompression coupled with transposition can be performed, depending on the site of entrapment. Median neuropathy (carpal tunnel syndrome) characteristically produces a sensory disturbance of the palmar aspect of the hand, thumb, 2nd and 3rd fingers, and the lateral aspect of the 4th finger. Radial neuropathy produces weakness of wrist extension. A C8-radiculopathy produces weakness of all intrinsic hand muscles and does not cause splitting sensory loss of the 4th finger. Radial neuropathy does not cause weakness of the abductor digiti minimi. When there is axillary nerve entrapment, the patient complains of weakness and numbness of the shoulder, with signs of deltoid atrophy.

Case A 60-year-old man presents with loss of memory and gradual inability to plan and organize his business interactions or handle finances. This loss of cognitive abilities is causing great distress in his social and occupational functioning. He is requiring more assistance in basic activities of daily living. He has no history of psychiatric problems and no alcohol or drug abuse. The patient's physical exam and medical testing reveal no physiological reasons for the impairment. Question What is your provisional diagnosis?

Correct answer: Dementia of the Alzheimer's type, early onset Explanation Dementia of the Alzheimer's type involves multiple cognitive deficits including 1 or more of the following: aphasia or language disturbance; apraxia, or the inability to carry out motor activities; agnosia, or the inability to recognize familiar objects; and the disturbance of executive activities, such as planning, organizing, sequencing, or abstracting. The disease also has a gradual onset and continuing decline. Focal neurological signs and symptoms are usually absent. If the disease occurs before age 65, this is early onset. If it occurs after age 65, this is said to be late onset. Vascular dementia has the same general symptoms and signs as Alzheimer's disease. However, with vascular dementia, the onset is usually more sudden, acute, and is discovered sooner due to the existence of clinical or laboratory evidence of a vascular cause, a history of cerebrovascular disease, or multiple infarctions. Dementia due to medical conditions may present with similar symptoms as other dementias, but are caused by other medical circumstances, or neurological diseases. These infirmities might include head trauma, Parkinson's disease, Picks disease, Huntington's disease, HIV disease, multiple sclerosis, a lateral sclerosis, vitamin deficiencies, and numerous other medical conditions. Some patients with loss of memory and other cognitive deficits have a major depression or other mood disorder that is causing their memory loss and/or cognitive deficits. This is said to be the false dementia, or pseudodementia. Substance-induced dementia includes cognitive defects caused by inhalants, sedative drugs, hypnotic drugs, prescription side effects, overdoses of prescription drugs, or other substances. This is common in the elderly.

Case A 52-year-old woman presents with a 1-year history of bilateral shaking in her hands. The shaking tends to worsen when she is using her hands; her symptoms improve when she drinks a glass of wine on the weekends. Her 82-year-old mother also has a similar shaking in her hands that developed when she was in her 50s. Question What is the most likely diagnosis?

Correct answer: Essential tremor Explanation Essential tremor (or benign tremor) typically involves the hands or head, but not the legs. The cause is unknown, but it may be inherited. Patients note that the tremor develops when moving the hands. The symptoms may worsen in times of stress, and they may be alleviated by alcohol intake. Alcohol withdrawal may be associated with a tremor, but this is unlikely, given the duration of the tremor. The tremor of Parkinson's disease is typically a resting tremor; this patient is not experiencing other symptoms of Parkinson's disease (e.g., rigidity, bradykinesia, postural instability). Huntington's disease may be associated with abnormal movements and is inherited in a familial matter, but Huntington's disease is typically fatal within 20 years of onset. This patient notes that her mother has been suffering from her tremor for close to 30 years. Multiple sclerosis is not typically associated with tremor.

Case A 42-year-old woman presents with a tremor that has become more bothersome lately. She thinks it originally began as a mild tremor twelve years ago. It affects her hands more than her head. It usually occurs when she moves her extremities. She drinks 1 or 2 alcoholic beverages every day because alcohol seems to lessen the tremor. Her mother suffered from a similar disorder. Her neurological exam is essentially normal other than a tremor of the hands and head. Question What movement disorder does this patient have?

Correct answer: Essential tremor Explanation The patient has essential tremor, which is the most common movement disorder. This postural tremor may have its onset anywhere between the second and the sixth decades of life. Its prevalence increases with age. It is slowly progressive over a period of years and eventually becomes fixed but without other neurologic abnormalities. It may be familial with autosomal dominant penetrance. Essential tremor may increase with emotion be mildly asymmetrical. The frequency of the tremor is typically 5 to 12 Hertz. In addition to the hands and forearms, it can affect the head (isolated, or in addition to upper extremities), voice, face, jaw (and rarely, legs). Some exclusion criteria for essential tremor include isolated tremor in the tongue, chin, voice or leg, sudden onset of stepwise deterioration or other neurologic signs. Essential tremor may be alleviated by alcohol (50% - 70% of patients), which may be acceptable for episodic activities (like a social engagement) but is not considered appropriate for all-day or maintenance therapy. Other medications include propranolol, primidone, and topiramate. If t the tremor is severe or disabling, surgical therapy such as thalamotomy or thalamic deep brain stimulation may be considered. Dystonic tremor often occurs with dystonia (abnormal muscle contractions with abnormal movements and postures). Usually ha an irregular oscillation. Generally affects other parts of the body, less often seen in the hands or arms. Cerebellar tremor is an intention tremor or goal-directed. It increases in severity as the extremity approaches a target. Can affect the head and the upper body. Parkinsonian tremor generally occurs at rest and with walking. More asymmetrical than essential tremor. Psychogenic tremor includes an abrupt onset, a static course, spontaneous remission, and unclassified tremors. They often increase in frequency and amplitude with attention, and they decrease with distraction.

Case A 62-year-old woman presents with excruciating pain, burning, and swelling in her left forearm and wrist. She reports that symptoms initially began with a fracture 4 months ago. The fracture was casted, and the patient was told it had healed well with cast removal at 8 weeks. She is frustrated because her symptoms have persisted and worsened, rather than improving, as she was told they would. The patient has continued to use a sling and limit use of the left arm so she doesn't worsen her condition. She is unable to wear a jacket or long sleeves, as even the fabric touching her skin causes pain. She denies fevers, pain in other areas, and new trauma. She denies polyuria and polydipsia. Her past medical history is unremarkable; she is a menopausal woman, with no known medical conditions, no history of surgeries, no regular medications and no allergies. She lives with her husband, and she is a homemaker. She denies drug, alcohol, and tobacco use. On physical exam, she is a small, thin, pleasant woman, and she is fully oriented. Vitals are normal. No gait or balance abnormalities are noted when she walks or gets onto the exam table. Her left forearm has some mild edema and erythema, as well as tenderness with even light touch. Distribution of findings includes the region from the elbow to wrist, both the anterior and posterior surfaces. Left wrist strength and range of motion are decreased compared to the right. Distal pulses, capillary refill, and reflexes are normal. The remainder of the exam, including mental status, is normal. Question What medication would be most appropriate for this patient's likely diagnosis?

Correct answer: Gabapentin Explanation This patient's diagnosis is most likely a complex regional pain syndrome (CRPS). CRPS most often develops after a minor trauma and classic characteristics include pain out of proportion with findings and history, allodynia (pain sensation with normally non-painful stimuli) and motor and sensory disturbances in the affected extremity. The mechanism for CRPS development is not well understood. Prolonged immobilization following injury is a risk factor for development of CRPS. Diagnosis is clinical, and testing is done to rule out other disorders. Treatment is multi-modal and primarily consists of physical therapy, focusing on mobilization and desensitization. Other treatments are often off-label and targeted at chronic pain relief; they include corticosteroids, bisphosphonates, tricyclic antidepressants, anticonvulsants, and topical anesthetics. Gabapentin would be a reasonable choice for this patient. Heparin would be used if this patient's symptoms were attributed to a deep venous thrombosis (DVT). DVT is rare in the upper extremities; it is not associated with the hyperesthesia and allodynia shown in this patient. Methadone, a longer-acting opioid medication, can be used for chronic pain. However, for several reasons, this is not the most appropriate medication to use in CRPS. Opiates carry addiction potential and increase fall risk in the elderly. CRPS is often categorized as early (<6 months' duration) versus late (>6 months). For early CRPS, opiates should be avoided and even with late CRPS, other medications options should be utilized before initiating opiates. Probenecid is a medication which inhibits urate resorption. It is used for gout, a painful inflammatory condition which may cause acute pain, erythema and swelling in the affected joint(s). Gout tends to be more acute and affect joints (not the forearm, as with this patient). Vancomycin is an IV glycopeptide antibiotic; it is used for severe bacterial infections, including cellulitis. Cellulitis, which commonly presents with erythema, edema and tenderness in an extremity, may be considered on the differential for this patient. Cellulitis tends to occur more acutely. This patient had a normal 8 week exam with symptoms of CRPS, but no findings of cellulitis at that time.

Case A 39-year-old woman presents with her sister. The patient is confused, but she is resting comfortably. The sister explains that the patient had a fit and suddenly blacked out. Her body became very rigid and unmovable. Jerking motions of her arms, legs, and head started. They lasted for a few minutes and then stopped. She was unresponsive for about 5 minutes, during which time the sister called the paramedics. Question What is the most likely diagnosis?

Correct answer: Grand mal seizure Explanation The correct answer is grand mal seizures, or tonic-clonic seizures. They are characterized by a tonic phase—which includes a sudden loss of consciousness and rigidity that typically lasts less than a minute—followed by a clonic phase characterized by jerking motions (usually for a few minutes), and then a flaccid coma state. Patients with simple partial seizures do not experience a loss of consciousness. Absence seizures present with an impairment of consciousness (occasionally with mild tonic, clonic, or atonic symptoms), and the patient is typically not aware of the attacks. Complex partial seizures typically have impaired consciousness with focal motor symptoms or somatosensory symptoms, and they do not have the three phases described. Status epilepticus is defined as a fixed and enduring epileptic condition of at least 5 minutes, with repeated seizure episodes and no recovery period between them.

Case A 48-year-old woman presents after a seizure. Prior to the seizure, she experienced confusion and disorientation preceded by nausea, vomiting, and blurred vision. Symptoms appeared after working for several hours in the garden under the sun. Her medical history is significant for the presence of schizophrenia, for which she takes chlorpromazine at bedtime. Her temperature is 41° C, BUN and creatinine are elevated, and there is neutrophilia, hemoconcentration, and lactic acidosis. The seizure is possibly drug-related. Question What is the most likely diagnosis?

Correct answer: Heat stroke Explanation Heat disorders can be exertional and nonexertional; both can be drug-related. Neuroleptics (e.g., phenothiazines, thioxanthenes) may impair thermoregulation due to both anticholinergic and antidopaminergics effects. Anticholinergics inhibit sweating; this disturbs thermoregulation during exercise, or under conditions of environmental heat stress. Antidopaminergics elevate the set point of the temperature regulation center in the hypothalamus. This patient probably suffered heat stroke; it is a life-threatening condition characterized by elevated body temperature with nausea, blurred vision, confusion, disorientation, and seizures. Hemoconcentration, anuria, rhabdomyolysis, kidney dysfunction, lactic acidosis, and even disseminated intravascular coagulation may result. Heat cramps are a mild disorder characterized by painful muscle contractions due to temporary fluid and electrolyte depletion. There are no signs and symptoms of neurological dysfunction, and body temperature is normal. Neuroleptic malignant syndrome is an idiosyncratic reaction to neuroleptics, most commonly phenothiazines and butyrophenones; it is characterized by rigidity, fever, and autonomic instability. It is not connected with the exposure to heat and exertion. Men under the age of 40 are at greatest risk. Malignant hyperthermia is a nonexertional, idiosyncratic reaction to the anesthetic. Heat exhaustion is a condition with a severity that lies between heat cramps and heat stroke. Body temperature might be slightly elevated, and there may be neurological signs (e.g., headache), but there will be neither severe confusion nor seizures.

Question What is the most accurate statement concerning the treatment of migraine headache with sumatriptan?

Correct answer: If it is effective in the initial therapy, it is often effective in a recurrence of symptoms. Explanation If it is effective in the initial therapy, it is often effective in aborting a recurrence of symptoms. Sumatriptan is a serotonin agonist; it is believed that its effectiveness is related to the importance of serotonin in the regulation of cranial and intracranial vasoactivity. The most common side effect is nausea with or without vomiting, but these are frequent symptoms of a migraine headache itself. It may also cause sedation and drowsiness, but irritability is not common. The recommended oral dose is 25 mg, and the recommended subcutaneous dose is 6 mg. Sumatriptan is indicated for the treatment of a migraine headache, but it is not recommended for prophylaxis. Clinical trials have shown that sumatriptan is effective in reducing the severity of a migraine headache in approximately 70-80% of patients, but recurrence of the headache is seen in as many as 35-45%. A second dose of sumatriptan is effective in treating the recurrence in the majority of patients.

Case A 69-year-old woman suffered a massive stroke 6 weeks ago. She is now recovering from the stroke, but she has residual paralysis and sensory impairment of her right arm. She is also unable to speak and unable to turn her eyes to the right. Question The most likely site of her lesion is in the area that is supplied by what artery?

Correct answer: Left middle cerebral artery Explanation To determine the vessel involved, the site of the lesion must be determined. The sensory and motor homunculi are both located on the lateral surface of the cerebral hemispheres. Each half supplies the contralateral side of the body. From this, it is possible to determine that the area affected is on the lateral surface of the left cerebral hemisphere. The motor and sensory homunculi are placed such that the lower half of the body is more medial and superior with the representation of the area below the knee extending onto the medial surface of the hemisphere. The left middle cerebral artery is responsible for motor function of the upper extremity and face, so upper extremity weakness and facial plegia are present with infarctions in this territory. Lower extremity weakness may be present but is less prominent. The left hemisphere is also the language side where connections exist among Wernicke's area, Broca's area, and the oral motor cortex. Both the Broca's motor area (area #44) and the Wernicke's area are in the area supplied by the middle cerebral artery on the lateral aspect of the hemispheres. Involvement of the right anterior cerebral artery would result in motor and sensory function in the left lower extremity, sparing the hands and face. Involvement of the left anterior cerebral artery would affect motor function in the right lower extremity but would spare the hands and face. Involvement of the right middle cerebral artery would result in weakness of the upper extremity as well as facial plegia on the left side. Left lower extremity weakness may be present but is usually less pronounced. Involvement of the left or right posterior cerebral artery results in visual deficits.

Question A 45-year-old woman presents with a 1-month history of double vision, tripping, urinary incontinence, and restless legs; the symptoms are accompanied by weakness, numbness, and tingling sensations. She denies any headaches, difficulty with speech, or memory impairment. Her physical examination does not demonstrate impairments in mental status, speech, or memory. Her muscular strength is 2+ bilaterally in the upper and lower extremities, although it is diminished from a previous exam 1 year ago. Deep tendon reflexes are also increased compared to 1 year ago. What is the most likely diagnosis?

Correct answer: Multiple sclerosis Explanation Multiple sclerosis is correct because this patient demonstrates most of the distinguishing characteristics; spasticity (what she describes as restless legs), diplopia, unsteadiness, numbness, tingling, and urinary urgency/hesitancy (sphincter abnormalities). Vitamin E deficiency is incorrect because its main features are ataxia, slurred speech, sensory loss, and absent deep tendon reflexes. This patient doesn't report or exhibit speech impairment. Vitamin B12 deficiency is incorrect because its main features are polyneuropathy, mental status changes, and optic neuropathy. These patients do not exhibit or report mental status issues. Friederich's ataxia is incorrect because its main features are clumsy hands, ataxia, pes cavus, and sensory disturbances. Charcot-Marie-Tooth Disease is incorrect because its main features are polyneuropathy, distal weakness, impaired deep tendon reflexes, and sensory loss.

Case Ico-delete Highlights A 5-year-old boy presents with an altered gait. His mother reports that he first walked independently at 15 months of age, but that for the past 3 months his gait has become waddling and he has been taking a less active part in physical activities with his friends. He has even begun to fall on occasion. Past medical history is unremarkable. The mother recalled that a maternal uncle died in his late teens of an unknown condition. Physical examination reveals normal vital signs and growth parameters. Gait is waddling. Running is awkward, and hopping on either foot can only be performed with great difficulty. Strength of the deltoids and hip flexors was 3/5, with normal strength in the distal muscle groups. The calves appear enlarged bilaterally. He needs to climb up upon himself to arise from a sitting position on the floor. Question What test is most likely to yield a specific diagnosis?

Correct answer: Muscle biopsy Explanation Male gender, age of onset, gait alteration, proximal weakness, pseudohypertrophy of the calves, and positive Gower sign (needing to climb up upon himself), as well as the family history, strongly suggest Duchenne muscular dystrophy, which is an X-linked recessive condition caused by any of a number of mutations of the dystrophin gene. The condition is progressively debilitating. In some cases, it is associated with intellectual disability and myocardial disease. Muscle biopsy will usually show muscle necrosis with replacement of muscle tissue by fat and fibrous tissue (contributing to the pseudohypertrophy of the calves). An elevated aldolase level is nonspecific for any number of muscle disorders. Similarly, an elevated creatinine kinase is also nonspecific for any number of muscle disorders. Electromyography will help to distinguish between neuropathy and myopathy, but it rarely provides an exact diagnosis. Nerve conduction velocities will help distinguish between neuropathy and myopathy, but they will not provide a specific diagnosis.

Case A 44-year-old presents with periods of fluctuating weakness that seems to improve when she lies down. She has not experienced chest pain, shortness of breath, or fevers over the last month. On physical exam, you note nystagmus and abnormal facial expressions. She states that, otherwise, she feels she is in overall excellent health. She is physically active and her diet is good. You order a complete blood count, comprehensive metabolic panel, and thyroid panel; all results are within normal limits. She tells you that she recently saw her family physician and learned she was in the early stages of rheumatoid arthritis. Question What disease may be coexisting?

Correct answer: Myasthenia gravis Explanation The clinical picture is suggestive of myasthenia gravis. Small motor units, such as ocular muscles, are most often affected, which produces nystagmus. The major clinical features are fluctuating fatigue and weakness that improve after a period of rest and after administration of acetylcholinesterase inhibitors. Rheumatoid arthritis is more common in patients with myasthenia gravis than in the general population. Systemic lupus erythematosus is an autoimmune disease that affects multiple organs; it occurs mainly in young women. The systemic features include fever, anorexia, malaise, and weight loss; these features are not present in this patient. Parkinson's disease is a clinical syndrome of rigidity, bradykinesia, tremor, and postural instability. These symptoms are not present in this patient. Bell's Palsy is an idiopathic facial paresis attributed to an inflammatory reaction of the facial nerve. It affects only 1 side of the face and impaired taste is common. The common clinical presentation in multiple sclerosis is tingling or unsteadiness in a limb, spastic paraparesis, retrobulbar neuritis, diplopia, disequilibrium, or a sphincter disturbance such as urinary urgency or hesitancy. These findings are not present in this patient.

Case A 64-year-old woman presents for evaluation of myxedema. She is experiencing constipation, headaches, weakness, fatigue, lethargy, somnolence, cold intolerance, decreased sweating, paresthesias, arthralgias, and muscle cramps. She has dry, coarse skin. Her face and extremities are swollen, and her voice is coarse. Question What gait abnormality would you most likely expect to see in this patient?

Correct answer: Myopathic Explanation Myxedema is the accumulation of hydrophilic mucopolysaccharides in subcutaneous tissues in severe hypothyroidism. Neuromuscular symptoms are present in 30-80% of patients with hypothyroidism, and they usually improve or disappear with correction of the hypothyroidism. The most common complaints are of muscle cramping, proximal symmetrical muscle weakness, muscle stiffness, and exercise intolerance. These manifestations can occur at any time during the presentation of hypothyroidism. Proximal muscle weakness presents as dystrophic or circumduction gait. These patients usually complain of difficulty climbing stairs because they are actually using the handrails or their own hands to pull themselves up with their arms. Thyroid myopathy is also characterized by slowness of both muscle relaxation and muscle contraction. This is due to the decreased amount of myosin ATPase, leading to the slowing of release and reaccumulation of calcium in the endoplasmic reticulum. In thyroid disease, segmental demyelination of peripheral nerves with decreased nerve conduction velocities may also be observed; there may also be a decrease in vibration, joint-position, and touch-pressure sensations. Astasia-abasia is seen most commonly in the very late stages of normotensive hydrocephalus. Patients may demonstrate the inability to walk, inability to stand, sit, rise from a chair, or turn over in bed; this advanced stage is called "hydrocephalic astasia-abasia." Ataxic gait is a wide-based gait with difficulty or inability to tandem walk due to the midline cerebellar diseases. Patients sway from side to side as they walk. Typically, patients who have midline cerebellar degeneration have difficulty with tandem gait, but they have normal coordination in their arms. Unilateral cerebellar lesions often cause incoordination of the arm and leg on the same side as the lesion. A cerebellar abnormality is implicated in ataxia only if weakness, spasticity, and position-sense abnormality are absent. A steppage gait occurs when the patient has either a foot drop from weakness of the foot dorsiflexors or poor position sense in the feet. With a foot drop, the patient flexes the hip so that the foot will clear the floor. Patients with poor position sense have a wide-based gait; their feet slap the floor; and they require visual assistance to walk. Limp is a gait abnormality usually due to pain ("antalgic gait") or skeletal deformities, e.g., hip deformity with or without associated muscular weakness ("Trendelenburg gait"), with the body shifted over the affect hip. In magnetic gait (when feet seem attached to floor as if by a magnet), each step is initiated in a "wresting" motion carrying feet upward and forward. Magnetic gait and gait disturbances are typically the initial and most prominent symptom of the triad of normotensive hydrocephalus (dementia, incontinentia, and gait disturbance) and may be progressive due to expansion of the ventricular system with a progressive traction of motor fibers in pyramidal tract. The gait disturbance in normotensive hydrocephalus can be classified as mild (unsteadiness, impaired balance, especially when encountering stairs and curbs, usually described as cautious gait), marked (evident difficulty walking or considerable unstable gait when the patient needs mobility aids to provide some stability), or severe (unaided gait not possible, as there is a constant tendency to fall backwards; gait is broad based in order to increase the base of support and provide some stability; and steps height is decreased, as well as foot-floor clearance and the speed of walk). This style may remind one of Parkinsonian gait, except there is no rigidity or tremor.

Case A 23-year-old man presents after being thrown from a motorcycle onto a cement median. He is awake and appears alert during the exam. He believes that his helmet was knocked off at the point of impact, but he cannot recall the specifics of his crash. A head CT scan is consistent with a cerebral contusion. Question What finding on a CT scan would you see with this diagnosis?

Correct answer: No cortical disruption or necrosis, but multiple small isolated hemorrhages throughout the cortex with edema Explanation The correct response is no cortical disruption or necrosis, but multiple, small, isolated hemorrhages throughout the cortex with edema. A cerebral contusion is a focal lesion of the brain, secondary to trauma. Contusions usually occur over the crest of gyri, largest at the surface of the brain and tapering in size into the white matter. Cerebral contusions can occur under the site of impact, or on the contralateral side of the brain (a contrecoup lesion). The CT scan of a cerebral contusion characteristically reveals punctate hemorrhages and edema under the site of impact. A cerebral contusion is defined as "bruising" of the brain without interruption of the cortex. Frequently, a subarachnoid hemorrhage overlies the area of contusion. A biconvex-shaped hematoma over the cerebral hemisphere is indicative of epidural hematoma. A peripheral ring of enhancement around areas of a hematoma is seen 1 to 6 weeks after a traumatic intracerebral hematoma, not in cerebral contusion. Panventricular enlargement is a feature of hydrocephalus.

Case A 34-year-old man presents with a right-sided periorbital headache of 20 minutes duration. The pain is described as excruciating and bore-like, and it is associated with tearing and redness of the right eye, right eyelid swelling, and nasal congestion. The headaches have been occurring at multiple times nearly daily over the past 2 weeks, and each episode lasts approximately 30 to 60 minutes. The pain is not relieved with acetaminophen or ibuprofen. There is no nausea, vomiting, or photophobia. There is no history of amaurosis, visual scintillations, diplopia, focal weakness, numbness, neck stiffness, or other common neurologic complaints. The patient has had similar bouts of headaches over the past 5 years; the bouts typically last 3 to 4 weeks. The patient has an otherwise unremarkable medical history. General exam is remarkable for right eye conjunctival injection. The neurologic exam is significant for a mild right ptosis; the right pupil is 2 mm and reactive to light; the left pupil is 4 mm and reactive to light. There are no other neurologic findings. Question What is the most effective abortive therapy for this patient's headache?

Correct answer: Nasal oxygen Explanation This patient has cluster headaches (CH). CH is a relatively rare primary headache disorder characterized by recurrent periorbital or temporal headaches; they are associated with ipsilateral conjunctival injection, lacrimation, rhinorrhea, eyelid edema, and oculosympathetic signs (ptosis and miosis). The pain of CH is severe, typically characterized as "excruciating" in nature. CH occurs in cycles (the cluster period), usually lasting several weeks to months. The headaches are relatively brief in duration (15 to 180 minutes), but they can occur multiple times daily (up to 8) during the cluster period. Unlike other primary headaches, CH is more common in men. CH is classified as episodic or chronic, depending upon the duration and frequency of the cluster period. In episodic CH, the cluster period lasts from 7 days to 1 year; cycles are separated by pain-free remissions of at least 2 weeks. In chronic CH, the cluster period is longer than 1 year; remissions last less than 2 weeks. Abortive therapies for acute CH include nasal oxygen (5 to 10 L/minute), sumatriptan (subcutaneous), and dihydroergotamine (DHE, subcutaneous or intramuscular). Oral analgesics (acetaminophen, aspirin, ibuprofen, narcotics) are much less effective in aborting acute CH. A brief course of corticosteroids is often effective in shortening the CH cycle. Prophylactic agents in CH include lithium, methysergide, verapamil, and valproic acid.

Case A 51-year-old woman presents with chronic right lower leg pain. She notes swelling, burning, and extreme pain in the right calf through ankle. She has found nothing that helps her symptoms. The patient believes her symptoms all started with what she believed to be an uncomplicated ankle fracture, following a fall, approximately 8 months ago. After casting and bracing, symptoms continued, and she now limits walking to avoid aggravating her right lower leg. The burning and pain keep her awake at night, stating even the bed sheets on the leg trigger pain. She notes that there is less hair on her right leg when she shaves. She denies the need to move the leg at night, fever, and symptoms in other body regions. The remainder of her review of systems and past medical history are unremarkable. On physical exam, the patient is pleasant and cooperative, with normal vitals. She struggles a bit to step onto the exam table. The lower right extremity is mottled, diaphoretic and mildly edematous. Sensation testing is altered over the entire involved area. Range of motion is limited at the right ankle. Tenderness is noted over the lower extremity soft tissue, bony prominences and joint lines. The orthopedic notes are obtained and shown. History: 51 y/o woman here for 8 wk closed FX f/u. Complains of much pain, wants more prescription pain meds. She doesn't think her ankle healed. Reports she's been compliant with wearing brace since cast removed 2 wk ago. Exam: Expected tissue atrophy, but otherwise no edema, erythema or skin breakdown. X-ray: Well-healed distal tibia fracture Plan: Advised patient fracture healed. Ambulate as tolerated. No medications recommended. Question What finding would be an expected diagnostic test finding, given this patient's likely diagnosis?

Correct answer: Normal electromyelogram (EMG) Explanation This patient's diagnosis is most likely a complex regional pain syndrome (CRPS). CRPS most often develops after a minor trauma and classic characteristics include pain out of proportion with findings and history, allodynia (pain sensation with normally non-painful stimuli) and motor and sensory disturbances in the affected extremity. The mechanism for CRPS development is not well understood. Prolonged immobilization following injury is a risk factor for development of CRPS. Diagnosis is clinical and testing is done to rule out other disorders. Of the choices listed, the only expected finding is a normal electromyelogram (EMG), a nerve conduction test to evaluate skeletal muscles and their associated nerves. 'Punched-out' lesions on X-ray are a finding classically associated with multiple myeloma, a malignancy of the plasma cells. Patients with multiple myeloma may experience bone pain, but may also experience fatigue, anemia and frequent infections. The allodynia and skin changes are not associated with multiple myeloma. Upon imaging the patient's lower right extremity, one would not expect to find an abscess on MRI. While an abscess or cellulitis may cause extreme pain and tissue tenderness, infection would not explain the skin changes (decreased hair growth, sweating). CRPS is not thought to be a primarily inflammatory process and patients with CRPS do not have elevated white blood cell counts or elevated C-reactive proteins.

Case A 16-year-old previously healthy boy fell on a cemented surface while skateboarding 3 weeks ago, and he lost consciousness for 2 minutes. He was not wearing a helmet. A CT scan showed no abnormalities. He is now back at school and complains of difficulty concentrating in class, frequent spells of dizziness, and headaches. His neurologic exam is normal. Question Based on his history, what is the most likely cause of the patient's symptoms?

Correct answer: Postconcussive syndrome Explanation This adolescent displays the characteristic features of postconcussive syndrome (PCS), which is usually diagnosed on the basis of presence of 3 out of the following 8 features after a significant injury event: 1) headache; 2) dizziness; 3) fatigue; 4) irritability; 5) insomnia; 6) concentration or 7) memory difficulty; and 8) intolerance of stress, emotion, or alcohol. It is unlikely to be a traumatic brain injury because he had a short duration of loss of consciousness, normal head CT, and his current neurologic exam is normal. Symptoms of dysthymia or depression can seem to overlap with those of PCS, but he had no such symptoms prior to the traumatic event. Complex partial seizures are unlikely; they are usually associated with focal seizure activity in the form of tonic/tonic-clonic movements. Subdural hemorrhage is usually evident on head CT, but delayed subdural hemorrhages are rarely seen. In those situations, they are associated with abnormalities on neurologic examination.

Case A 70-year-old man with hypertension, diabetes, obesity, and coronary artery disease presents with a 3-day history of fever, headache, nausea, vomiting, lethargy, and myalgias. His wife states that the symptoms began the day following a summer's evening walk around a lake. Today, his condition has worsened; he has developed diminished level of consciousness, behavioral changes, and abnormal movements. His physical exam reveals a lethargic man with photophobia, cranial nerve deficiencies, neck stiffness, and abnormal extremity movements. His skin exam is notable for the finding in the image. Question What is the next most appropriate step in the management of this patient?

Correct answer: Perform a CT scan of the brain Explanation This patient's most likely diagnosis is West-Nile virus encephalitis, which is caused by a flavivirus mosquito-borne infection. Laboratory samples and blood cultures should be collected before the start of IV therapy. Even in uncomplicated cases of encephalitis, most authorities recommend a neuroimaging study (e.g., magnetic resonance imaging [MRI], or if unavailable, a contrast-enhanced head computed tomography [CT] scan) before lumbar puncture, if there are focal complaints or findings, signs to search for evidence of elevated intracranial pressure obstructive hydrocephalus, or mass effect due to focal brain infection. Performance of a head CT scan with and without contrast agent should be performed in virtually all patients with encephalitis. Head CT scanning also helps exclude brain hemorrhage or infarction as a cause of an encephalopathic state. Patients with signs of increased intracranial pressure should have their heads elevated. Intraventricular ICP monitoring is controversial. Some authorities believe that dangerous focal edema with a pressure gradient between the temporal lobe and the subtentorial space usually is not detected by the monitor; this failure of detection can lead to a false sense of security. In fact, monitor placement may potentially aggravate a pressure gradient. The treatment for viral encephalitis is supportive in nature in most cases; with the important exceptions of HSE and varicella-zoster encephalitis, the viral encephalitides are not treatable beyond supportive care. Interventions include control of straining and coughing, hydration, analgesia, as well as shock and seizure prophylaxis and treatment. Investigational agents to consider in severe cases include ribavirin and interferon alpha-2b. Although most histologic features are nonspecific, brain biopsy is the criterion standard because of its 96% sensitivity and 100% specificity. The presence of Negri bodies in the hippocampus and cerebellum are pathognomonic of rabies, as are HSV Cowdry type A inclusions with hemorrhagic necrosis in the temporal and orbitofrontal lobes.

Case A 39-year-old woman presents due to shaking hands. She states that the shaking occurs in both hands, worsens with use, and improves when she drinks alcohol. She is otherwise healthy and has no other complaints. Her neurological examination is normal, aside from a bilateral tremor when she reaches for an item. Her symptoms have been affecting her daily activities, and she is requesting treatment. Question What is the recommended first-line treatment in this patient?

Correct answer: Propranolol Explanation Propranolol is correct. The patient's symptoms are consistent with essential tremor. In patients requiring medication, beta-blockers are first line. Propranolol is the preferred agent. Levodopa is used in the treatment of Parkinson's disease. While Parkinson's disease does present with tremor, it is a resting tremor; it is usually accompanied by other symptoms, such as bradykinesia, rigidity, and postural instability. Gabapentin may be used in the treatment of essential tremor, but it is reserved as second-line therapy. Bromocriptine is used in the treatment of Parkinson's disease. Deep brain stimulation may be used in the treatment of tremor. However, it is not considered a first-line agent.

Case A 21-year-old woman presents with double vision that occurs only when she looks to her right side. The double vision began when she woke up in the morning. She had an episode of left leg numbness while at summer camp 6 years ago, but it resolved over 3 days and she never told anyone. Also, 3 years ago she saw her private physician after a 7-day episode of right eye pain and visual blurring. He attributed her symptoms to an ocular migraine. There is no history of head trauma. She hasn't had any infections, fevers, or immunizations recently. She is awake, alert, and in no acute distress. Positive findings include mild pallor and atrophy of the right optic disc. Bedside visual fields and acuity are normal. Testing external ocular motion in both eyes together reveals that there is no left eye movement beyond midline when attempting to look to her right, accompanied by right eye lateral nystagmus. When the left eye is tested with the right eye closed, eye movements are full. No other motor signs are found. No skin, sensory, or hearing findings are found. A magnetic resonance imaging scan (MRI) of the head with gadolinium enhancement reveals a 2 x 3 cm lucency in the region of the right parietal white matter without swelling or enhancement. Multiple sclerosis (MS) is suspected. Question Given this history, how would this patient's condition best be subtyped?

Correct answer: Relapsing Remitting MS (RRMS) Explanation The patient has had 3 discrete clinical exacerbations (right parietal white matter/left leg numbness, right optic nerve/ visual blurring, eye pain with residual disc pallor, and median longitudinal fasciculus/resolved diplopia), with full clinical resolution. It should be kept in mind that this is a clinical descriptive and disability rating. Therefore, residual abnormalities on exam and ancillary lab testing do not define the subtype. The patient is best subtyped at this time as Relapsing Remitting MS. Subtyping may change over time with new clinical events. This patient might later develop persistent clinical disability (e.g., severe ataxia, persistent urinary incontinence, extremity weakness, diplopia, etc.). She would then best be reclassified as Secondary Progressive MS, whether these occurred as residua of discrete relapses or as an independent progressive deterioration independent of relapses. Unlike the classic Relapsing/Remitting presentation of MS, some patients present with a progression of neurological disability with no clear relapses, per se. These are subtyped as Primary Progressive MS. Some patients may initially rate a Primary Progressive MS rating, yet later develop episodes of superimposed relapses and remissions. These patients are then reclassified as Progressive Relapsing MS.

Case A 16-year-old girl is brought to the emergency room for evaluation of continuous seizures. The patient was first diagnosed with idiopathic epilepsy at age 10. She was started on valproic acid. Phenobarbital and carbamazepine (Tegretol) have been tried without success, and the phenobarbital resulted in transient severe personality changes. 2 previous EEGs have been normal. This spell occurred abruptly at the family dinner table without apparent warning. Past medical history has been normal except for the usual childhood diseases without complications. 2 days prior to presentation, she saw her primary physician for nausea and vomiting; as he believed she had the flu, he started her on prochlorperazine (Compazine), which relieved her nausea. Physical exam reveals a well-developed, well-nourished young woman lying on a gurney; there is continuous but variable motor jerking of all 4 extremities. The jerking varies in intensity from side to side. The head intermittently turns from side to side. Eyes are closed tightly and cannot be passively opened. Deep tendon reflexes cannot be assessed. The response to plantar stimulation is withdrawal bilaterally. There are no signs of trauma to the head or elsewhere. Skin is warm and dry with normal color, and vitals are normal. The general medical exam is otherwise normal. Labs are done stat, and they include normal complete blood count and differential, normal chest X-ray, normal urinalysis on cathed specimen, normal chemistry profile, and normal arterial blood gasses. Question Pseudoseizure (psychological rather than epileptic event) is suspected. What test would be of greatest value in this acute clinical setting in differentiating pseudoseizures from true epileptiform seizures

Correct answer: Stat electroencephalogram (EEG) Explanation If an electroencephalogram can be obtained on a stat basis, it can be of immeasurable help in differentiating pseudoseizures from true seizures. During a spell suggesting generalized tonic-clonic seizure, the electroencephalogram should show generalized spike and wave activity; after the spell, it should show generalized slowing and disorganization lasting 30-90 minutes. Even if the active phase of the spell were over before the electroencephalogram, if done in this 30-90 minutes post-event time frame, it could be compared with a later electroencephalogram, which would probably be normal in case of pseudoseizure (5). If an electroencephalogram is normal during or immediately after this event, it is major evidence against epilepsy and for pseudoseizures. In view of the above, and with a history of 2 previous normal electroencephalograms, obtaining an outpatient electroencephalogram or one during subsequent admission would most likely be a waste of time and money. If a video camera were available, video recording the patient during a spell, especially during electroencephalography, may provide excellent evidence or even lead to a definitive diagnosis. A serum prolactin level is a very good test in differentiating seizure from pseudoseizure; most generalized seizures, and many complex partial seizures, result in elevated levels during this initial 30-minute period following the event (1). It should be done but the results may not be back for several days. In a study by Holtkamp et al., serum creatine kinase (CK) in psychogenic status had a mean of 39 U/L (range 16-90), but in true epileptic status the mean was 699 with a range of 57-2,625 (1). In this study, creatine kinase began to rise after a delay of 3 hours and peaked after more than 36 hours. Therefore, it will not help in an emergency room setting, but it should be done and repeated (e.g., at 12, 24, and 36 hours after the event), as it can provide good evidence later on for true seizures (levels will rise) or against seizures, and for pseudoseizures if levels do not rise or rise only minimally. Through the use of 24-hour electroencephalography and video monitoring, comprehensive epilepsy centers can be extremely valuable and even definitive in differentiating pseudoseizures from true epileptic seizures. However, they are tertiary centers for evaluation of patients who have not received definitive diagnoses or have not responded to treatment for either. Also, these centers are not that common, and referral can be prolonged and complicated. As valuable as psychiatric referral could be if the patient is having pseudoseizures, a consult at this time would not be helpful. The patient's condition would preclude any meaningful interview. Also, pseudoseizure is a diagnosis of exclusion, and a full evaluation for other causes of her spells has not yet been done. Furthermore, premature referral could negatively impact cooperation from the patient and family. Psychiatric referral should be considered only after a firm diagnosis of pseudoseizures has been made, or where it becomes apparent that behavioral input is necessary to help in this differential. Summary of objective diagnostic testing that can be done to differentiate seizures from pseudoseizures: Electroencephalograms, if available, can be very helpful during the acute phase; with tonic and/or clonic movements, one would expect to see epileptiform abnormalities. Psychomotor seizures may show in the electroencephalogram, but not necessarily. A negative electroencephalogram is strong evidence of pseudoseizures, but not proof. An encephalogram done within 90 minutes after an epileptic seizure frequently shows transient postictal changes that will not be seen after a psychogenic seizure (5). Prolactin levels, done within 30 minutes of a seizure, may be helpful since most generalized seizures, and many complex partial seizures, result in elevated levels (1). Serum creatine kinase (CK) in psychologic status had a mean of 38 U/L (range 16-90), but in true epileptic status, the mean was 699 with a range of 57-2,625. In the study cited in (1), with true epileptic seizures, creatine kinase begins to rise after a delay of 3 hours, and it peaks after more than 36 hours. EEGs, if they can be obtained on a stat basis, can be most valuable in picking up the presence or absence of epileptiform abnormalities during the spells or the presence or absence of postictal changes in the 30-90 minute period following true seizures.Video recording of the patient, if a video camera is available, can be most helpful in later efforts at diagnosis. A combination of EEG and video recording could be of major importance. At a later time, continuous video and electroencephalography (vEEG) may be arranged if necessary, and it constitutes the gold standard for diagnosis.

Case A 44-year-old woman presents with what she describes as "tingling in her right arm". It comes on along her palm, and it has been increasing in severity. She also notices it more at night and while attempting to carry her shopping bags. Of late, her pain has been so bad that she has been losing sleep. Examination reveals a positive Phalen's test, with weakness and atrophy of the thenar muscles. She is not a known diabetic or hypertensive, and she is otherwise healthy. Her thyroid function is normal. Her primary care physician advised a splint and vitamin supplements, neither of which have worked after 3 months of regular use. Question What is the next best step in management?

Correct answer: Surgical release Explanation Surgical release is the correct answer. Surgery should be considered when symptoms do not respond to conservative measures and in patients with severe nerve entrapment. Evidence of severe nerve entrapment includes muscle atrophy, demonstrable muscle weakness, and nerve conduction studies showing impaired conduction. The patient has 3 of the 4 features mentioned above, with failure of conservative management, muscle weakness, and thenar muscle atrophy; therefore, surgery is indicated. The use of gabapentin is incorrect. While it is a useful drug in symptom management, it does not halt or reverse the disease process, and it would be the wrong option in a person with nerve damage. Local corticosteroid injection is incorrect. It is used when conservative measures fail, but in the setting of definite nerve damage, surgical treatment is the option of choice. Systemic corticosteroid use is incorrect. Trials have shown them to be superior to NSAID's and vitamin supplements; however, as with local corticosteroid injections, in the setting of definite nerve damage, surgical treatment is the option of choice. Ultrasonic massage is incorrect. It is a conservative symptom relieving option, and it is not useful in halting or reversing the disease process.

Case A 45-year-old woman presents with a 2-week history of daily headaches. She describes feeling as if a rubber band is around her temples, making it extremely hard to concentrate when trying to complete tasks at work or at home. Further questioning reveals that the patient is under a significantly increased amount of stress at work due to a new, extremely rigid boss who started last month. She denies any auras, nausea, vomiting, or specific neurological issues, but she does admit to some sensitivity to excessive noise and glaring lights. She has always treated these with two 500 mg tablets of acetaminophen as needed, but this current episode has had little or no relief with this regimen. Question What is the most likely diagnosis?

Correct answer: Tension headache Explanation The most likely diagnosis is tension headache. Tension headaches are the most common type of primary headache disorders, often referred to as stress headaches. Patients will describe pericranial tenderness, which is typically described as vice-like or tight and is exacerbated by emotional stress, fatigue, noise, or glare. The headaches are usually general, and they are not associated with focal neurological symptoms. Cluster headaches are types of headaches that occur in more commonly in middle-aged men. Patients will complain of severe unilateral periorbital pain that occurs daily for several weeks and potentially is associated with one or more of the following symptoms: ipsilateral nasal congestion, rhinorrhea, lacrimation, and redness of the eye. Patients are restless and agitated during the acute phase of cluster headaches. They more typically occur at night and awaken the patient from sleep. Episodes vary in length from 15 minutes to 3 hours. These occurrences may last for several weeks and then resolve for many weeks at a time. The above patient scenario does not match the description for a cluster headache. Migraine headaches are described as being unilateral in nature and usually pulsatile. They may be accompanied by nausea, vomiting, photophobia, and even phonophobia. An aura, usually visual, will precede the head pain, but migraines can occur without auras as well. The description of the headache in this patient is not similar to a migraine headache. Post-traumatic headaches occur following a closed head injury. This type of headache usually appears 1 or 2 days after the injury; the headache may worsen after a few weeks and will gradually subside. These headaches are described as a constant, dull ache that is sometimes accompanied by throbbing, nausea, or vomiting. Disequilibrium may also be present, and it is worsened by head or body movement. The aforementioned symptoms do not match the patient scenario. Giant cell arteritis, sometimes referred to as temporal arteritis, is a severe inflammatory disorder that is seen in elderly patients. It commonly affects the superficial temporal, vertebral, ophthalmic, or even the posterior ciliary arteries. The headache is typically lateralized to the affected artery side, and it may be accompanied by myalgia, malaise, anorexia, and weight loss. Physical examination may reveal tenderness to palpation over the scalp and/or affected artery. A severe consequence of untreated giant cell arteritis is loss of vision, so prompt diagnosis and medical intervention are warranted.

Case A 37-year-old Hispanic man presents with a 4-month history of mild-to-moderate headaches; on average, he gets them 3-4 days per week. He has tried over-the-counter analgesics with minimal relief. He is seeking care now because he had been promoted to store manager several months prior to presentation, and he is worried that his headaches are affecting his concentration. His headaches are generalized in location, described as starting at the base of his head and extending all over, feeling "tight" in nature. He denies memory loss, photophobia, nausea/vomiting, rhinorrhea, lacrimation, and upper respiratory symptoms associated with the headaches. He also denies seizures, syncope, incoordination, vertigo, weakness, and paresthesias. The patient mentions his concerns for his work several times. Although he enjoys his work, he admits to having some anxiety about being able to handle his new duties. His family history is negative for headache. Physical exam is performed; vitals, neurological, cardiovascular, and HEENT findings are all normal. Question What is the most likely diagnosis?

Correct answer: Tension headache Explanation This patient presents with a most likely diagnosis of tension headache. Tension headaches, the most common type of headache, present as rather diffuse/generalized head pain, without the classic characteristics of the other types of headaches. A component of psychological stress is often present. Cluster headaches are a severe type of headache, most often occurring in middle-aged men, in which the severe headache is accompanied by several other symptoms, such as lacrimation, nasal rhinorrhea, and congestion. The pain is usually localized in the periorbital region. About 1/3 of patients with a brain tumor will present with a headache, but tension-type headaches are far more common in the general population. Headaches secondary to brain tumor can vary in character and intensity. New onset headaches later in life, especially those without a coinciding significant psychosocial stressor, are of greater concern; they have a larger possibility of being caused by an intracranial mass. Nausea and vomiting or neurologic symptoms may accompany headaches caused by a brain tumor. Migraine headaches are also quite common. They are classically described as throbbing in nature and are accompanied by nausea (and possibly vomiting), photophobia (sensitivity to light), and phonophobia (sensitivity to sounds); visual changes and aura are also possible. Migraines are more common in women than men, but they can occur in men. They are usually a chronic condition, beginning in adolescence or the early adult years. Stress can be among many triggers for migraines. This patient's pain pattern and age of onset are inconsistent with migraine, and he lacks the associated symptoms. Subarachnoid hemorrhage (SAH) is classically described by patients as an acute headache that is the "worst headache of my life." There may be preceding trauma and associated nausea and vomiting; nuchal rigidity (due to meningeal irritation) and mental status changes are also seen. This patient's headache history is inconsistent with SAH.

Case A 64-year-old chronic alcoholic presents with a history of confabulation, confusion, and loss of memory. On examination, he is found to have nystagmus, ataxia, and slight bilateral ophthalmoplegia. Question What chronic deficiency is causing his condition?

Correct answer: Thiamine Explanation The patient is suffering from Wernicke-Korsakoff syndrome, a late form of brain damage occurring in long-term alcoholics who rely mainly on alcohol for nutrition and are severely deficient in vitamin B1 (thiamine). In addition to chronic alcoholism, this syndrome can develop in patients with thiamine deficiency from other causes (e.g., protracted IV therapy, severe hyperthyroidism, cancer of the stomach, and after prolonged vomiting). 1 form of the syndrome, Wernicke's encephalopathy, starts suddenly with confusion; this progresses to stupor and death. Patients can be saved by a timely injection of a large dose of thiamine. The other form, Korsakoff's psychosis, features severe memory loss with the inability to store new data, as well as a process of confabulation in which the affected person makes up stories to fill in gaps in memory. Korsakoff's psychosis is usually irreversible and proceeds to profound dementia. The syndrome is named after German neurologist Karl Wernicke and Sergei Sergeivich Korsakoff, a Russian psychiatrist and neurologist. Riboflavin is a constituent of the flavoproteins; these are concerned with tissue oxidation. It is a yellow-green, fluorescent, water-soluble compound. Disorders due to riboflavin deficiency in humans include angular stomatitis, cheilosis, and nasolabial dyssebacia. Other abnormalities include vascularization of the cornea, scrotal dermatitis, a magenta-colored tongue, and anemia. The active form of pyridoxine (vitamin B6) is pyridoxal phosphate, the coenzyme for a large number of different enzyme systems involved in the metabolism of the amino acids, including aminotransferases. Vitamin B6 is widely distributed in plants and animal tissues. Deficiency in humans causes dermatitis, cheilosis, glossitis, and angular stomatitis. Peripheral neuropathy results when the deficiency is associated with isoniazid therapy for tuberculosis. Vitamin B12 is a cobalt-containing porphyrin (cobalamin). Together with folate, it is essential for DNA synthesis; a deficiency of either or both causes a failure of DNA synthesis and disordered cell proliferation. Hemopoiesis is particularly susceptible, and division of cells is delayed and eventually halted. Changes occur in the morphology of bone marrow cells: megaloblastic changes in erythrocyte series, metamyelocytes in granulocyte precursors, and megakaryocytes in other rapidly dividing cells (e.g., the gastrointestinal tract). The deficiency causes megaloblastic anemia. The majority of patients with vitamin B12 deficiencies also have pernicious anemia due to a lack of absorption of intrinsic factor from the ileal cells which have gone through changes due to a deficiency of vitamin B12. Nicotinic acid, which readily converts into nicotinamide in the body, is an essential part of the 2 important pyridine nucleotides, NAD and NADP; they are hydrogen-accepting coenzymes for dehydrogenases at many steps in the pathways of glucose oxidation. NAD is also the coenzyme for alcohol dehydrogenase. A special feature of this vitamin is that it is normally synthesized in the body in limited amounts from the amino acid, tryptophan. A deficiency of niacin causes pellagra, a nutritional disease formerly endemic among poor peasants who subsisted chiefly on maize. The greater part of the niacin in maize is in a bound form, niacytin, which is unavailable to humans.

Question A 22-year-old man presents with a temp of 101°F, a headache, stiff neck, and some limitation of flexion of his neck for the past 2 days. The patient also has generalized myalgia for the past 3 days. The patient was advised rest, adequate hydration, and treatment with antipyretics. The recommended treatment caused the resolution of symptoms. What is the most likely diagnosis?

Correct answer: Viral syndrome with associated muscle spasm Explanation Stiff neck is a common symptom that is often accompanied by local pain and limitation of movement in one or more directions. The most likely cause of a mild elevation of temperature with associated stiffness in the neck is a viral syndrome. The symptoms of viral syndrome include moderate fever, headache, nausea, vomiting, cough, diarrhea, myalgia, and arthralgia. The symptoms usually regress with symptomatic treatment such as rest, adequate hydration, and antipyretics such as acetaminophen or ibuprofen. Cervical muscle spasm can be caused by a number of other conditions including strain, exposure to draught, stress (occupational, visual compensation, or mental), meningitis, tetanus, injury, arthritis, and tumor. In bacterial meningitis, there are overt signs and symptoms of meningeal inflammation, including fever, headache, neck stiffness, and an altered level of consciousness. As the disease progresses, signs of increased intracranial pressure such as hypertension, bradycardia, or coma may appear. In about 50- 60% of patients, signs of meningeal irritation, which include Kernig and Brudzinski, are detected. Focal neurologic signs or seizures occur less frequently. As it is associated with high morbidity and mortality of bacterial meningitis, prompt initiation of antibiotic therapy is essential. Lymphoma causes lymph node enlargement in the cervical region, but rarely is it painful. Pott's disease can involve the cervical spine and cause similar symptoms to meningitis. Pott's disease results from an infection of the spine by the Mycobacterium Tuberculosis bacteria via hematogenous root and/or lymphatic drainage. Pain is the most common presenting symptom and has an insidious onset. It may also be mechanical in nature during the initial stages of the disease. Persistent spinal pain and local tenderness, limitation of spinal mobility, febrile state, and neurological complications including paralysis present as the destruction continues. It requires anti-tubercular therapy; however, these diseases are considerably less common than a viral syndrome. Tetanus also causes neck stiffness, but the initial symptoms include headache and spasm of the jaw (lock jaw). It requires aggressive management and does not resolve by itself. References

Case A 28-year-old man presents following a motor vehicle accident 20 minutes prior. He states that it was just a fender bender, but he feels he might have whiplash. His neck is stiff and sore, and he has developed numbness and tingling on the medial surface of his right arm and into his right fourth and fifth digits. On physical examination, his bicep strength is +5/5 on the left and +5/5 on the right. His biceps tendon reflex is 2+ on the left and 2+ on the right. In addition, his triceps tendon reflex is 2+ on the left and 2+ on the right. His finger extension, wrist extension (ulnar), and grip strength on the right is diminished compared to the left. The remainder of the physical exam is normal. Question Based on the above presentation, what is the cervical nerve root most likely affected?

Correct answer: C8 Explanation The most likely nerve affected in this case is the C8 nerve root. Impingement of C8 may cause numbness and tingling, primarily on the medial surface of the arm and into the ulnar side of the hand into the fourth and fifth digits. Impingement may also cause dysfunction of the hand, as it innervates the small hand muscles. Impingement of the C4 nerve root may cause neck and upper shoulder numbness and pain. Impingement of C5 nerve root may cause deltoid and shoulder numbness and pain, and biceps and brachioradialis tendon reflexes may be diminished. Numbness, if it occurs, is in the lateral arm. Impingement of the C6 nerve root can cause numbness and tingling down the arm into the thumb, with weakness in the bicep muscle and diminished brachioradialis tendon reflex in the affected extremity. Impingement of C7 also causes numbness and pain down the affected arm but into the middle finger, and the triceps reflex may be diminished on exam.

Question A 13-year-old boy is diagnosed with Autism Spectrum Disorder. The boy repeats phrases in a parrot-like fashion; he repeats whatever he hears, but comprehension is absent. What condition is exhibited by the child in the above description?

Correct answer: Echolalia Explanation Echolalia is a pathological, parrot-like, and apparently senseless repetition or echoing of a word or phrase spoken by a person or heard on TV; the repetition/echoing is done without actual comprehension. This is also referred to as echophrasia. Echolalia occurs in autism, catatonic schizophrenia, Gilles de la Tourette's Syndrome, and in transcortical aphasia. Paraphasia is a type of aphasia. The substitution of a similar sounding word for another word is called paraphasia. With paraphasia, the words can also be jumbled. Alexia is the inability to read and understand written language. Alexia is word blindness or text blindness. Alexia is also called optical aphasia or visual aphasia. Apraxia refers to the condition where a patient has difficulty performing a learned motor activity despite having the understanding, muscular capacity, coordination, and normal sensations to do so. A patient with apraxia cannot execute the purposeful movement. A writing disturbance is called agraphia. It is the inability to express thought in written language that is not due to mechanical dysfunction.

Case A 70-year-old woman is brought to your attention because of the slowly progressive gait disorder; she is also experiencing forgetfulness and problems with micturition. About 1 year ago, she started having weakness and tiredness in her legs, followed by unsteadiness and her steps became shorter and shorter. Sometimes, she forgets where she put things; she has forgotten to turn off the oven 2 times. Over the last month, she has started experiencing urinary urgency, the need for frequent urination, and the involuntary leaking of urine that worries her. She notes no other symptoms. Question What is the most likely cause of her urinary problems?

Correct answer: Detrusor muscle instability Explanation The correct response is detrusor muscle instability. A clinical triad of slowly progressive gait disorder, followed by impairment of mental function and sphincteric incontinence, highly suggests the presence of normal-pressure hydrocephalus. Ventricular expansion in hydrocephalus involves frontal horns, affecting the functions of frontal lobes and of basal ganglionic-frontal motor control. The central white matter is affected more than cortical gray matter, thalami, basal ganglia, and brainstem. The distortion of the central portion of the corona radiata by the distended ventricles affects the sacral motor fibers that innervate legs and the sacral bladder center, causing the abnormal gait and incontinence. Decreased inhibition of bladder contractions leads to the instability of bladder detrusors. In cases of normal-pressure hydrocephalus, the formation of CSF equilibrates with absorption; there might be variations in the CSF pressure, but overall pressure is considered normal. A urinary tract infection will probably manifest with a strong, persistent urge to urinate, a burning sensation when urinating, and passing frequent, small amounts of urine that has unusual smell. Its appearance may be cloudy; it may be red, pink, or cola-colored if there is blood in the urine. Dementia is a consequence of the distortion of the periventricular limbic system and frontal lobes in the cases of normal-pressure hydrocephalus. However, incontinence is only a result of a lack of concern for micturition in extremely advanced cases, in which there is frontal lobe dysfunction. Your patient has insight into her problems and worries; therefore, frontal lobe dysfunction resulting in a lack of concern for micturition is probably not the cause of her urinary problems. Pelvic sphincter weakness will probably manifest as stress incontinence; incontinence happens when a weak sphincter cannot hold the urine and physical pressure is placed on the urinary system, such as when a patient coughs, sneezes, or laughs.

Case A 54-year-old woman with history of type II diabetes mellitus presents with a new foot ulcer. She has minimal pain at the site. She noticed a small fissure on the ball of her foot about 2 weeks ago, and she discovered yesterday that the lesion had become larger and was draining purulent smelling material. Her foot has also become swollen and red. She has been on hypoglycemic medication for the last 5 years; recently, her metformin dose was increased to 850 mg twice daily; it is combined with glipizide 10 mg twice daily. She is also on benazepril 10 mg daily for mild hypertension and microscopic proteinuria discovered at her annual physical earlier this year. Physical exam reveals a temperature of 99.1°F, pulse 72/min, and respirations 20/min; blood pressure is 123/84 mm Hg. The plantar surface of the right foot has a 2 cm shallow ulcer with a film of purulent, serous drainage. There is a 1 cm surrounding band of erythema around the ulcer margin, and her foot has 2 + edema. She has greatly decreased pinprick sensation in both feet, and she cannot feel vibration. Dorsalis pedis pulses are 4/5 bilaterally. Laboratory evaluation reveals a WBC count of 17.7 X 103/ìL, with 93% neutrophils. Her HbA1c is 8.8%, and glucose is 241 mg/dL. Question What could have been done to prevent this wound (and future wounds) from forming in this patient?

Correct answer: Education and risk factor prevention Explanation The optimum therapy for foot ulcers is prevention, which is done through identification of and education for high-risk patients. This patient has several indicators of poor diabetic control: her elevated HbA1c, obvious lack of insight as to the importance of careful daily foot surveillance, documented microscopic proteinuria, and evidence of diabetic neuropathy on physical exam. The feet of all diabetic patients are to be examined by a physician at every office visit, or at least twice a year; they should be examined daily by the patient. Patients should be very careful in selecting footwear because diabetic neuropathy can affect their ability to feel that shoes are ill-fitting. Shoes should fit well and have a wide toe box; diabetics should not wear shoes with open toes or heels. In order to minimize the risk of pressure ulceration, special orthotic shoes should be prescribed if the patient has any deformity to their feet. Diabetic patients should wash their feet daily in lukewarm water; they should use mild soap and dry their feet gently and thoroughly. Toenails should be cut straight across and not too close to the nail bed. Nail care should be referred to a podiatrist if the patient has problems performing it safely. Adhesive plasters or tapes should not be used on the feet of diabetics, nor should any other items that could damage the skin (e.g., corn pads, electric heating pads, and harsh chemicals). Walking barefoot should be avoided so as to lessen the risk of injury to the foot. Despite preventative measures, diabetic foot ulcers and amputation are common problems; they often require a multi-disciplinary approach, including orthopedics, vascular surgery, endocrinology, podiatry, and infectious disease specialists. Diabetes is the leading cause of non-traumatic lower extremity amputation in the United States. Improvement in glycemic control is very important, as is evaluation of the other risk factors for vascular disease (i.e., tobacco abuse, hypercholesterolemia, and hypertension). All of the other answer choices are poor ideas for wound prevention. Hot water soaks and walking barefoot increase the risk of developing foot ulcers. Amputation need not be considered yet in this patient, since she has not yet tried more conservative therapy. Prophylactic antibiotics are not indicated; they would increase her risk of developing drug-resistant bacteria in her skin flora.

Case Your patient is a 77-year-old man complaining of stiffness and slowness that started several months ago and is more pronounced on the left side of his body. Neurological examination reveals tremor in rest, bradykinesia, cogwheel rigidity in his neck and limbs, and postural instability. His face is expressionless (mask-like) and he has a stooped posture; when he walks, there is mild retropulsion and no arm swings. Question What transient factor would you also expect to see?

Correct answer: Freezing of gait Explanation Your patient most probably has Parkinson disease or parkinsonism (tremor in rest, rigidity, bradykinesia, postural instability). Parkinsonian gait is characterized by the absence of arm swings, small shuffling steps, hypokinesia, and reduced walking speed. The patient has difficulty starting and stopping after starting. Freezing of gait is most often seen in the advanced stages of the disease. It is typically transient and lasts less than a minute, during which gait is halted and the patient feels that his/her feet are glued to the ground. When the patient overcomes the block, walking can be performed relatively smoothly. The most common form of freezing is "start hesitation", which is when the patient wants to start walking. It is provoked by placing a patient in a narrow space or in stressful situations, such as in the doctor's office. Later in the course of the disease, it can appear spontaneously. Titubation is seen when the patient's trunk moves from side to side as he/ she walks; this is called truncal ataxia. It is a sign of cerebellar disease (usually the midline of cerebellum in cerebellar degenerations). Spastic gait is characteristic of upper motor neuron disease. It is not transient. Hemiparetic gait involves 1 leg. In a hemiparetic gait, the spastic stiff leg is circumducted (the leg swings in a semicircle from a medial to lateral direction) and the toe hits the ground before the heel does. Often, the arm on the paretic side does not swing and is flexed at the elbow. Spasticity causes the extremity to be stiff. It is not transient. In paraparetic spastic gait, both legs are stiff and the arms are moving much more than the legs. Thighs are adducted, giving the picture of a "scissors" gait. A slapping gait occurs in lower motor neuron disease or poor position sense in the feet. With a foot drop, the patient flexes the hip so that the foot will clear the floor. Patients with poor position sense have a wide-based gait; their feet slap the floor, and they require visual assistance to walk. Chorea is a disorder characterized by irregular, jerky, rapid, uncontrolled, and involuntary movements that seem to move randomly from 1 part of the body to another. Walking is bizarre, with inserted excessive postures and leg movements.

A 12-year-old girl presents with a 3-day history of progressive dysarthria, dysphagia, and weakness. The patient was well until 3 days ago, when she developed the onset and subsequent gradual worsening of dysarthria. She attributed this to a sore throat, which she had about 2 weeks before. Three days prior to presentation, she had the onset of mild dysphagia, which occurred primarily with liquids. 24 hours prior to presentation, she developed weakness in both upper extremities; the weakness worsened and began to involve the lower extremities. This limb weakness was not worsened by activity, but it was not improved by rest. She developed tingling in her toes 24 hours prior to presentation. When she became unable to walk without assistance (on the day of presentation), she decided to seek medical care. Past medical history is significant for measles and mumps. Because of family religious beliefs, she has not had any immunizations. Physical examination reveals a well-developed, well-nourished girl. She is awake, alert, cooperative, and in no acute distress. Temperature is 98.7 F by mouth, blood pressure is 140/80 mm Hg, heart rate is 84/min and regular, and respirations are 22/min and unlabored. Signs of trauma include multiple scratches and abrasions over most of her extremities; they are in varying stages of healing. Speech is moderately dysarthric. She experiences some mild choking when she tries to drink a glass of water. She can smile weakly, but cannot raise her eyebrows against resistance. She shows mild bilateral weakness of eye adduction. Pupillary responses are normal. There is mild to moderate upper extremity and mild lower extremity weakness, greater distally than proximally. Her motor strength is sustained over at least 30 seconds without fatigue. Reflexes are hypoactive to absent, and the response to plantar stimulation is downgoing bilaterally. Sensation is intact, except for mildly impaired position and vibratory sensation in both feet. A complete blood count, chemistry profile, chest X-ray, and EKG are all found to be normal. Computed tomography of the brain, with and without contrast, is negative. Question What is the most likely diagnosis?

Correct answer: Guillain-Barre Syndrome Explanation Guillain-Barre is the most common cause of acute and subacute generalized paralysis. In 60% of cases, it is preceded by mild respiratory or gastrointestinal infection 1-3 weeks before the onset of paralysis (1). Although it more typically begins distally and spreads proximally, the pattern seen here is a well-known variant. While classically described as a motor neuropathy, varying degrees of sensory neuropathy can be seen, as evidenced here by her paresthesias and decreased position and vibration sensation. Reduced or absent deep tendon reflexes are usual findings. Oculomotor nerves are paralyzed in only the most severe of cases; even the pupils may be unreactive. Diphtheria of the faucial-pharyngeal type produces an inflammatory exudate in the throat and trachea. Neurological sequelae usually begin after about 5-12 days, however, and usually with soft palate paralysis, which may produce dysphagia but also regurgitation and nasal (dysphonic) speech (1). Soon, trigeminal, facial, vagus, and hypoglossal nerves become involved. Loss of accommodation and blurring of vision (with preserved light reaction) usually appears by weeks 2-3. External ocular muscles are rarely involved. While cranial nerve findings may resolve around this time, the patient may go on to develop a mixed sensorimotor peripheral neuropathy at 5-8 weeks into the clinical course. Although this peripheral neuropathy may resemble Guillain-Barre, the other symptoms mentioned above and this long time delay, are not seen in this patient. Botulism is a rare form of blood borne illness caused by the exotoxin of C. botulinum (1). This exotoxin acts at the presynaptic endings of neuromuscular junctions, interfering with acetylcholine release. Outbreaks of botulism are usually due to ingestion of home preserved foods, especially vegetables and cured ham (1). There is no indication of an outbreak in this case. Rarely, botulism can be due to contaminated wounds (she played in horse fields and had multiple skin lesions). Symptoms usually come on within 12-36 hours (not unlike this patient), but they usually consist of nausea, vomiting, and anorexia. Generally, the first neurologic symptoms are blurring of vision and diplopia. These symptoms are rapidly followed by other cranial nerve findings, weakness of neck, trunk, upper then lower extremity weakness, and respiratory insufficiency. While the clinical course of botulism may be confused with Guillain-Barre, sensory loss would not be expected in botulism. Multiple Sclerosis of an acute and fulminant nature could be considered. The symptoms did come on rather suddenly, but their clear and rapid progression, with all symptoms being bilateral and symmetric, would be most unusual. While a negative computed tomographic scan does not fully rule out MS, in a patient with such dramatic and extensive bulbar findings, some brainstem or bilateral cerebral computed tomogram lesions would be expected. For the same reason, normal pupils would be most unusual. Also, with such widespread findings, Babinski signs would be expected. The deep tendon reflexes, if changed, would be increased. Tetanus might come to mind in a patient with skin injuries associated with playing in fields contaminated by horse feces (a common source of Clostridium tetani), who has had no tetanus immunizations, and who has the onset of bulbar symptoms first, with subsequent descending motor symptoms. However, while all the other listed diseases produce weakness, tetanus produces a progressively severe motor irritability that usually begins in the jaw with trismus. In some cases, it begins with a feeling of stiffness in the jaw and neck, slight fever, aches, and pains. This progresses to spasm as it spreads to other cranial nerve muscles, the neck, trunk, and extremities. This proceeds to tonic rigidity and tonic spasms that can be spontaneous or induced by the slightest stimulation. Certainly, sensory symptoms or signs would not be expected. Deep tendon reflexes would be increased. Myasthenia Gravis can be considered since the patient appears to a have a motor weakness that began in bulbar musculature then spread distally (distended). This pattern of weakness can be seen in myasthenia gravis, but it is also seen in a relatively common variant of Guillain-Barre. However, in myasthenia gravis, the weakness is more proximal than distal, which is the opposite of the more distal severity expected in neuropathies like Guillain-Barre. Myasthenic weakness tends to improve after rest and worsen with exercise, a pattern not seen in this patient. Again, sensory findings would not be expected in myasthenia; also, they would not be expected in a patient of this age.

Case Ico-delete Highlights A 12-year-old girl presents with a 3-day history of progressive dysarthria, dysphagia, and weakness. The patient was well until 3 days prior to admission to the hospital; at that time, she developed the onset and subsequent gradual worsening of dysarthria. She attributed the dysarthria to a sore throat that she had had about 2 weeks earlier. 3 days prior to admission, she also had the onset of mild dysphagia; it mostly occurred with liquids. 24 hours prior to admission, she developed weakness in both upper extremities, which increased and began to involve the lower extremities. This limb weakness was neither worsened by activity nor improved by rest. She also developed tingling in her toes 24 hours prior to presentation. When she became unable to walk without assistance on the day of admission, she decided to seek medical attention and was admitted to the hospital. Past medical history is significant for measles and mumps. Because of family religious beliefs, she has not had any immunizations. She is very athletic, and frequently plays soccer with friends and siblings in the fields on her grandfather's horse farm. Physical examination reveals a well-developed, well-nourished girl. She is awake, alert, cooperative, and in no acute distress. Temperature is 98.7 F by mouth, blood pressure of 140/80 mm Hg, heart rate is 84/min and regular, and respirations are 22/min and unlabored. There are multiple scratches and abrasions in varying stages of healing over most of her extremities. Her speech is moderately dysarthric. She experiences some mild choking when she tries to drink a glass of water. She can smile weakly, but she cannot raise her eyebrows against resistance. She shows mild bilateral weakness of eye adduction. Pupillary responses are normal. There is mild to moderate upper extremity and mild lower extremity weakness, greater distally than proximally. Her motor strength is sustained over at least 30 seconds without fatigue. Her gait is ataxic, and she cannot walk without assistance. Reflexes are hypoactive to absent, and the response to plantar stimulation is downgoing bilaterally. Sensation is intact, except for mildly impaired position and vibratory sensation in both feet. A complete blood count, chemistry profile, chest X-ray, and EKG are all normal. Computed tomography of the brain, with and without contrast, is negative. A nerve conduction study reveals a moderate degree of mostly motor demyelinating peripheral neuropathy, highly suggestive of Guillain-Barre. Question What statement best describes the patient's prognosis?

Correct answer: Her rapidly evolving clinical course indicates a poor prognosis Explanation Her rapid and continuing worsening over 3 days suggests a relatively poor prognosis. At the onset, the typical case of Guillain-Barre Syndrome (GBS) includes paresthesias and slight numbness in the fingers and toes. Clinically, the main sign is generally symmetrical weakness that evolves over several days to a week or two. In 5% of patients however, weakness progresses to rapid deterioration. Respiratory failure and total motor paralysis may occur within a few days. (4) This patient must be watched very closely for the very real possibility of respiratory failure and the need for ventilatory support (2). Mortality is expected to be less than 5% with good medical support (1). With a demyelinating pattern on EMG, her prognosis is better. A consistent indicator of residual muscle weakness is an EMG pattern of axonal damage, with the more severe degrees of damage suggesting the worse prognosis (1). About 85% of patients with GBS have a full functional recovery within a year; however, some may be left with minor residuals such as areflexia on exam (2). Between 5-10% of patients with Guillain-Barre have 1 or more relapses. These cases are referred to as Chronic inflammatory demyelinating peripheral neuropathy (CIDP) is a type of GBS known to be slowly progressing or relapsing (1).

Case A 3-year-old boy presents following a car accident. He had not been buckled in, and he was thrown against the windshield. When the paramedics arrived at the scene, he was crying and clinging to his mother. Upon arrival at the emergency department, he is sobbing and does not react to her anymore. He sustained lacerations to the face and both arms. His blood pressure is 150/84, pulse 62, and the pupils react sluggishly. He also has a temperature of 39.2°C. Question What do his symptoms indicate?

Correct answer: Increased intracranial pressure Explanation This child most certainly sustained head injuries with increased intracranial pressure in the accident. Since he was not buckled in, it is very likely that his face and arms were cut when he was thrown against the windshield. His pupil reaction, disorientation, and not recognizing his mother all support that theory. Elevated temperature is common in patients with head injuries and is called central fever. The only symptom that points to viral infection is the fever, but there would not be any changes in pupil reaction. Reye's syndrome is caused by aspirin medication in the presence of a viral infection. It affects the central nervous system, but it is not related to trauma. A child may cry and be very anxious in a hospital setting, but that would not cause temperature elevation and sluggish pupils. Guillain-Barré syndrome, or acute idiopathic polyneuritis, is a rapidly progressing ascending motor neuron paralysis of unknown etiology, which often follows viral infections.

Case A 32-year-old man presents with a severe headache; he has had 2 similar headaches within the past week. He describes a burning, 'hot poker'-type of pain located primarily behind his right eye. He notes that his eye waters profusely with the headache; in addition, his nose is initially congested, then it starts running. Only his right side is affected. The headache is so severe that he cannot work or sleep through it, and he is unable to concentrate on anything else. The headaches have been unresponsive to over-the-counter pain medications. The episodes seem to last about 1 hour. He denies any other symptoms. This patient has no chronic medical conditions, and he takes no regular medications. Question What is the most likely underlying pathophysiology of this patient's condition?

Correct answer: Increased trigeminal nerve and parasympathetic activity, leading to vasodilation Explanation This patient presents with a history indicating cluster headaches. While the exact mechanism is unclear, it is known that there is increased trigeminal nerve and parasympathetic activity, leading to vasodilation in the intracranial vasculature. Key features of cluster headaches include unilateral pain, involvement of the eye and nose (autonomic/parasympathetic system), an episodic pattern of attacks, and excruciating pain. Antigen binding to IgE, leading to release of histamines, leukotrienes, and prostaglandins is the typical type I hypersensitivity reaction of allergies. Seasonal allergies could cause a headache, along with nasal and ophthalmic symptoms; however, this patient's severity of headaches exceeds a usual allergic sinus headache. If this patient's symptoms had been caused by an allergic response he would have presented with a history of allergies, chronic lower-grade headaches, and sinus symptoms; he would not have acute, episodic attacks of high intensity. While the growth of an intracranial tumor could present with headaches, this patient's history is not suggestive of cerebellar dysfunction. If there were a tumor in this patient's cerebellar region, some motor, and possibly language, dysfunction would be expected. Headaches with lacrimation and rhinitis are not typically seen from a tumor. Increased emotional stress and tight neck musculature is associated with tension-type headaches, which are the most common form of headaches. Typically, these headaches are more chronic; they are associated with psychosocial stressors and respond to analgesics. Nasal and ophthalmic symptoms are not seen with tension-type headaches. The reactivation of the varicella zoster virus (shingles) can produce severe throbbing, stinging, or other pain symptoms along the affected dermatome. Shingles would be a disorder to consider on this patient's differential, but the patient's young age, lack of skin lesions, and the episodic nature of the attacks fit much better with a diagnosis of cluster headaches.

Question Ico-delete Highlights A 5-year-old girl presents with a 5-day history of fever, coughing, sneezing, and watery nasal discharge. According to her mother, she passed out and began shaking yesterday. 1 hour before that, her temperature peaked at 40.1 °C (104.2 F). The episode lasted for 1.5 minutes, and she remained drowsy for about an hour afterwards. 1 year ago, she suffered a similar shaking episode during a bout of otitis media. The child reached all developmental milestones at appropriate ages, received all immunizations, and has no other relevant past medical or family history. Her only medication is acetaminophen. Vital signs are as follows: BP 110 / 70 mm Hg, HR 86 bpm, RR 20 rpm, and temperature 37.3 °C (99.1 F). On physical examination, there is some wheezing in both lung fields. The remainder of the examination, including a full neurologic exam, was normal.

Correct answer: No further workup is needed Explanation About 4% of all healthy children in the United States suffer at least 1 febrile seizure. Most occur in the first days of a benign febrile illness, usually a self-limited viral or bacterial disease. The overwhelming majority of febrile seizures are self-limited events. About 2-4% of patients with a febrile seizure eventually develop epilepsy in later childhood and adult life. A rare complication is status epilepticus. The post-convulsion EEG (postictal EEG) is always normal. However, patients suffering 1 febrile seizure are at a higher risk for subsequent events: 1/3 of patients with 1 episode suffer recurrences, and children who suffer a 2nd episode have a 50% chance of having further febrile seizures. Risk factors include: High temperature. Family history of febrile seizures. Daycare attendance. Neonatal discharge from hospital after 28 days. Parental report of developmental delay. Routine laboratory tests (e.g. serum chemistry, brain imaging) are seldom useful. A spinal tap should be done if there is suspicion of meningitis and in children under 12 months, but it is not warranted in this patient. In most cases, fever reduction using antipyretics or sponging is all that is necessary. If fever persists, prophylactic anticonvulsants (e.g. diazepam, phenobarbital or valproate) can be used. Diazepam and barbiturates can cause somnolence, particularly after a loading dose is given. Valproate is contraindicated in the presence of vomiting and acidosis, both of which are common in children with acute febrile illnesses. Phenytoin and carbamazepine are not effective in the prophylaxis of febrile seizures.

Case A 46-year-old man presents with severe insomnia and anxiety. While hospitalized, an overnight polysomnogram was performed over 2 consecutive nights. Sleep latency was 60 minutes; REM latency was 45 minutes. He reports feeling paresthesias deep within his legs while lying in bed, especially while falling asleep. He denies recent illness or illicit drug use. His physical exam and lab work are within normal limits. Question What is the most likely cause of the patient's symptoms?

Correct answer: Restless legs syndrome Explanation Restless legs syndrome (RLS) is a neurological disorder with symptoms of unpleasant sensations in the legs, such as insects crawling inside the legs, burning, tugging, or creeping. There is an uncontrollable urge to move the limb when at rest (lying down and trying to relax activates the symptoms). Most people with RLS have difficulty falling asleep and staying asleep. Women may be slightly more affected than men. Symptoms may begin at any stage of life, although the disorder is more common with increasing age. The severity of the disorder appears to increase with age. In some patients, symptoms will improve over a period of weeks or months. In most cases, the cause of RLS is idiopathic. A family history of the condition is seen in many cases, suggesting a genetic component. People with familial RLS tend to be younger when symptoms start, and they have a slower progression of the condition. Hypoglycemia can worsen the condition. The disorder is diagnosed clinically by evaluating the patient's history and symptoms. Needle electromyography and nerve conduction studies should be considered if polyneuropathy is suspected on clinical grounds, even if results of neurologic examination appear normal. Ropinirole is approved for the treatment of moderate-to-severe RLS. Benzodiazepines may be prescribed for patients who have mild or intermittent symptoms. These drugs help patients obtain a more restful sleep, but they do not fully alleviate the symptoms and can cause daytime drowsiness. For more severe symptoms, opioids may be prescribed since they can cause relaxation and decrease pain. Anticonvulsants, such as carbamazepine and gabapentin, are also useful for some patients; they can decrease the creeping and crawling sensations. There may be no symptoms associated with deep vein thrombosis (DVT), but the classical symptoms of DVT include pain, swelling, redness of the leg, and dilation of the surface veins. Peripheral neuropathy, of which alcoholic peripheral neuropathy is a type, causes tingling or burning pain in the feet. At times, it may be so severe that it interferes with walking, which is a result of injury to sensory fibers. As the condition worsens, the pain typically decreases and numbness increases. Periodic limb movements in sleep are associated with periodic episodes of highly repetitive limb movements during sleep. These repetitive episodes of muscle contractions are usually grouped into series.

Case Ico-delete Highlights A 50-year-old man was found unconscious in a park. On physical examination, his vital signs are a pulse 90/minute, blood pressure 140/90mmHg, respirations 24/minute. He appears disheveled and smells of alcohol. Noisy respirations with a weak gag reflex are noted. A bottle of wine is found in his pocket, he has an old scar on his forehead, and there is a 2-week-old sutured laceration in his scalp. His right pupil is 6 millimeters and dilated. His left pupil is 3 millimeters. He responds only to noxious stimuli, and he moves the left side of his body more than the right. Question What is the next step in the management of this patient?

Correct answer: Secure an airway Explanation The initial step would be to check airway, breathing, circulation and the cervical spine, and disability as part of the primary survey. Airway needs to be evaluated for patency. The tongue is the most common item that obstructs airway in unconscious patients. Always maintain inline cervical spine stabilization. Suction is applied to remove blood, dental, and mucus fragments. To open the airway, one can use the jaw-thrust maneuver in order to maintain cervical spine stabilization. There are a variety of airway devices that enable the physician to maintain the nasopharyngeal and oropharyngeal airways. Repeat checking all 4 vitals again. Obtain intravenous access, apply a cardiac monitor, disrobe the patient, and do a secondary survey to evaluate for injury. This survey would include checking pupillary response, the level of consciousness, and seizure activity. The next few steps would then include a combination of the following: doing a finger stick for estimation of blood glucose, which should followed by administration of 50 milliliters of 50% dextrose solution after giving thiamine 100 milligrams as an intravenous push (particularly in alcoholics) if the blood glucose is found to be less than 80 mg/dl. If the patient fails to respond, then naloxone 2 milligrams as an intravenous push should be administered (give a small amount if 'track lines' are evident on the patient). This could be followed by giving charcoal to the patient and/or completing the remainder of the physical examination; it should include a mental state examination to evaluate memory, orientation, perceptions, insight, intellectual function, and affect. Following this, and if available, one would then obtain history from other people present who may know the patient. Questions should include what the patient was like before becoming unconscious, what the normal baseline mental status is for the patient, whether there was a rapid or slow change in mental status, whether there is any deterioration since seeing the patient, any history of trauma, and what medications or street drugs the patient could have taken.

Case A 67-year-old man is brought in with a 4-month history of tremors, blurred vision, and repeated falls. AN MRI performed at that time showed diffuse cerebral and brainstem atrophy. Two months ago, another healthcare provider prescribed levodopa in progressively higher doses; it afforded only marginal, transient improvement. Two weeks ago, he began to suffer visual hallucinations, confusion, occasional contractures in his neck, and choreoathetoid movements in his upper limbs. His past medical history includes essential hypertension that is well controlled with hydrochlorothiazide and an uneventful cholecystectomy 30 years ago. The patient is a retired accounting clerk and he led a fairly active life before the symptoms began. Currently, he is using near-maximal doses of levodopa/carbidopa. Vital signs are within normal range. On physical examination, he is confused and disoriented to time and place. There is a large bruise on his back from when he fell backward over the living room table. His posture is rigid, with hyperextension of the trunk and a low-amplitude, high-frequency tremor in both hands, as well as a coarser tremor in his right arm. He has difficulty following the examiner's finger with his eyes, particularly during horizontal deviation. His gait is unstable; he sometimes has to support himself against the walls to avoid falling. The remainder of the neurological and physical examinations shows no abnormalities. Question What is the most appropriate next step?

Correct answer: Start tapering the levodopa dose. Explanation Parkinsonism poorly responsive to levodopa suggests a diagnosis other than Parkinson's disease. Some possibilities are certain forms of secondary Parkinsonism (e.g., from atherosclerosis or multiple trauma) and atypical Parkinsonian syndromes (also called Parkinson-plus syndromes), such as corticobasal degeneration, progressive supranuclear palsy, frontotemporal dementia, dementia with Lewy body disease, and multiple system atrophy. This patient's hyperextended posture, backward falls, and ocular signs are strongly suggestive of progressive supranuclear palsy (PSP, or Steele-Richardson-Olszewski syndrome). Alien limb phenomenon, apraxia, and cerebellar signs may also be seen in this condition. The deficits in ocular motility are often perceived as blurred vision by the patient. This condition progresses more quickly than Parkinson's disease and it usually has a fatal outcome in 5-6 years. While the dramatic response to levodopa seen in Parkinson's disease does not occur, some patients improve transiently, and a trial with doses higher than those used for Parkinson's is warranted. If there is little or no response, or intolerable side effects develop (e.g., dystonia, hallucinations, delusions), the dose is slowly tapered (abrupt withdrawal can worsen the motor symptoms) and withdrawn. Small doses of levodopa sometimes produce modest functional improvement. This patient's hallucinations and dystonias are probably caused by the high levodopa/carbidopa doses, and they should improve after tapering the drug. Specific treatment with antipsychotics is therefore not indicated, and the treatment might worsen the patient's already severe movement disorders and postural instability. Amantadine is useful as an adjunct to levodopa in patients with Parkinson's disease, and it is effective at reducing levodopa-induced dyskinesia. However, antiparkinsonian therapy yielded mediocre efficacy and is causing severe side effects in this patient; therefore, it should be discontinued. For the same reasons, anticholinergics such as benztropine are not useful.

Question Ico-delete Highlights A 55-year-old man with a history of heavy alcohol use presents for treatment of an infected gash in his hand. He is not currently intoxicated. His clothes fit loosely, and he mentions that he thinks he has lost weight since becoming homeless earlier in the year. In preparation for giving him antibiotics, he receives an intravenous infusion of 5% dextrose solution. 2 hours later, he is confused and ataxic, with bilateral 6th nerve palsies (intranuclear ophthalmoplegia) and nystagmus. What is the patient most likely exhibiting?

Correct answer: Wernicke encephalopathy Explanation Ablation or destruction of the limbic areas highlights the role of limbic areas in the processing and storage of memory. Because of the protection afforded by the redundancy of having 2 hemispheres with overlapping functions, tumors, injuries or infarctions affecting limbic areas are often silent unless they cause pressure effects or electrical excitation. Toxic or metabolic damage, which typically affects symmetrical areas, can have a dramatic impact by contrast. The most common, clinically significant example is that of thiamine deficiency, which occurs in malnutrition, especially among alcoholics. Acute thiamine deficiency produces Wernicke encephalopathy. The cardinal features of this condition are delirium with ataxia, oculomotor dysfunction (especially 6th nerve palsy- intranuclear ophthalmoplegia), and sometimes anisocoria. Because thiamine is a cofactor for the cerebral metabolism of glucose, administering glucose to someone on the verge of thiamine deficiency can precipitate acute encephalopathy. Wernicke encephalopathy is prevented or rapidly reversed by administration of thiamine. Chronic thiamine deficiency causes alcohol-induced persisting amnestic disorder (Korsakoff's syndrome). The main sites of damage are paraventricular, symmetrical lesions in the mammillary bodies, the thalamus, and the hypothalamus. Lesions may also occur in the midbrain, pons, medulla, fornix, and cerebellum. Cerebellar and ocular signs may not be present. Diagnosis depends on a careful mental status examination. Patients with Korsakoff's syndrome have relatively preserved long-term memory, are socially appropriate, use language normally, and can do simple calculations. Their profound recent memory loss seems so disproportionate to the rest of their mental status examination that observers may misdiagnosis malingering, i.e., willfully faking abnormality for personal gain. Patients with Korsakoff's syndrome will occasionally try to conceal their deficits by confabulation, making up or agreeing to untrue scenarios given by the examiner. This is not the same thing as lying, but it is often mistaken for it. The mild macrocytosis is a common incidental finding in chronic alcoholics, but it does not account for the observed behavioral changes.

Case Ico-delete Highlights A 16-year-old boy presents with acute onset of stiff neck, fever, headache, and vomiting. On exam, he appears lethargic, has limited range of motion of his neck, and a petechial rash is noted. Fever is 103°F. The patient's medical history is non-contributory. Question What is the most likely etiology of the suspected diagnosis?

orrect answer: Meningococcus Explanation The correct response is meningococcus. Meningitis is an inflammation of the membranes covering the brain and spinal cord. The most common causes of meningitis are bacterial infections that start in other parts of the body and spread to the brain or spinal cord via the bloodstream. Meningitis can be caused by bacteria, viruses, chemical irritation, or tumors. Symptoms include fever, severe headache, nausea and vomiting, stiff neck (nuchal rigidity), photophobia, and mental status changes. Complications include hearing loss, brain damage, loss of vision, and possible deafness. Meningococcal meningitis is a life-threatening infection caused by the Neisseria meningitidis bacterium. Most cases occur in children and young adults. The onset is often abrupt, and early treatment is essential to achieving reduced morbidity and mortality. In addition to the usual symptoms, a petechial rash (non-blanching, red-brown spots on all parts of the body) is characteristic and caused by capillary leak and platelet dysfunction. Routine vaccination against meningococcus is recommended for adolescents in the United States; the first dose at age 11 or 12, and a booster dose at age 16. If this patient has not had routine medical care in many years, then he will not have been vaccinated against this disease. Enterovirus is a term given to cases of meningitis caused by viruses. Enteroviruses, such as the Coxsackie and echovirus, account for the majority of cases of viral meningitis. Other viruses that can cause meningitis include mumps, varicella, and the human immunodeficiency virus. Viral meningitis is usually milder than that caused by bacteria; symptom onset is more gradual, and patients usually recover with conservative management. Syphilitic meningitis is a progressive, life-threatening complication of untreated syphilis infection. In addition to the usual symptoms of meningitis, patients may present with prominent mental status changes and focal neurologic deficits. Serum VDRL or RPR along with CSF (cerebrospinal fluid) examination is diagnostic. Pneumococcal meningitis is caused by the bacteria Streptococcus pneumoniae. Meningitis occurs as a consequence of bloodstream infection with subsequent invasion of the central nervous system. Patients often present with a history of recent pneumonia, upper respiratory infection, infective endocarditis, or ear infection.

Case Ico-delete Highlights A 73-year-old man presents with sudden onset of vomiting, headache, reduced vision, numbness, and tingling of the extremities. He also has difficulty swallowing and speaking, as well as loss of coordination and balance. There is no history of trauma, fever, or chills. His past medical history is significant for amyloid angiopathy. His pulse is 64 bpm, blood pressure is 140/88 mm Hg, and respirations are 16/min. Question What is the most likely diagnosis?

Correct answer: Lobar intracerebral hemorrhage Explanation Lobar intracerebral hemorrhage occurs when there is bleeding into the parenchyma of the cerebrum, which is caused by the rupture of a blood vessel. Lobar intracerebral hemorrhage may be caused by trauma (brain injury) or abnormalities of the blood vessels; these include aneurysm, arteriovenous malformation (AVM), or angioma (tumors involving blood vessels). Amyloid deposits in the blood vessels (amyloid angiopathy) may also cause cerebral hemorrhage; this is a condition found more commonly in the elderly. Other risks for lobar intracerebral hemorrhage include bleeding disorders, the use of aspirin or anticoagulant medications, liver disease, and certain autoimmune disorders. Symptoms of intracranial hemorrhage develop suddenly, often during activity. Symptoms include a change in alertness or level of consciousness, sudden onset of vomiting, headache, vision changes, decreased sensation (numbness and tingling), difficulty swallowing, difficulty speaking or understanding speech, difficulty writing or reading, and loss of coordination and balance. CT scan of the head is an effective tool in early diagnosis. Treatment goals may include supportive measures and lifesaving interventions (e.g., intubation and hyperventilation). An epidural abscess is a disorder characterized by inflammation and collection of pus between the dura and the bones of the skull or spine. The infection is usually bacterial (Staphylococcus is common). Symptoms include deep, localized tenderness in the ear or back, headache (progressively worsening), fever, excessive sweating, general discomfort or uneasiness (malaise), as well as nausea and vomiting. A cerebral abscess is a space-occupying lesion in the brain caused by an inflammation and a collection of infected material within the brain tissue. Symptoms include headache; stiff neck, shoulders, or back; vomiting; changes in mental status; seizures; fever; and focal neurologic deficits. Once symptoms occur, they progressively worsen. A neurologic examination shows losses that correlate to the location of the abscess. A spinal cord abscess is a rare disorder characterized by inflammation and a collection of infected material between the spinal cord and the spine (epidural). Symptoms include fever, chills, lower backache, sudden or rapid progression of neurologic symptoms, loss of movement of an area of the body, weakness or paralysis, loss of sensation of an area of the body, localized numbness, and loss of bladder or bowel control. Male patients commonly experience impotence. An examination often shows localized tenderness over the spine. A neurologic examination may indicate spinal cord compression, with involvement of the lower body (paraplegia) or the entire trunk, arms, and legs (quadriplegia). The extent of neurologic losses corresponds with the location of the lesion. A subdural hematoma involves the collection of venous blood in the space between the inner and the outer membranes covering the brain, generally following trauma. Symptoms include loss of consciousness, drowsiness, headache, impaired vision, enophthalmos, unilateral eyelid drooping, decreased sensation or numbness, unilateral paralysis, confusion and delirium, changes in personality, decreased memory, slow thought process, impaired speech, and anhidrosis on one side of the forehead. Focal neurological deficits and abnormal reflexes may be present. Based on the lack of trauma history and the patient's underlying condition, the more likely diagnosis is intracerebral hemorrhage.

Case Ico-delete Highlights A 32-year-old woman with a history of a tremor presents seeking help. She has experienced emotional stress from work over the past 6 months. Upon physical exam, a tremor is observed in her right hand and head. Laboratory findings showed no abnormal findings; however, her family history reveals that her father was also diagnosed with a tremor around the same age. Question What is the most likely explanation of the findings?

orrect answer: Essential tremor Explanation The clinical picture is suggestive of essential tremor in a patient demonstrating symptoms that are commonly found in patients in this condition. A positive family history is commonly found due to responsible genes at 3q13, 2p22-p25, and 6p23. Essential tremors can be brought on by emotional stress. Huntington's disease is associated with uncontrollable impairment of voluntary movements, as well as stumbling and depression; therefore, it is not the correct answer. Idiopathic torsion dystonia is associated with painful muscle contractions, and therefore not the correct answer to this problem. Multiple sclerosis is associated with numbness, blurred vision, and weakness, and is therefore not the correct answer to this problem. Parkinsonism is associated with bradykinesia, rigidity, and a shuffling gait; therefore, Parkinsonism is not the correct answer.

Case A 35-year-old man presents with headaches; he describes them as occurring on 1 side of his head with a sensation of a sharp, stabbing pain just above his left eye. The episodes last for 2 hours, have occured daily for 2 - 3 weeks, and have been presenting in this pattern every 3 - 4 months for the past 2 years. The patient describes his nose as feeling congested during these periods; his left eye also appears red. Aggravating factors appear to be alcohol and stressful situations. He denies any family history of migraines or headaches similar to his. Diagnostic and laboratory studies are pending. Question What is the most likely diagnosis?

Correct answer: Cluster headaches Explanation Cluster headaches are headaches that occur more commonly in middle-aged men. Patients will complain of severe, unilateral, periorbital pain that occurs daily for several weeks; they may be associated with 1 or more of the following symptoms: ipsilateral nasal congestion, rhinorrhea, lacrimation, or redness of the eye. Patients are restless and agitated during the acute phase of this type of headache. Cluster headaches more typically occur at night, and they awaken the patient from sleep. These episodes vary in length from 15 minutes up to 3 hours; at the same time, these occurrences may last for several weeks and then resolve for many weeks at a time. Tension headaches are a common type of primary headache disorder; they are often called stress headaches. Patients will describe pericranial tenderness, described as vice-like or tight; the tenderness is exacerbated by emotional stress, fatigue, noise, or glare. These headaches are usually general and not associated with focal neurological symptoms. Cerebral aneurysms, also called intracranial aneurysms, are most commonly asymptomatic until they rupture. When a rupture occurs, a subarachnoid hemorrhage occurs; the patient will often describe their headache as 'the worst headache I've had in my life'. Intervention must be initiated quickly to ensure survival. Hemicrania continua are considered a separate type of primary headache syndrome. They also present with unilateral head pain and autonomic symptoms, but unlike cluster headaches, patients will have continuous pain without any pain-free time periods. These types of headaches are completely resolved with indomethacin. Classic migraine headaches are described as unilateral in nature and usually pulsatile. They may be accompanied by nausea, vomiting, photophobia, and even phonophobia. An aura, usually visual, will precede the head pain, but migraines can also occur without auras.

Case A 23-year-old woman was diagnosed with myasthenia gravis a few years ago, and she now presents for a routine check-up. She is currently being treated with 15 mg of neostigmine taken orally every 4 hours. In myasthenia, there is autoantibody production against the acetylcholine nicotinic post-synaptic receptors. This results in the reduction of the number of acetylcholine receptors. Question The nicotinic acetylcholine receptors are composed of a, b, g, and d subunits in the molar ratio of what sequence?

Correct answer: 2 : 1 : 1 : 1 Explanation Nerve cells and their target cells abut but do not touch each other. An electrical signal traveling along the nerve cell is transmitted across the gap that is called the synapse. The cell that sends the signal across the synapse is called the presynaptic cell; the cell receiving the signal that traveled across the synapse is called the postsynaptic cell. Receptors on the membrane of the presynaptic cell are called presynaptic receptors; receptors on the postsynaptic cell are called postsynaptic receptors. The electrical signal that traveled along the presynaptic cell is passed across the synapse to the postsynaptic cell as a chemical signal. The chemicals that travel across the synaptic gap are called neurotransmitters. Typically, neurotransmitters are released from vesicles in the presynaptic cells. The vesicles fuse with the presynaptic membrane and release their contents into the cleft. The vesicles are called presynaptic vesicles. In vertebrates, acetylcholine is an excitatory neurotransmitter. The neurotransmitter has a postsynaptic receptor that has been visualized by electron microscopy. This shows that the receptor is composed of 4 different polypeptides subunits labeled a, b, g, and d. These subunits occur in a molar ratio of 2 : 1 : 1 : 1, and they have a total mass of 255,000 daltons. Each of the subunits spans the membrane forming a cylinder with a water-filled channel. In addition, each of the subunits exhibits sequence similarity implying that each arose as a gene duplication and diverged from a single gene.

Question What is the therapeutic window for using recombinant tissue plasminogen activator in acute ischemic stroke?

Correct answer: 3 - 4.5 hours Explanation Tissue plasminogen activator (TPA) catalyzes the conversion of plasminogen to plasmin, which promotes fibrinolysis. Ischemic stroke is caused by sudden occlusion of a cerebral artery by thrombus or embolus, producing a focal neurologic deficit. Timely reperfusion of ischemic brain tissue has been shown to limit irreversible neuronal injury in both humans and animals. The benefit of TPA in acute ischemic stroke was demonstrated in a large, multicentered national study. Patients whose symptoms began within 3 hours of treatment with TPA had significantly less disability at the end of 3 months than those given a placebo. Although treated patients had a higher incidence of secondary brain hemorrhage (6.4% vs. 0.8 %), there was no difference in mortality between the 2 groups. For patients who can be treated with IV rtPA, it must be given within 3 to 4.5 hours from symptom onset in patients with acute ischemic stroke. Prior to treatment, it is essential to exclude cerebral hemorrhage on a head CT scan. The presence of a bleeding diathesis, uncontrolled hypertension, recent prior stroke, and past history of cerebral hemorrhage are important contraindications for using TPA in acute ischemic stroke. TPA given later than 6 hours of stroke onset is associated with unacceptable risk of cerebral hemorrhage; it is not known whether fibrinolytic therapy can be given between 3 and 6 hours of stroke onset.

Question Ico-delete Highlights A 24-year-old man presents with head trauma received in a motor vehicle accident (MVA) 30 minutes ago. Paramedics relate that the patient experienced a loss of consciousness for 1 minute, with a complete recovery. At the subsequent evaluation, the physical exam reveals a patient with a Glasgow Coma Scale (GCS) of 12, right hemiparesis, and a left fixed dilated pupil. What is the most probable finding in the brain CT scan?

Correct answer: A left epidural hematoma Explanation The patient presents with a history of head trauma, a brief initial period of unconsciousness, a lucid interval lasting minutes, subsequent deterioration in neurological status revealed in a Glasgow coma scale of 12, right hemiparesis, and left fixed dilated pupil. This is a classical presentation of left epidural hematoma, which is seen as a biconvex hyper-density (whiteness) on a CT scan. A right epidural hematoma would produce a left hemiparesis and a right fixed dilated pupil. Depending on where and how quickly it develops, an acute subdural hematoma (SDH) may present in a number of ways. In the majority of cases, coma is present at the time of injury. For others, there will be a delay in the decrease in level of consciousness as the hematoma expands. Some patients will remain conscious. The classic presentation of a subarachnoid hemorrhage (SAH) is sudden onset of a severe headache, often termed a "thunderclap headache". Patients may describe this as the "worst headache of my life".

Case A 9-year-old boy presents after an episode that his mother describes as "zoning out." He was eating dinner when he looked up and began to stare at the wall. He would not respond to her verbal questioning. She stated that he then looked at her and began to cry. Question What is the most likely diagnosis?

Correct answer: Absence seizure Explanation The correct answer is absence seizure; they are characterized by an impairment of consciousness, occasionally with mild tonic, clonic, atonic, or autonomic symptoms. The patient is typically unaware of the attacks. They almost always begin in childhood, and they end by age 20. Atypical absence seizures are those characterized by more striking changes in tone; seizures may be more gradual in onset and completion than typical absence seizures. Patients with simple partial seizures do not experience a loss of consciousness. Complex partial seizures typically have impaired consciousness with focal motor symptoms or somatosensory symptoms, and they do not have the 3 phases. Grand mal seizures are characterized by a tonic phase, which includes a sudden loss of consciousness and rigidity that typically lasts less than a minute. A clonic phase follows, which is characterized by jerking motions usually lasting for a few minutes. The clonic phase is followed by a flaccid coma state.

Question A 66-year-old English teacher is in the hospital. Her neurologist gives her a magazine to read, and she is unable to do so. What is this phenomenon called?

Correct answer: Alexia Explanation When a patient is able to hear things and repeat them, it is called echolalia. With echolalia, the patient does not understand what he has heard. This is also referred to as echophrasia. Paraphasia is a type of aphasia. The substitution of a similar sounding word for another word is called paraphasia. With paraphasia, the words can also be jumbled. Alexia is a type of aphasia. An aphasia where there is a problem with reading is called alexia. Alexia is word blindness or text blindness. Alexia is also called optical aphasia or visual aphasia. Apraxia refers to the condition where a patient has difficulty performing motor acts, despite having the muscular capacity and coordination to do so. A patient with apraxia cannot execute the intended movement. A writing disturbance is called agraphia. There are various forms of agraphia. With absolute agraphia, even simple letters cannot be written. This is also referred to as literal agraphia.

Case A 68-year-old man presents for poor concentration. After recently retiring, the patient began having difficulty starting his day. Over the past month, he began drinking occasionally. His short-term memory appears to be failing; however, his long-term memory appears unaffected. He has no history of heart disease, his blood pressure is 135/88 mm Hg, and his pulse is 80 BPM. After an evaluation of his mental status, he is diagnosed with depression due to his retirement and radical changes in his daily schedule. He is prescribed antidepressants and advised to participate in a support group. 2 years later, the same patient returns with continued loss of memory, despite compliance with medications. He has had trouble finding his daughter's house recently, even though she has lived in the same house for many years. His wife says the patient's personality has changed; he has become suspicious of those around him, and he thinks his dog is a tiger. She also says he is unable to perform activities of daily life. The patient has no headaches, nausea, or seizures. His blood alcohol concentration is 0.04%. His thyroid-function tests, thiamine levels, serum vitamin B12 levels, CBC, measurement of blood urea nitrogen, serum electrolyte, blood glucose levels, liver-function tests, serologic test for syphilis, erythrocyte sedimentation rate, and test for HIV antibody are all within normal limits. Question What is the most likely diagnosis?

Correct answer: Alzheimer's disease Explanation Alzheimer's disease is the correct answer; Alzheimer's disease is a chronic, primary, and progressive cognitive impairment type of dementia, and it matches the patient's condition. Alzheimer's disease is the most common cause of dementia. The DSM-IV criteria for the diagnosis of Alzheimer's disease include the development of multiple cognitive deficits manifested by both memory impairment and impairment of 1 (or more) of the following cognitive disturbances: aphasia, apraxia, agnosia, or disturbance in executive functioning. The course of the disease is characterized by gradual onset and continuing cognitive decline. It can cause significant social and/or occupational impairment. It is not associated with other central nervous system conditions (such as Parkinson's disease, Huntington's disease, subdural hematoma, normal-pressure hydrocephalus) or systemic conditions (such as hypothyroidism, vitamin B or folic acid deficiency, niacin deficiency, hypercalcemia, neurosyphilis, HIV infection) that cause progressive memory and cognition disturbance. It is not associated with substance-induced conditions. Multi-infarct dementia usually occurs in patents with hypertension and/or atherosclerotic or rheumatic heart disease. Dementia due to drug and alcohol intoxication, cancer, AIDS, major stroke, or heart, renal, or liver failure is acute and potentially reversible; it is referred to as secondary dementia. Acting out is an immature self-defense mechanism where unacceptable feelings are expressed in actions (e.g., a depressed person steals a car). Since depression in the elderly is often associated with memory loss, it may mimic Alzheimer's disease, which may be called pseudodementia. Unlike Alzheimer's disease, depression can be treated with psychotherapy or pharmacotherapy. This patient has been cooperative with his treatment plan and still his condition gradually became worse; therefore, pseudodementia is not the correct answer. Having intracranial tumors, especially after 2 years of progression, would result in symptoms of intracranial pressure. These symptoms are headache, nausea, seizures,and other neurologic symptoms. The patient doesn't have any of these symptoms. Legal intoxication of alcohol is defined as 0.08% - 0.10% blood alcohol concentration. Coma occurs at a blood alcohol concentration of 0.40% - 0.50%. Long-term alcohol abuse may result in Korsakoff's syndrome, or alcohol amnestic disorder, a form of dementia that is caused in part by thiamine deficiency. This patient drinks alcoholic beverages on and off. His blood alcohol concentration is 0.04%; this is not in the alcoholic intoxication range. His thiamine levels are also within normal limits.

Case A 71-year-old woman presents with memory and behavioral changes. The daughter reports that her mother frequently pauses mid-conversation, repeating what she had said previously. She has misplaced her house keys on numerous occasions. Recently, the patient phoned her daughter 3 times in 1 day, not remembering any recent conversation. The patient has also accused her daughter of trying to steal money and food from her home. There is no history of head trauma or seizures or frank disorientation. Her overall health is good. She has no major medical problems and takes no medications. General physical examination is normal. Neurologic exam reveals the patient to be alert and awake. The patient is oriented to self, and she knows that she is at the doctor's office, but she does not know today's date. She is able to register 3 items, but she is unable to repeat them after 2 minutes; her memory of newsworthy events is also poor. She is able to discuss remote autobiographical events with ease. Her speech is hesitant, with frequent pauses and word-finding difficulty. She is unable to name a list of fruits and cannot identify parts of a watch. She is unable to copy a polygonal figure, and she experiences difficulty in drawing a clock. She is able to follow simple commands, but she cannot follow multiple-step commands. She scores 20 out of 30 on the mini-mental status exam. Her cranial nerves, motor and sensory testing, and gait are normal. Reflexes are 2+ bilaterally, and the plantar responses are flexor. Question What is the most likely diagnosis?

Correct answer: Alzheimer's disease Explanation This patient has Alzheimer's disease (AD). AD is a progressive neurodegenerative disease of cortical neurons, and it is the most common cause of dementia in individuals over 60. The incidence of AD increases exponentially each decade after 60. Pathologically, AD is characterized by predominant loss of cortical neurons in the temporal, parietal, and frontal lobes. Dropout of cholinergic neurons in the deep temporal nucleus basalis of Meynert appears to have an important role in the development of some of the neuropsychiatric sequelae of AD (see below). Clinical manifestations of AD include loss of memory and other higher intellectual functions (orientation, attention, concentration, abstract reasoning, judgment, motivation); disturbances of language (aphasia); loss of ability to performed learned activities (apraxia); and visual-spatial difficulty. Neuropsychiatric symptoms are also prevalent in AD: agitation, anxiety, depression, paranoia, insomnia, and auditory or visual hallucinations. Other causes of dementia include multi-infarct (vascular) dementia, normal pressure hydrocephalus (NPH), pseudodementia of depression, and chronic subdural hematoma (SDH). These disorders can usually be excluded with detailed history and examination, neuroimaging (CT, MRI), and laboratory tests. Vascular dementia is characterized by stepwise cognitive decline in association with stroke and focal neurologic deficits. Many patients with vascular dementia have a history of hypertension, diabetes mellitus, or coronary artery disease. NPH is a rare cause of dementia in the elderly. Patients with NPH have additional symptoms of gait instability and incontinence. Chronic SDH may be associated with only minor head trauma, which may only be disclosed by obtaining a detailed patient history. In addition to slowed thinking and memory loss, headache may be a presenting feature. Psychiatric disorders may mimic dementia in some older people. Inattention, poor concentration, and other signs of depressive illness characterize pseudodementia of depression.

Case Ico-delete Highlights A 70-year-old man presents because he is increasingly forgetful. After retiring at 65 from his job as a schoolteacher, he had enjoyed cooking for his wife and family, but he cannot prepare a meal anymore; he forgets ingredients or steps during the procedure. When addressed about the problem, he sometimes gets angry. In one instance, he even threatened to hurt his wife. He also frequently forgets conversations and gets upset with his spouse for not telling him what is going on. Otherwise, the patient is quite active socially; he attends senior citizen meetings and goes on bus trips. His wife mentions that his appetite is decreasing and he has lost several pounds lately. He always liked to write letters and poems, but he now has problems finding the right words. The patient denies alcohol, nicotine, and substance abuse. His medical history consists of mild hypertension, which is treated with a diuretic, as well as benign prostate hyperplasia, which will be treated surgically. In addition, he had an appendectomy at age 13 and a tonsillectomy at age 18. The family history is as follows: his father had Parkinson's disease and died at 69; his mother died at 78 from cervical cancer. General examination is unremarkable. Results of the lab test show normal blood count, thyroid hormones, vitamin B12 levels, erythrocyte sedimentation rate, and CRP. Fluorescent treponema pallidum antibody absorption test is negative. Mental exam: Mini-Mental State Exam (MMSE) is administered, and the patient's score is 24. Reading and writing abilities are impaired. Memory test with 10 objects to remember shows 3, 5, and 6 in 3 trials. After 5 minutes, only 1 object can be recalled. An MRI shows no significant cerebrovascular disease or masses, but it does show moderate cortical volume loss in the medial temporal structures. Electroencephalography (EEG) shows generalized slowing. Question What is the most likely diagnosis?

Correct answer: Alzheimer's disease Explanation This patient suffers from mild cognitive impairment, which is most likely caused by the beginning of Alzheimer's disease. Criteria for the diagnosis of Alzheimer's disease include onset between ages 40 and 90, deficits in at least two cognitive areas, undisturbed consciousness, progression of deficits for more than 6 months, and dementia. There are about 10% hereditary rates. A. Alzheimer described the disease first in 1907 as progressive brain atrophy with senile dementia. A definite diagnosis can only be made at autopsy. Patients usually die of infections (pneumonia) within 10 years of diagnosis. Treatment is only supportive. Anticholinesterase inhibitors slow dementia; antipsychotics and antidepressants are given for psychosis. J. Parkinson first described Parkinson's disease in 1817 as "shaking palsy." It is a hypokinetic syndrome and characterized by 3 cardinal symptoms: tremor, rigor (cogwheel rigidity), and akinesia (or bradykinesia). The mask-like face expression and flat affect make it sometimes look like depression. It is a pathogenetic dopamine deficiency that is caused by neuron destruction in the substantia nigra. It can lead to dementia. In 5-10% of cases, autosomal dominant heredity can be seen. Patients can also suffer from dysphagia, hypersalivation, bradyphrenia, urinary retention, constipation, and seborrhea. They usually see a doctor when they have problems with basic tasks like buttoning a shirt. Treatment includes dopamine, anticholinergics, and amantadine. Surgical pallidotomy can be necessary for refractory cases. New treatment experiments are the implantation of embryonic dopamine-producing cells. Progressive supranuclear palsy is a late stage of syphilis, the onset of which is 8-15 years after primary infection; it begins with a decrease in vitality and ends with dementia. Patients have problems with articulation and can develop psychosis and epilepsy. It can be ruled out in this case because the syphilis serology is negative. Vascular dementia, or multi-infarct dementia, shows vascular lesions on brain imaging and focal neurologic signs. It usually starts 3 months after a stroke, and it is characterized by abrupt cognitive deterioration and fluctuating stepwise progression of the dementia. Pick's disease clinically resembles Alzheimer's disease, but it affects more women at an earlier age (onset around age 50). First described in 1892 by A. Pick, it pictures a frontotemporal brain atrophy, which is clinically characterized by personality changes and ultimately dementia. Symptoms include loss of ethical inhibition and increased appetite. Intelligence and memory are undisturbed in the beginning; mask-like facies and rigor are possible.

Case A 56-year-old man presents with weakness of his grip. On exam, the problem is bilateral. During the next few office visits, you note the development of hyperactivity of his deep tendon reflexes (DTRs), extensor plantar reflexes, and dysarthria. The patient's sensory system remains normal, and he denies any urinary symptoms. Question What is the most likely diagnosis?

Correct answer: Amyotrophic lateral sclerosis Explanation The clinical picture is suggestive of amyotrophic lateral sclerosis (ALS). ALS is a progressive bilateral muscle disease that causes fasciculations and hyperreflexia, plantar reflexes, and dysarthria. Sensation and bladder function remain normal. The common clinical presentation seen in multiple sclerosis is weakness, numbness, tingling, or unsteadiness in a limb; spastic paraparesis; retrobulbar neuritis; diplopia; disequilibrium; or a sphincter disturbance such as urinary urgency or hesitancy, which is not present in this patient. Alzheimer's patients present with memory impairment and at least 1 of the following: language impairment, apraxia, agnosia, and impaired executive function. Rigidity and bradykinesia are primary signs; tremors are rare. These symptoms are not present in this patient. Huntington's disease is characterized by chorea and dementia. Clinical onset is usually between 30 and 50 years of age. Patients present with writhing choreiform movements of the body, which are not present in this patient. In myasthenia gravis, small motor units (such as ocular muscles) are most often affected, producing nystagmus. Major clinical features of this disease are weakness that improves after a period of rest and fluctuating fatigue, and these symptoms improve after administering acetylcholinesterase inhibitors. These symptoms are not present in this patient.

Case Ico-delete Highlights A 32-year-old woman presents due to "I'm feeling odd." Further questioning reveals the patient experiencing a significant amount of involuntary movements and a severe lack of coordination; she recently had a significant decrease in cognitive abilities. Because she is an established patient, you recall that the patient got married about a year ago; past family history is unknown due to the fact that the patient was adopted as a child. Physical examination reveals rapid, involuntary movements of fingers bilaterally, impaired ability to concentrate, minor slurred and disorganized speech, as well as difficulty responding with the appropriate words or phrases when prompted. Question What would be the recommended testing at this time to confirm the patient's diagnosis?

Correct answer: Genetic testing Explanation The correct response is genetic testing. The patient in the above scenario is displaying signs of Huntington's disease. This fatal progression of chorea, dementia, and behavioral changes is an autosomal dominant disorder that occurs in all ethnic groups with a prevalence rate of 5 per 100,000. More commonly than not patients will have a strong family history of this disease state; the unknown family history in our patient makes this information, although missing, extremely valuable. Onset of symptoms typically occurs between the ages of 30 - 50 years old and usually leads to death within 15 - 20 years after symptoms begin occurring. Early symptoms and signs include abnormal movements or intellectual changes, both of which are evident in the patient above. Severe progression of rigidity, chorea, akinesia, and severe psychiatric changes will accompany the progression of the disease. Diagnostic testing at this time to confirm a diagnosis of Huntington's disease should be via genetic testing with ultimate guidance from a genetic counselor. The gene responsible for causing Huntington's disease has been identified exclusively on chromosome 4. CT scanning or even MRI imaging will not confirm the diagnosis, but rather support it; cerebral atrophy, as well as caudate nucleus atrophy, will be found as the disease progresses. Acid-fast bacilli smear is the inappropriate choice for this scenario; this diagnostic testing may be utilized in a patient to support the diagnosis of tuberculosis. Rapid plasma reagin testing is also the incorrect choice; this diagnostic study along with the serologic test for syphilis (STS), venereal disease research laboratory (VDRL), and the fluorescent treponemal antibody (FTA) assist health care providers in diagnosis as well as successful treatment of syphilis.

Case Ico-delete Highlights A 35-year-old man presents with unusual sensations in his legs. He states that his symptoms began in college and were mild, but they have been steadily worsening over the last 4 years. His symptoms begin whenever he is lying down attempting to go to sleep. He describes the sensation as an uncomfortable crawly sensation deep within his legs. If he tries to ignore them, the feeling worsens. He has brief relief with movement, but the feelings return as soon as he is still again. He was given a trial of a benzodiazepine to take at bedtime, but it did not really help. In addition, he complains of being sleepy during the day as a result of the medication. He also tried leg massages and exercise without much relief. His past medical history is significant for depression and anxiety. The patient denies snoring, vivid dreams, or cataplexy. He states that he has a depressed mood and reduced energy and concentration, but he denies suicidal ideation. His physical exam is normal. Complete blood cell count and chemistry panels are normal. Question What is true regarding the patient's condition?

Correct answer: An abnormal electrical impulse conduction study is frequently seen. Explanation An abnormal electrical impulse conduction study is frequently seen in restless legs syndrome (RLS), which is a neurological disorder characterized by unpleasant sensations in the legs and an uncontrollable urge to move when at rest in an effort to relieve these feelings. RLS sensations are often described as burning, creeping, tugging, or like insects crawling inside the legs. Often called paresthesias or dysesthesias, the sensations range in severity from uncomfortable to painful. Needle electromyography and nerve conduction studies should be considered if polyneuropathy is suspected on clinical grounds, even if results of the neurologic examination are apparently normal. The most distinctive aspect of the condition is that lying down and trying to relax activates the symptoms. Many patients with RLS have difficulty falling asleep and staying asleep. The disorder is diagnosed clinically by evaluating the patient's history and symptoms. Women may be slightly more affected than men. Calcium deficiency causes mainly muscle spasm and twitching and is usually associated with other symptoms of arm and leg numbness, numbness around the mouth, nervousness, heart palpitations, and an inability to sleep, none of which are found in this case. In most cases, the cause of RLS is not known; in some cases, it may be caused by iron deficiency, folic acid deficiency, vitamin B12 deficiency, or arthritis. Sometimes, RLS can be associated with the use of antidepressant medications, necessitating the use of an alternative treatment.

Question A patient with early onset of hypertension, hematuria, and palpable kidneys was diagnosed to have adult polycystic kidney disease, and he is being evaluated for associated berry aneurysms. What investigation is considered the "gold standard" for the detection of aneurysms of the cerebral blood vessels?

Correct answer: Angiogram Explanation Adult polycystic kidney disease is associated with cysts of the liver in 30 to 40% of patients. Other associations include berry aneurysms of the cerebral blood vessels, colonic diverticula, abdominal wall hernias, and mitral and aortic regurgitation. APKD is an autosomally dominant disorder associated with the PKD 1 gene on chromosome 16. Angiography involves taking several X-rays after the injection of a radio opaque dye in the blood vessel thus making the outline of the vessel visible. From this outline, it would be discernible as to whether there is or is not an aneurysm present. Computer-assisted tomography (CAT), like conventional radiography, produces X-ray images of various structures. The CAT scan, however, is able to reveal subtle differences that can distinguish gray and white matter, the ventricles, the thalamus, the basal ganglia, and others. It can produce an image of a single plane or section of the brain. (The term tomography comes from the Greek word meaning, "cut".) Positron emission tomography (PET) provides live, real time images of the functioning of the brain. PET scanning measures the differential rates of glucose metabolism in the active brain. Both CAT and PET images are obtained by a detector that rotates around the patient's head. However, CAT images reflect the absorption of radiation, whereas PET images are produced by the emission of radiation. Electroencephalography measures the electrical potentials of the surface layers of the cerebral cortex by means of electrodes placed on the skull.

Case A 37-year-old woman presents with a history of right-sided facial weakness and periauricular discomfort since she awoke this morning. She is afebrile. Question What is the most likely diagnosis?

Correct answer: Bell's palsy Explanation The correct answer is Bell's palsy, which is a condition with typically sudden onset that affects the facial nerve, causing unilateral facial weakness. Trigeminal neuralgia presents with sharp pain on one side of the mouth that radiates to the ipsilateral ear, eye, or nostril. Multiple sclerosis is a demyelinating disorder; it causes a multitude of symptoms that typically include diplopia or blurred vision early on, followed by an insidious onset of progressive weakness, numbness, and/or tingling in the extremities. Myasthenia gravis commonly presents with ptosis and diplopia, as well as difficulty swallowing, fatigue, and muscle weakness. Primary lateral sclerosis is an upper motor neuron disease that causes limb weakness, stiffness, and fasciculations.

Case A 33-year-old African American woman presents with a new onset of left facial droop. The droop was present when she awoke this morning, and it has worsened since then. She reports that she had been feeling fine. She denies headaches, visual changes, and weakness or tingling in the extremities. Exam reveals an anxious, overweight woman. She is afebrile. Cranial nerve exam reveals weakness of the entire left side of the face, including the forehead. The left eye does not close entirely, and there is a widened palpebral fissure. Sensation is intact. All cranial nerves, other than VII, appear intact. Her tympanic membranes are clear, and there are no skin lesions around the face or head. The rest of her exam is unremarkable. Labs reveal normal CBC, serum electrolytes, thyroid function tests, and liver transaminases. A chest X-ray is clear. Question What is the most likely cause of this patient's facial droop?

Correct answer: Bell's palsy Explanation This patient has a classic presentation of Bell's palsy. Bell's palsy is a sudden, unilateral facial weakness and affects about 1 out of every 60-70 people in their lifetime. The etiology is unknown at this time, but herpes simplex virus type 1 has been associated with some cases. The presumed pathogenesis of the palsy is that inflammation of the facial nerve causes swelling, then secondary compression, and ischemia occurs where the nerve courses through the temporal bone. Pain behind the ear may precede the paralysis by 1-2 days. Hyperacusis and loss of taste may occur if the lesion is proximal enough; lacrimation may be affected as well. Sensation is not affected, although patients may experience numbness or heaviness in the face. In some cases, there is a mild lymphocytosis of the CSF. MRI may reveal swelling and uniform enhancement of the geniculate ganglion and facial nerve. Also MRI may reveal a swollen facial nerve entrapped within the temporal bone in some cases. Prognosis depends on the degree of paralysis. Approximately 80% of patients will recover within a few weeks to a month. Of those with an incomplete paralysis in the first week, nearly all will experience complete recovery within several months. Of those with a more complete paralysis—as in this case—nerve conduction studies and electromyography can indicate prognosis. The likelihood of completely recovery is 90% if the nerve branches in the face retain normal excitability to supramaximal electrical stimulation, but the likelihood is only about 20% if electrical excitability is absent. Treatment includes symptomatic care, such as taping the upper eyelid closed during sleep and using lubricating agents in the eye to prevent corneal desiccation. Some studies have shown improvement in patients treated with glucocorticoids; other studies showed improvement with prednisone and acyclovir together. Sarcoidosis can cause a form of bilateral facial paralysis (facial diplegia) called uveoparotid fever (Heerfordt's syndrome). Given this patient's African American background, sarcoidosis should be considered as a possible etiology of her symptoms. The unilateral nature of the paralysis, combined with a negative chest X-ray, however, makes sarcoidosis unlikely. A serum angiotensin-converting enzyme (ACE) could be drawn if sarcoidosis is a concern. Ramsay-Hunt syndrome is a facial palsy associated with a vesicular eruption in the pharynx, external auditory canal, and other surrounding areas of skin. This is thought to be secondary to herpes zoster infection of the geniculate ganglion; often the eight cranial nerve is also affected. This patient did not have any skin lesions around the ear, making this diagnosis very unlikely. Even so, this demonstrates the importance of a thorough examination in facial palsy patients, including careful examination inside the ears. Lyme disease is a frequent cause of facial paralysis in areas where Lyme infection is endemic, but the majority of patients who have facial palsy caused by Lyme disease note an antecedent rash adjacent to the site of a tick bite. Blood tests for Lyme disease can help identify that as an etiology, but this patient denied any risk factors for tick bites such as camping/travel, making this diagnosis less likely. The family history of CVA and the fact that she is currently taking oral contraceptives do confer some increased risk for thrombotic events in this patient. With a stroke, however, the forehead is usually spared from facial palsy since the upper facial muscles (the frontalis and orbicularis oculi muscles) are innervated by corticobulbar pathways from both motor cortices. The lower facial muscles are innervated only by the contra-lateral hemisphere. This patient's facial palsy is complete and includes the forehead because the facial nerve itself is affected in Bell's palsy.

Case A 60-year old woman has a history of stress related migraine headaches. Many conventional treatments have failed. Question What treatment would you now try on this client?

Correct answer: Biofeedback Explanation Biofeedback is a form of behavior therapy involving relaxation and the power of suggestion. It is used to treat psychosomatic-related ailments such as asthma, depression, migraine headaches, hypertension, stress, chronic pain, and digestive problems. Token economy involves the use of rewards to condition behavior. These rewards may include money, privileges, food passes, and gifts. This therapy is utilized a lot in inpatient or residential settings, such as prisons, mental hospitals, nursing homes, and substance abuse treatment centers. Aversion therapy is a type of conditioning in which a negative stimulus is connected with an undesired behavior. For example, to assist an individual to stop smoking, one might add an unpleasant taste or smell when a smoker lights up a cigarette. This type of therapy is used frequently in smoking, weight loss, or other undesired behaviors. Systematic desensitization is a behavioral technique in which a client with some type of avoidant behavior linked to a specific stimulus is asked to construct a hierarchy of anxiety-provoking images until the avoidant behavior diminishes. Examples of fears the technique may work on include heights, snakes, airplane travel, elevators, escalators, and many others. Flooding is a technique in which the client is exposed to the highest possible anxiety-provoking experience related to whatever they fear the most. For example, instead of proceeding slowly as in systematic desensitization, the client is immediately exposed to a tall building, snake, elevator, escalator, or whatever they fear the most. This is a highly controversial therapy, and it should only be used with a great degree of caution and knowledge.

Case A 57-year-old white man presents to the emergency department reporting a syncopal spell 1 hour ago. He was in his normal state of health until approximately 7 days ago, when he developed acute diarrhea that lasted 4 days and then resolved; however, for the past 3 days, he has had progressively worsening nausea and vomiting. Past medical history is significant for congestive heart failure secondary to non-ischemic cardiomyopathy, atrial fibrillation, hypertension, chronic renal insufficiency (with baseline creatinine 2.0), and benign prostatic hypertrophy. Previous surgeries include laser surgery for a detached retina and transurethral retrograde prostatectomy. He has no known drug allergies but indicates an intolerance to diltiazem due to hypotension. Current medications include furosemide, digoxin, enalapril, carvedilol, and tamsulosin. Review of systems is as follows: General—Admits to decreased appetite for the past 3 days. Admits to fatigue and malaise. HEENT—Admits to visual disturbances for the past 3 days described as yellow and green halos around lights. Cardiovascular—Admits to chronic dyspnea on exertion at 3 blocks. A complete review of systems was otherwise negative. On physical examination, temp 98.8°F, pulse 40 and slightly irregular, respirations 16, blood pressure 108/60. The patient appears alert with nausea and some vomiting during interview and exam. Bradycardia is present with slightly irregular heart rhythm. Normal S1 and S2 without obvious rub, murmur, or gallop. Lungs fields are clear without rales, rhonchi, or wheezes. The remainder of complete physical examination is unremarkable. Lab: Notable for K+ of 5.8 mEq/L, BUN 40 mg/dL, creatinine 4.2 mg/dL, digoxin level 4.8 ng/mL. EKG demonstrates complete AV dissociation present while CXR shows no acute disease and abdominal X-ray displays normal findings. Question What is the most likely cause of his syncope?

Correct answer: Bradycardia with complete AV block Explanation Bradycardia with complete AV block is the correct answer. The EKG shows complete AV dissociation, i.e., third-degree heart block. His pulse rate is 40 and irregular. Bradycardia and/or complete heart block with a heart rate in the 40s or below is the likely cause of this patient's syncope—in this case, secondary to digoxin toxicity. This patient's initial symptom was diarrhea, which then led to dehydration, a common precipitant of acute or chronic renal failure. In the setting of renal failure, digoxin (which is principally metabolized by the kidneys) can accumulate in the bloodstream resulting in digoxin toxicity. Digoxin toxicity has been associated with a wide variety of cardiac arrhythmias, including complete heart block. Symptoms of digoxin toxicity typically include anorexia, nausea, vomiting, and perception of green and yellow halos around lights. Atrial fibrillation with rapid ventricular response is incorrect. Extremely rapid heart rates can be associated with syncope. Although this patient has a history of atrial fibrillation, he is on digoxin and carvedilol, both of which are used to suppress tachycardia and keep heart rate under control. The patient had not reported any symptoms of palpitations or tachycardia. Hypoglycemia is incorrect. Severe hypoglycemia can lead to loss of consciousness or coma and represents a medical emergency, but it does not represent true syncope. Hypoglycemia is a known complication of insulin or hypoglycemic medication in diabetic patients; however, there is no clinical history or lab work in the case that points to hypoglycemia. Hypoxemia is incorrect. Hypoxemia may cause a non-syncopal loss of consciousness; however, this patient has no known pulmonary disease and has not reported pleuritic chest pain or acute dyspnea. His heart failure symptoms are stable. Physical exam showed clear lung fields and chest X-ray showed no acute disease. Neurocardiogenic syncope is incorrect. In a setting of complete heart block, hyperkalemia, and no prior history of syncope, a diagnosis of neurocardiogenic syncope is unlikely.

Case A 64-year-old man presents with a 1-week history of headaches. His wife states that for several months her husband has been increasingly moody, isolating himself from the family and friends and spending most of the time locked in his room. He becomes irritated easily and has had several non-purposive violent outbursts. She did not notice any clouding of consciousness. She also heard him talking to himself in his room. The patient denies any head trauma, and he does not take any medications. On examination, the patient looks tired, distressed, hostile, non-cooperative, and insists that nothing is wrong. From time to time, he tilts his head as if he is listening to something, but he denies hearing voices. Cambridge Neurological Inventory (a standardized neurological assessment of psychiatric patients) reveals the presence of "soft neurological signs." Mental status testing reveals disturbances in memory and higher cognitive functions, including calculations and proverb interpretation. His physical examination is normal, as are his laboratory tests (i.e., CBC, ESR, biochemical profile, TSH, B12 and folate, VDRL, liver and kidney function tests, and urine toxicology screen). Question What is the next diagnostic step?

Correct answer: Brain imaging Explanation It is considered good clinical practice to perform a CT or other neuroimaging test in all patients presenting with psychosis. Red flags in this patient include the history of late onset of psychosis, headaches, and soft neurological signs. Once possible causes of behavioral changes are excluded, the next step should be brain imaging to rule out subdural hematoma, vasculitis, cerebral abscesses, or tumor. Dexamethasone suppression and adrenocorticotropic hormone (ACTH) stimulation tests test are for hypercortisolism and hypocortisolism. Antinuclear antibody (ANA) is used in the diagnosis of systemic lupus erythematosus (SLE). Although SLE may present with psychiatric symptoms, it is much more common in women than in men, appears most often in people between the ages of 10 and 50, and African-Americans and Asians are affected more often than people from other races. Abnormally high levels of porphyrins in a 24-hour urine collection are used to diagnose porphyria. Porphyria can present with psychiatric symptoms; patients may even have a family history of psychosis. The psychiatric symptoms are usually associated with electrolyte changes, peripheral neuropathy, and episodic, severe abdominal pain. The use of antipsychotic medications, also known as neuroleptics or major tranquilizers, is indicated in specific psychiatric disorders (e.g., schizophrenia or mania) that produce psychosis, dangerous behaviors that produce an imminent threat or harm to self or others, or in the cases of disruptive behaviors that fail to respond to behavioral interventions; however, a trial of antipsychotics will not help to make a diagnosis.

Case A 19-year-old woman presents following a motor vehicle accident 20 minutes prior. She states that it was just a fender-bender, but she feels she might have whiplash. Her neck is stiff and sore, and she has developed numbness and tingling in her left arm down to her thumb. On physical examination, her bicep strength is +3/5 on the left and +5/5 on the right. Her bicep tendon reflex is 2+ on the left and 2+ on the right. In addition, her brachioradialis tendon reflex is 1+ on the left and 2+ on the right. The remainder of the physical exam is normal. Question Based on the above presentation, what cervical nerve root is most likely affected?

Correct answer: C6 Explanation The most likely nerve affected in this case is the C6 nerve root. Impingement of this nerve can cause numbness and tingling down the arm into the thumb, with weakness in the bicep muscle and diminished brachioradialis tendon reflex in the affected extremity. Impingement of the C4 nerve root may cause neck and upper shoulder numbness and pain. Impingement of C5 nerve root may cause deltoid and shoulder numbness and pain. Bicep tendon reflex may be diminished. Impingement of C7 also causes numbness and pain down the affected arm, but reflex may be diminished into the middle finger and the triceps. Impingement of C8 may cause numbness and tingling that occurs primarily on the lateral surface of the arm and into the lateral hand into the fourth and fifth digits. It may also cause dysfunction of the hand, as it innervates the small hand muscles.

Case A 44-year-old Caucasian woman with no significant personal or family history is being evaluated for progressive uncontrollable flailing of her extremities over the past year. Over the past 2 months however, these movements have become slower and more rigid. These motions interfere with her daily activities. Her family expresses concerns regarding behavioral changes, which have ranged from irritability, untidiness, and loss of interest. Additionally, there is slowing of her cognition, impairment of intellectual function, and memory disturbances. Upon physical exam, she is found to have saccadic eye movements and facial tics. There is generalized fidgeting, with sometimes violent movements of the hands, and an unstable dancelike gait. Her extremity muscles are contracted and clonus is evident. She appears to have a depressed affect, and she performs poorly upon cognitive, attention, and judgment testing. Question What is the most sensitive and specific diagnostic test for this condition?

Correct answer: CAG gene expansion Explanation The correct response is CAG expansion. This patient's most likely diagnosis is Huntington's disease (HD). In the absence of a family history of HD, patients with a suggestive clinical presentation should undergo genetic testing to exclude or confirm HD. DNA repeat expansion forms the basis of a diagnostic blood test for the HD gene. Direct gene testing via polymerase chain reaction is commercially available, and it can identify the HD gene and carrier states. This test is a sensitive and highly specific indicator of the inheritance of HD. If the genetic test is negative for HD, then testing for systemic lupus erythematosus (SLE), antiphospholipid antibody syndrome, thyroid disease, neuroacanthocytosis, DRPLA, Wilson disease, and other less common causes of chorea may be reasonable, depending on the individual case. Anti-double stranded DNA antibody testing is a specific laboratory test used in the evaluation of lupus. No single imaging technique is necessary or sufficient for diagnosis of Huntington disease. Measurement of the bicaudate diameter (i.e., the distance between the heads of the 2 caudate nuclei) by CT scan or MRI is a reliable marker of HD. Radiographically, caudate atrophy leads to the typical dilation of the frontal horns of the lateral ventricles; however, the sensitivity of a CT scan is insufficient to justify its role in the investigation of patients with suspected HD, unless genetic test findings and other diagnoses need to be excluded. Lumbar punctures are not used in the work-up and evaluation of patients with HD; this procedure is useful in the diagnosis of Guillain-Barré syndrome and multiple sclerosis. A muscle biopsy is a valuable diagnostic test used in the identification of illnesses such as muscular dystrophy, trichinosis, toxoplasmosis, myasthenia gravis, polymyositis, dermatomyositis, ALS, and Friedreich ataxia.

Case A 4-month-old febrile infant presents with loss of appetite, irritability, seizures, focal sensory and motor deficits, and an acute petechial rash. On physical examination, a bulging fontanelle is noted; rectal temperature is 102.8°F. Question What test would be most helpful in evaluating this patient?

Correct answer: CSF analysis Explanation The clinical picture is suggestive of meningitis, which is diagnosed after obtaining and evaluating the cerebrospinal fluid (CSF). Although a bulging fontanelle is an indication of increased intracranial pressure (ICP), it is not an automatic contraindication for a lumbar puncture (LP) unless the patient displays additional signs of increased ICP. LP will be the most important study and diagnostic tool for this child. It is likely that a CT or MRI would be done to look at the level of swelling in the brain and possibly determine the level of ICP. Normally, meningitis causes fever, lethargy, and mental status changes; however, these symptoms are often hard to detect in young children. In infants, signs and symptoms may include appearing to be slow or inactive (lack of alertness), irritability, vomiting, or feeding poorly. When CSF infections are suspected, blood should be obtained for a CBC, general chemistry panel, and culture; however, obtaining CSF analysis is the most important, as these other tests will likely yield nonspecific abnormalities. Because the clinical presentation of bacterial and viral meningitis may be indistinguishable, laboratory studies of the CSF are critical in differentiating these entities. A chemistry panel, UA, or serum glucose are not most helpful for the diagnosis of meningitis.

Case A 1-year-old boy presents with a history of vomiting, lethargy, and respiratory distress of sudden onset; there is no prior illness or injury. The parents state that the boy is "always irritable" and keeps them up all night. The parents have missed several well-baby visits; they both work and are unable to keep the appointments. Physical examination reveals red bruises on the boy's arms that appear to be in the shape of fingers, as well as a full fontanel. Ophthalmologic exam demonstrates the presence of retinal hemorrhage. Question What is the most appropriate next step in the management of this patient?

Correct answer: CT scan of the brain and a skeletal survey Explanation This child is presenting with signs of shaken baby syndrome (SBS), a form of child abuse. If a child presents with an injury that is not consistent with the story told by the caregivers, or if there are inconsistencies in that story, practitioners should have a heightened suspicion of child abuse. Other warning signals include a history of frequent accidents with injuries, including bruises and broken bones. CT scan of the brain and skull. CT scan of the brain and a skeletal survey will help identify new and old injuries. SBS has a particularly high mortality and occurs when an infant (under age 1) is violently shaken or thrown against a soft surface (e.g., a bed); a subdural hematoma may develop and lead to respiratory failure. The full fontanel and retinal hemorrhage indicate the need for immediate radiographic imaging of the brain and skull. Undiagnosed fractures are common among abused children, so a radiographic skeletal survey is also appropriate as soon as the infant's medical condition permits. Reassuring the parents is inappropriate; sending a child home with parents in the setting of suspected abuse is negligent. Although the patient has presented with a history of vomiting, IV fluid administration is not the most important next intervention. Administration of an antipyretic is not indicated because there is no sign of an infection. The presence of a bleeding diathesis is unlikely and does not explain the other presenting symptoms. Referral to an ophthalmologist may be appropriate at a later date because the infant may have permanent visual deficits. It is not a part of the urgent care of this child. In addition to the work-up, social services should be notified in any case of suspected child abuse.

Case A 15-month-old toddler presents with sudden onset of generalized tonic and clonic convulsions for the last 30 minutes. There is no history of trauma, fever, vomiting, or irritability prior to the onset of convulsions. This is the first episode of seizure, and there is no history of convulsions in the family. Birth history, neonatal period, and developmental milestones are normal. Question After initial stabilization by securing the airway, oxygenation, and controlling the acute convulsion, what is the investigation of choice for the above child?

Correct answer: CT scan of the head Explanation CT scan of the head is the best diagnostic imaging study in a previously asymptomatic child who presents with status epilepticus, particularly if there is suspicion of intracranial hemorrhage, a space-occupying lesion, or midline shift. It is a non-invasive and rapid procedure and is widely available. It has a short imaging time and requires less sedation in young children. Though CT imaging is largely replaced by MRI, it still remains the most sensitive modality to detect intracranial hemorrhage and calcifications during the first 24 hours. Other indications are head injury, hypoxic ischemic encephalopathy, inflammatory disorders like neurocysticercosis, CNS tuberculosis, basal exudates, pyogenic abscess, and malformations like hydrocephalus and porencephaly. Myelination, posterior fossa, and brain stem structures are not well visualized by CT scan. As the child presented with his 1st episode of convulsion with status epilepticus without any prior symptoms, a CT scan of the head would be the first choice of investigation. Cranial ultrasonography (USG) is mainly used in the diagnosis and follow-up of newborns with intraventricular hemorrhage and for assessment of ventricular size. It can be performed through the open anterior fontanel, and it can also be performed at the bedside of the patient. It can visualize the ventricles, periventricular tissue, and a part of cerebral cortex. As the above child is 15 months old and the anterior fontanel is either closed or is very small at this age, it cannot be performed in this patient. Electroencephalogram (EEG) is commonly used to confirm a clinically doubtful convulsion, to characterize the type of convulsion, to locate an epileptic focus, or to distinguish a simple febrile convulsion from epilepsy. It can help to distinguish between a seizure and a non-seizure state, such as a fainting spell, hypoxic episode, or a breath-holding spell. It is especially helpful in the diagnosis of absence attacks, herpes encephalitis, and myoclonic seizures. It is not the investigation of first choice in the above patient. Lumbar puncture (LP) and examination of cerebrospinal fluid (CSF) is usually indicated for the diagnosis of conditions like meningitis, encephalitis, subarachnoid hemorrhage, demyelinating illnesses, and slow virus infections of the CNS. As the above child was asymptomatic before the episode of convulsion, lumbar puncture is not the investigation of first choice in this child. MRI scan of the head can demonstrate myelination abnormalities, edema, and infarcts more clearly as compared to CT. MRI is the study of choice for evaluating the spine, spinal cord, and spinal canal in children. It is more sensitive than CT scan for most brain tumors involving the brainstem, posterior fossa, and spinal column. It is not the investigation of first choice for acute intracranial hemorrhage during the first 24 hours. Also, it is not widely available for use during emergency.

Case A 73-year-old man presents after a 15-minute episode of right eye vision loss, which he described as being "like a shade being pulled down". Question What diagnostic test is most likely to be abnormal?

Correct answer: Carotid ultrasound Explanation The case described above is a classic example of amaurosis fugax, which is caused by transient occlusion of the ophthalmic artery, a branch of the internal carotid artery. This occurs most often in the setting of internal carotid artery stenosis. Like other transient ischemic attacks, it is often a harbinger of an impending stroke.

Case Ico-delete Highlights A patient presents to the office. His gait is wide-based, staggering, and unsteady. He appears to have difficulty with turns and hangs on to his wife as he crosses your waiting room. Question What type of gait does he have?

Correct answer: Cerebellar ataxia Explanation The gait characterized by cerebellar ataxia is associated with a disease of the cerebellum or its affiliated tracts. The gait is staggering, unsteady, and wide-based. The patient has difficulty with turns, and cannot stand steady when feet are together and eyes closed (Romberg's sign). Scissors gait is apparent when each leg is advanced slowly and the thighs tend to cross each other. The gait is stiff and the steps are short. This is associated with bilateral spastic paresis of the legs. Steppage gait is associated with a foot drop, often secondary to lower neuron disease. The patient drags their foot, or lifts the knee high, and slaps the foot against the ground. This gait may be unilateral or bilateral. Sensory ataxia gait is unsteady and wide-based. The feet are thrown forward, out, then down. There is a distinct two-slap sound, secondary to the heel striking first then the forefoot. This is associated with polyneuropathy or posterior column damage. A myopathic gait is where there is weakness on one side. This displays a drop in the pelvic area on the contralateral side of the pelvis while walking (Trendelenburg sign). When there is a bilateral weakness, it will display as a dropping of the pelvic girdle on both sides of the body while walking that results in a waddle. This is seen in patients who are afflicted with myopathies, such as muscular dystrophy.

Case A 4-year-old boy presents with erratic movements of his left arm and leg. These movements have been present since birth and occur on the left side. They have not been getting worse. The boy's mother admits that he has been using his right hand for almost all activities since the age of 1. Further questioning reveals that the child exhibited delayed milestones, such as rolling over at 6 months, sitting up at 1 year, and walking at 2 years. He does not speak, and he does not respond to his mother's verbal commands. His physical exam is noteworthy for spasticity, hyperreflexia, ataxia, involuntary movements, and weakness of his left arm and leg. His left arm and left leg are measured to be shorter and are atrophied in comparison to the right. Additionally, the child does not respond to verbal stimuli. Question What is the most likely diagnosis?

Correct answer: Cerebral palsy Explanation The most likely diagnosis in this patient is cerebral palsy. Cerebral palsy is a chronic, non-progressive impairment of muscle tone, strength, coordination, or movements. Cerebral palsy is due to cerebral insult or injury before birth, during delivery, or in the perinatal period. The most common form of cerebral palsy (75% of cases) involves spasticity of the limbs. Depending on the type and severity of the motor deficits, associated neurologic deficits or disorders may occur as follows: seizures in up to 50%; mild intellectual disability in 26%; and severe intellectual disability in up to 27%. Disorders of language, speech, vision, hearing, and sensory perception are found in varying degrees and combinations. Gross motor milestones of concern with cerebral palsy include head control at age 2 months, rolling at age 4 months, sitting at age 6 months, and walking at age 1 year. Infants with cerebral palsy may have significantly delayed gross motor milestones or show an early hand preference when younger than 1.5 years, suggesting the relative weakness of 1 side. While this patient demonstrates a hemiplegic pattern, other patterns are possible, such as monoplegia or quadriplegia. The findings on physical examination in those with spastic cerebral palsy are variable and are predominantly those of spasticity, clonus, hyperreflexia, ataxia, and involuntary movements. Physical indicators of cerebral palsy include joint contractures secondary to spastic muscles, growth delay, and persistent primitive reflexes. Microcephaly is frequently present. In patients with hemiplegia, the affected arm and leg may be smaller and shorter than the unaffected limbs. Duchenne-type muscular dystrophy, which is also called pseudohypertrophic muscular dystrophy, is a progressive disease that affects boys. Early signs of disease include pseudohypertrophy of the calf, planovalgus deformity of the feet, and proximal muscle weakness. Muscle weakness in the hips may be exhibited by the Gower sign, in which the patient uses the arms to support the trunk while attempting to rise from the floor. Other signs are hesitance when climbing stairs, acceleration during the final stage of sitting, and shoulder weakness. Lesch-Nyhan syndrome is characterized by hyperuricemia, self-mutilative behavior, choreoathetosis, spasticity, and intellectual disability. Hyperuricemia results from urate overproduction and can cause uric acid crystalluria, nephrolithiasis, obstructive uropathy, and gouty arthritis. The onset of Huntington's disease is usually age 30-50. The initial symptoms may consist of either abnormal movements or intellectual changes, but ultimately both occur. The earliest mental changes are often behavioral. Symptoms may include irritability, moodiness, antisocial behavior, and a psychiatric disturbance. A more obvious dementia subsequently develops. Dyskinesia may initially manifest as fidgetiness or restlessness, but eventually, choreiform movements and some dystonic posturing may occur. Progressive rigidity and akinesia (rather than chorea) sometimes occur in association with dementia, especially in cases with childhood onset. Rett syndrome affects 1 of every 10,000 to 15,000 girls. After 6-18 months of normal development, motor skills and mental abilities seem to slowly regress. Certain handwringing and other stereotyped hand movements appear as the disease progresses and are characteristic of the disease.

Case A 19-year-old male college freshman living in a dormitory presents with fever and chills, intense headache, vomiting, and stiff neck. His temperature is 39.2°C and pulse is 140/min. After examination, the clinical diagnosis of meningitis is made. Lumbar puncture reveals fluid that is turbid and under pressure. The cerebrospinal fluid (CSF) and blood are sent for microbiological studies. The patient is placed on intravenous ceftriaxone. Gram stain of smears made from centrifuged deposit of CSF shows plenty of polymorphonuclear leukocytes, intracellular Gram-negative diplococci, and a few extracellular Gram-negative diplococci. A culture of CSF on chocolate agar media incubated at 37°C in the presence of 5% CO2 grows colonies of Gram-negative diplococci that are oxidase-positive. By conventional tests and agglutination reaction, the isolated pathogen is identified as Neisseria meningitidis group B. Question What, if any, prophylactic measures may be taken by other students staying in the dormitory to prevent transmission of the infection?

Correct answer: Chemoprophylaxis with rifampin Explanation At present, chemoprophylaxis is the only prophylactic measure available against N. meningitidis group B. Among the drugs listed, rifampin is the one used for this purpose. Mass prophylaxis with rifampin during large outbreak settings is a cause for concern, as it might lead to circulation of rifampin-resistant strains. However, mass chemoprophylaxis with rifampin might be considered in outbreaks involving limited populations, such as in a dormitory, single school, or college. It would be especially advisable in cases caused by serogroup B, for which an effective vaccine is not yet available. Rifampin resistance develops due to mutations in the rpoB gene. Alternative drugs that are recommended for chemoprophylaxis are ciprofloxacin and ceftriaxone. Ceftriaxone is administered intramuscularly. Ciprofloxacin is not indicated in small children. Vancomycin is not used for chemoprophylaxis of meningococcal infection. Most of the Gram-positive normal flora is susceptible to this drug, so this antibiotic is used as a constituent of modified Thayer martin medium for selective isolation of pathogenic neisseria. The medium can be used for isolation of N. meningitidis from nasopharyngeal swabs for detection of carriers. Oral penicillin is not used for chemoprophylaxis of meningococcal meningitis, although most strains of meningococci are sensitive to penicillin. It is not found effective in eradicating meningococci from the nasopharynx. Tetravalent meningococcal conjugate vaccine (MCV4) is not useful against group B meningococcal infection. It contains polysaccharides from A, C, Y, and W-135 serogroups of N. meningitidis conjugated with diphtheria toxoid, and it is useful for preventing infection by these 4 serogroups. This vaccine is considered more effective than MPSV4. It stimulates both B-cell and T-cell dependent immune responses. As a result, the vaccine induces a strong booster effect that confers long-term protection, and it helps in reducing nasopharyngeal carriage. It is also reported to be safe and immunogenic in 2- to 10-year-olds. Centers for Disease Control and Prevention recommends this vaccine for routine immunization of adolescents, college freshmen living in dormitories, military recruits, and other populations at increased risk of developing invasive meningococcal disease. MCV4 has been licensed for use in individuals 11-55 years old. Close contacts of the patient with meningococcal disease are at greater risk of acquiring the infection. Chemoprophylaxis is indicated in those individuals and in populations with increased carrier rates. Nasopharyngeal mucosa is the site of colonization of N. meningitidis. Overcrowding, as is often found dormitories, causes an increase in carrier rates. Prevalence of carriage is reported to be highest among adolescents and young adults. Transmission occurs through direct contact with large droplets of respiratory secretions from patients or asymptomatic carriers. After colonization, development of invasive disease depends on the strain, environmental, and host factors. Chemoprophylaxis eradicates the carrier state and thus prevents spread of infection. It should be administered as early as possible, ideally within 24 hours of identifying the index case, in order to be effective in preventing secondary cases.

Case A 55-year-old man starts to develop behavioral and mental changes. In addition, he is having coarse, spasmodic, involuntary movements that involve his face and his extremities. Question What is the classification of this abnormal movement?

Correct answer: Chorea Explanation Chorea is a spasmodic involuntary movement that involves the face and limbs. Chorea can be seen with Huntington's disease (Huntington's chorea). Athetosis is slow, writhing, twisting involuntary movement. Almost all parts of the body can be affected by athetosis. Speech can also be affected when the athetosis is affecting the tongue. Depending on the characteristic of the movements that are seen, it is sometimes modified and called choreoathetosis or athetotic dystonia. Athetosis can be seen with cerebral palsy and some other conditions. Ballism is a flailing, jerky movement of the limbs. It affects the proximal muscles. If the abnormal movement is on one side, it is called hemiballism. Asterixis is also called "liver flap". It is a flapping movement of the hands or arms. During the physical examination to test for asterixis, have the patient hold up his hands (dorsiflex) and spread his fingers. Asterixis is seen with metabolic encephalopathies. Dystonia is an abnormal movement where there is abnormal contraction of muscles. Dystonia can be divided into primary and secondary. Spasmodic torticollis is a form of dystonia.

Case A 40-year-old man presents with a 3-week history of a daily headache that awakens him from sleep. The headaches occur around the left eye. He notes that the left eye waters and his left nostril gets stuffy with each headache. Question Ico-delete Highlights What is the most likely diagnosis?

Correct answer: Cluster headache Explanation Cluster headaches are typically seen in middle-aged men. Unilateral periorbital pain can occur daily for several weeks. They are often accompanied by ipsilateral nasal congestion, rhinorrhea, lacrimation, and redness of the eyes. Episodes often occur at night, and they last 15 minutes to 3 hours. Sinusitis commonly presents with unilateral face fullness, pressure, and tenderness over the cheeks. Tension headaches are caused by muscle contractions that usually elicit a "band-like" tightness rather than throbbing pain. Often brought on by stress, they are usually not associated with nausea or vomiting, and they more commonly cause bilateral rather than unilateral pain. Intracranial masses cause an increase in intracranial pressure, which can cause headaches with vomiting. Headaches related to intracranial masses are typically worst in the morning, as recumbency increases intracranial pressure. Headaches due to intracranial pathology are likely to worsen over a short period and to be progressive in nature, rather than presenting as a cyclical course. Migraine headache pain is usually described as pulsatile in nature and is commonly unilateral. Other common manifestations include nausea, vomiting, and photophobia. They are more common in women than in men.

Case A 53-year-old man presents with a 2-week history of severe headaches. The headaches occur primarily at night. The patient is pacing while he is talking. The pain surrounds 1 eye and lasts for 30 to 90 minutes. He also states that there is ipsilateral lacrimation, conjunctival injection, and nasal congestion during the attacks. The patient states that he has had these headaches 2 or more times in a day over a period of several weeks. He cannot point to any one thing that causes them. On examination, the patient has features of partial Horner's syndrome. His vital signs are temperature: 97.0°F, heart rate: 80/min, respiration 16/min, and blood pressure: 126/80mmHg. Question What type of headache is the patient experiencing?

Correct answer: Cluster headache Explanation Cluster headaches come in groups and attacks may be separated by months or years. There are 2 types: episodic and chronic. The patient is often a middle-aged man. Cluster headaches are characterized by excruciating, unilateral pain (stabbing sensation) often occurring in the ocular, frontal, or temporal areas. These headaches generally last for a period of 15 minutes to 3 hours, recurring at the same time of day. They typically occur during the night. Pain often radiates to the upper teeth, jaw, and neck. The pain is normally accompanied by 1 or more of the following: ipsilateral conjunctival injection or lacrimation; ipsilateral nasal congestion or rhinorrhea; ipsilateral eyelid edema; ipsilateral forehead and facial sweating; ipsilateral miosis or ptosis; and a sense of restlessness or agitation. Restlessness is one of the characteristic features of cluster headaches noted in patients; behaviors such as pacing or rocking the head and trunk with head in hands are common. Other symptoms of cluster headache include facial flushing or pallor, tenderness on palpation of the ipsilateral carotid artery, bradycardia, and tenderness of the scalp and facial areas. The absence of aura, nausea, or vomiting helps in distinguishing cluster headaches from migraines. The treatment of choice for acute cluster headaches includes the administration of oxygen (7L per minute for 15 minutes), sumatriptan, or a combination of both. Other therapeutic measures include intranasal dihydroergotamine, intranasal capsaicin, or intranasal lidocaine. Prednisone and verapamil are often used in prophylaxis.

Case A 45-year-old man presents with left-sided headaches. He describes the pain as sharp, almost stabbing in nature. Headaches last 30-60 minutes, he rates the pain at 9/10, and he finds it difficult to sit still during the episodes. He notes that, during this time, he also has tearing of his left eye. The headache has been occurring daily for the past 15 days. He had a similar episode occur about 2 months ago, and the headaches resolved spontaneously. Question What is the most likely diagnosis?

Correct answer: Cluster headache Explanation Cluster headaches have a classic presentation of severe unilateral pain that lasts anywhere from 15 minutes to 3 hours. These "clusters" may occur once or multiple times daily, and they tend to occur daily for weeks to months. Attacks may be accompanied by nasal congestion, lacrimation, rhinorrhea, conjunctival erythema, miosis, ptosis, or anhidrosis on the affected side. Following a series of attacks, patients typically have remission of several weeks or even months. Brain tumors are associated with headaches, but they typically do not follow the above pattern. Headaches occurring with brain tumors may become more severe over time, and they usually do not resolve spontaneously. Migraine headaches tend to be associated with unilateral pain, but the attack typically lasts 4-72 hours. Tension headaches are less severe than cluster headaches. Patients usually describe the pain as a bilateral tightness sensation. The headaches are not usually associated with other symptoms. Transient ischemic attack mimics the symptoms of stroke. Symptoms usually last 1-2 hours, but most last less than 24 hours.

Case A 35-year-old man presents due to headaches. He describes them as only occurring on 1 side of his head; they tend to have the sensation of a sharp stabbing pain just above his left eye. Each headache lasts for 2 hours at a time; they have been occurring daily for 2 - 3 weeks, and they have had the same pattern every 3 - 4 months for the past 2 years. During these periods, the patient admits to his nose feeling congested; his left eye appears red, and it excessively waters. Aggravating factors appear to be alcohol and stressful situations. He denies any known family history of headaches similar to his or migraines. The physical examination is within normal limits. Question Based on the history and physical examination, what does the patient most likely have?

Correct answer: Cluster headaches Explanation Cluster headache is the correct answer. Cluster headaches are types of headaches that occur more commonly in middle-aged men. Patients will complain of severe unilateral periorbital pain that occurs daily for several weeks; they are potentially associated with 1 or more of the following symptoms: ipsilateral nasal congestion, rhinorrhea, lacrimation, or redness of the eye. Patients are restless and agitated during the acute phase of this type of headache. Cluster headaches more typically occur at night and awaken the patient from sleep. These episodes vary in length from around 15 minutes up to 3 hours; at the same time, these occurrences may last for several weeks and then resolve for many weeks at a time. Tension headache is a common type of headache and a primary headache disorder, but it is not the correct answer. Tension headaches are also called stress headaches. Patients will describe pericranial tenderness that is described as vice-like or tight; it is exacerbated by emotional stress, fatigue, noise, or glare. These headaches are usually general, and they are not associated with focal neurological symptoms. Classic migraine headaches are described as being unilateral in nature and usually pulsatile. They may be accompanied by nausea, vomiting, photophobia, and even phonophobia. An aura, usually visual, will precede the head pain, but migraines can also occur without auras as well. The description of the headache in the patient above is not similar to that of a migraine headache. Cerebral aneurysms, which are also called intracranial aneurysms, are most commonly asymptomatic until they rupture. When a rupture occurs, a subarachnoid hemorrhage occurs, and patient will describe their headache as "the worst headache I've had in my life". Intervention must be initiated quickly to ensure survival. Hemicrania continua are considered a separate type of primary headache syndrome. It also presents with unilateral head pain and autonomic symptoms, but unlike cluster headaches, patients will have continuous pain without any pain-free time periods. These types of headaches also are completely resolved with indomethacin.

Case Ico-delete Highlights A 40-year-old women presents with a 7-day history of pain in her right arm. The patient denies any trauma or injury to this extremity just prior to the pain starting, but she does admit to having a Colles' fracture in this arm around 2 months ago. She denies any injury to her back, neck, or other musculoskeletal system prior to the event of pain. She describes the pain burning and throbbing with an extremely diffuse, uncomfortable aching accompanying it. She also states that the limb has become extremely sensitive to touch and to cold; she notes that it appears somewhat more swollen than her left arm. The patient is very upset because she does not know why her arm is so painful when she has not done anything to it. She is a nonsmoker; she does not drink, and she exercises three times a week. A physical examination is conducted of the extremity; it reveals a slightly cyanotic, mottled right arm with generalized pain of the entire extremity. Pulses are faint (1+), and ROM is limited. Question What is the most likely diagnosis?

Correct answer: Complex regional pain syndrome Explanation The patient being described is most likely in Stage I of Complex Regional Pain Syndrome (CRPS). This disorder is rare, causing instability of the vasomotor and autonomic components. CRPS is a disorder of a body region, most commonly the extremities, that is characterized by pain, swelling, limited range of motion, vasomotor instability, skin changes, and even patchy bone demineralization. Frequently, symptoms will begin after the patient has sustained an injury, had surgery, or experienced a vascular event, such as a stroke. 35% of patients with CRPS report not having a distinct precipitating event. Stage 1 of CRPS characteristically has such findings such as pain in the limb (burning/throbbing), diffuse, uncomfortable aching, sensitivity to touch or cold or even localized edema. The distribution of pain is not compatible with a single peripheral nerve, trunk, or root lesion. There will also be signs of variable vasomotor disturbances that result in altered color and temperature. At this stage if a radiograph was completed, most likely it will be normal but could show signs of patchy demineralization. A fracture would not produce such a disperse pattern of pain, swelling, and skin changes. If a fracture were present, it potentially would have signs at the site of the break, not throughout the extremity. Deep venous thrombosis is much lower on the differential due to the fact that it is much more common to occur in the lower extremity. No history gives a reason to believe that patient may be hypercoagulable at this point as well. A patient who has multiple sclerosis will more commonly have the following presenting symptoms: sensory dysfunction, vision loss, motor dysfunction, diplopia, gait and balance issues, vertigo, Lhermitte sign, or even bladder issues. Patients with MS will experience significant pain due to their illness. The patient scenario described above is inconsistent in similarity to pain in a patient with MS. Peripheral vascular disease is extremely low on the differential due to the lack of risk factors (smoking, history of coronary or other atherosclerotic disease events, her age) and the fact that the majority of the time peripheral vascular disease affects the lower extremities more than the upper extremities. The decreased pulses could lead one to think of a peripheral vascular disease; however this as well as the other signs and symptoms are characteristic of the confirmed diagnosis of CRPS.

Case A 40-year-old woman presents with a 7-day history of pain in her right arm. The patient denies any trauma or injury to this extremity just prior to the pain starting, but she does admit to having a Colles' fracture in this arm around 2 months ago. She denies any injury to her back, neck or other musculoskeletal system prior to the event of pain. She describes the pain as burning and throbbing with an extremely diffuse, uncomfortable aching accompanying it. She further states that this limb has become extremely sensitive to touch and cold; it does appear somewhat more swollen than her left arm. The patient is very upset; she does not know why her arm is so painful when she has not done anything to it. She is a nonsmoker. She does not drink, and exercises 3 times a week. Physical examination conducted of the extremity reveals a slightly cyanotic, mottled right arm with generalized pain of the entire extremity. Pulses are faint (1+) and ROM is limited. Radiograph studies are obtained and reveal spotty areas of apparent osteopenia of the right upper extremity. Question What is the most likely diagnosis for this patient?

Correct answer: Complex regional pain syndrome Explanation The patient being described is most likely in stage I of complex regional pain syndrome (CRPS). CRPS is formerly known as reflex sympathetic dystrophy. This disorder is rare, causing instability of the vasomotor and autonomic components. CRPS is a disorder of a body region, most commonly the extremities, that is characterized by pain, swelling, limited range of motion, vasomotor instability, skin changes, and even patchy bone demineralization. Frequently symptoms will begin after the patient has sustained an injury, had surgery, or experienced a vascular event, such as a stroke. 35% of patients with CRPS report not having a distinct precipitating event. Stage 1 of CRPS characteristically has such findings such as pain in the limb (burning/throbbing), diffuse, uncomfortable aching, sensitivity to touch or cold, or even localized edema. The distribution of pain is not compatible with a single peripheral nerve, trunk, or root lesion. There will also be signs of variable vasomotor disturbances that result in altered color and temperature. At this stage, if a radiograph was completed, it will most likely be normal, but could show signs of patchy demineralization or osteopenia; more severe stages will be accompanied by significant signs of osteopenia. A fracture would not produce such a disperse pattern of pain, swelling, and skin changes. If a fracture were present, it potentially would have signs at the site of the break, not throughout the extremity. Radiographs would reveal a disruption or abnormality in the patterns of the bone. Deep venous thrombosis (DVT) is much lower on the differential due to the fact that it is occurs much more commonly in the lower extremities. No history gives a reason to believe that patient may be hypercoagulable at this point as well. Often, in cases of DVT, a radiograph will not reveal any pertinent abnormalities. A patient who has multiple sclerosis (MS) will more commonly have the following presenting symptoms: sensory dysfunction, vision loss, motor dysfunction, diplopia, gait and balance issues, vertigo, Lhermitte sign, or even bladder issues. Patients with MS will experience significant pain due to their illness. The patient scenario described above is inconsistent in similarity to pain in a patient with MS. MRI of the brain of cervical cord will reveal the pathology that is typical in MS; usually, the presence of multiple hypointense 'black holes' representing areas of permanent axonal damage is strongly suggestive of MS as the diagnosis. Initially, no significant radiographic abnormalities may be found. Peripheral vascular disease is extremely low on the differential due to the lack of risk factors (e.g., smoking, a history of coronary or other atherosclerotic disease events, her age) and the fact that the majority of the time peripheral vascular disease affects the lower extremities more than the upper extremities. The decreased pulses could lead one to think of a peripheral vascular disease; however, this as well as the other signs and symptoms are characteristic of the confirmed diagnosis of CRPS. Due to this being more vascular in origin, no obvious abnormalities will be found on radiographic studies.

Case A 42-year-old woman presents with a 1-month history of severe, worsening pain in her right foot, ankle, and lower leg. The pain is constant and burning. She reports some initial swelling and warmth in the leg, which has lessened, and now her right lower extremity is always cool to the touch. Hair growth has dramatically decreased on her right leg. She has tried multiple over-the-counter pain medications and topical analgesics with no relief. She denies trauma to the affected limb and reports her symptoms just "came out of the blue". Her sleep is poor, and she reports fatigue secondary to unrelenting pain, but she denies any other symptoms. Past medical history: Medical conditions: None Medications: No chronic medications; multiple over-the-counter pain treatments Allergies: None Surgeries: Right knee ACL repair 4 months ago; bilateral tubal ligation at age 32 Family history: Unremarkable Social history: Denies alcohol, tobacco, and drugs. She lives at home with her husband and children, and she works as a secretary. The patient is thin, with normal vital signs. On neurologic exam, her right lower calf, ankle, and foot appear to be hypersensitive to light touch stimulus (cotton swab), compared to the left. The same region, which spans several dermatomes, exhibits decreased sensation to pinprick and decreased range of motion. The skin of the right lower extremity is cool; there are no lesions, rash, edema, or erythema. Ridges are noted on the right toenails, but not the left. The rest of her physical exam is normal. Blood count, glycohemoglobin, metabolic panel, D-dimer, as well as the X-ray and ultrasound of the entire right lower extremity are normal. Question What is the most likely diagnosis?

Correct answer: Complex regional pain syndrome Explanation This patient most likely has complex regional pain syndrome (CRPS). CRPS is indeed a complex syndrome often occurring in a limb; patients exhibit vasomotor and neurologic symptoms out of proportion with objective findings. It often occurs following some type of event, such as a surgery (note the patient's recent ipsilateral knee surgery), fracture, stroke, or myocardial infarction. Most of the time, there is no nerve lesion. CRPS is classified into 3 stages, and symptoms can vary depending on the stage. It is characterized by persistent burning and/or throbbing pain in an extremity. Swelling, redness, and increased hair/nail growth occur in stage I; it may progress to cool, pale skin and even osteoporosis in the latter 2 stages. Allodynia, the phenomenon in which a normally painless stimulus produces significant pain response, is a common finding. No single exam finding or test is diagnostic of CRPS, which is a clinical diagnosis. A deep venous thrombosis (DVT) could present with unilateral leg pain. However, the physical exam would be more likely to reveal redness, erythema, swelling, and a palpable tender cord. With a negative leg ultrasound and D-dimer, DVT is not likely. Diabetic neuropathy is a common cause of burning, persistent leg symptoms. However, this patient's presentation does not suggest diabetes, and the quick progression of unilateral symptoms is not consistent with diabetic neuropathy. A spinal cord lesion could present with unilateral or bilateral lower extremity neurologic symptoms. However, it is not a common cause of unilateral burning leg and foot pain. This patient's presentation is not consistent with a spinal cord lesion. Thrombophlebitis is the presence of inflammation of a superficial vein, often with a history of varicose veins. It is relatively common in the lower extremities. However, the physical exam would be likely to have similar symptoms as the DVT (redness, edema, and a painful, palpable cord)

Case Ico-delete Highlights A 35-year-old man presents with a 2-week history of constant burning and throbbing pain in his left hand. The pain also seems to affect his distant forearm. Contact with normal clothing and bed sheets worsen his hand pain. He also reports that his hand is swollen. He denies trauma to his hand, but does report an uncomplicated left wrist fracture 3 months ago, which resulted from a sports accident. By the time of cast removal at 7 weeks post-injury, patient denied any pain or edema in the affected limb. He was instructed to return to normal activities, as tolerated. He was initially treated with ibuprofen and opioid pain pills. He discontinued all pain medicines within 2 weeks of the fracture. He now describes his hand pain as an 8 out of 10, on a 1 - 10 pain scale. On physical exam, the left hand and forearm are noted to have some localized edema, warmer temperature, and increased hair growth, compared to his right hand and forearm. No rash or skin lesions are noted. With even light palpation of the affected region, the patient cries out in pain. Range of motion is decreased, and reflexes are increased the left upper extremity (in comparison to the right). The rest of his exam is normal. X-ray of the left wrist and hand are normal, with good fracture resolution. Question What is the most likely diagnosis?

Correct answer: Complex regional pain syndrome Explanation This patient most likely has complex regional pain syndrome (CRPS). CRPS is indeed a complex syndrome often occurring in a limb; patients exhibit vasomotor and neurologic symptoms out of proportion with objective findings. It often occurs following some type of event, such as a surgery, fracture, stroke, or myocardial infarction. Most of the time, there is no nerve lesion. CRPS is classified into 3 stages, and symptoms can vary depending on the stage. It is characterized by persistent burning and/or throbbing pain in an extremity. Swelling, redness, and increased hair/nail growth occur in stage I; it may progress to cool, pale skin, and even osteoporosis in the latter 2 stages. Allodynia, the phenomenon in which a normally painless stimulus produces significant pain response, is a common finding. No single exam finding or test is diagnostic of CRPS, which is a clinical diagnosis. Factitious disorder is a psychiatric illness in which the patient feigns symptoms in order to be in the "sick role". The careful practitioner will need to be aware that CRPS exists. Otherwise, it may be tempting to assume the patient is exaggerating his symptoms, as his/her pain seems out of proportion to objective findings. Herpes zoster (shingles) can cause inflammation in a single, unilateral nerve root, leading to burning pain sensation in a limb (or other body part). The skin lesions may not appear immediately. This patient's affected region is not consistent with 1 dermatome, and herpes zoster is quite uncommon in this age group. Nerve root impingement at the cervical spine (cervical radiculopathy) may occur following neck injury or degenerative changes. It can be associated with unilateral limb pain and paresthesias, but it typically affects the limb more proximally. The distal edema and allodynia are not classically associated with cervical radiculopathy. Thoracic outlet syndrome is a condition in which the blood vessels and/or nerves become compressed as they exit the chest wall and travel down the arms. It may be unilateral or bilateral, and it can cause pain and paresthesias in the neck, arms and hands. Thoracic outlet syndrome would be more likely if the patient described repetitive upper body movements. With more localized, unilateral symptoms following a fracture, CRPS seems much more likely.

Case A 28-year-old man presents with a history of headache, vomiting, and seizure. On examination he is febrile; pulse is 110/min; and blood pressure is 110/70 mm Hg. On central nervous system (CNS) examination, he appears alert; he is oriented to time, place, and person. He has aphasia and weakness of the right hand and leg. You suspect brain abscess. Question What test would be most helpful to confirm your diagnosis?

Correct answer: Computed Tomography (CT) scan of brain with contrast Explanation The correct answer is CT scan of brain with contrast. It is an excellent tool for establishing the diagnosis and can demonstrate evidence of a ring-enhancing lesion in a well-defined abscess, as well as features of cerebral edema in the stage of cerebritis. CT aids in determining the location of the abscess, its size, number of abscesses, mass effect, shifts, and the presence of intraventricular rupture. It provides information regarding the cause, and the paranasal sinuses and mastoids are also imaged concomitantly. Another excellent test is MRI, but this is usually not as readily obtained on an emergent basis. CT scan of the brain without contrast does not form a ring around the lesion, making the diagnosis difficult. Hence, it is not the right choice. Lumbar puncture is not advised in the setting of suspected brain abscess, especially when neurological deficits are present, as intracranial pressure may be elevated and there is the risk of brain stem herniation. White blood cell count may be elevated in a patient with a brain abscess but it is a nonspecific test and not terribly helpful in making the diagnosis of brain abscess. The WBC is only elevated in about 60-70% and is normal in the remainder of patients. Ultrasonography is not used for diagnosing a brain abscess in adults because the skull causes bone artifact. In children with an open anterior fontanelle, a sonogram can be used to diagnose an abscess.

Question A 22-year-old man presents after being hit on the right temple by a baseball 1 hour ago. He lost consciousness for 2 minutes after the impact, but he did not seek immediate medical attention. He has no other significant medical problems and family history is unremarkable. On presentation, he is comatose, with no response to painful stimuli. There is anisocoria with fixed dilatation of the right pupil. Vital signs are as follows: BP 100 / 66 mm Hg, RR 14 rpm, HR 62 bpm, temperature 36.6 °C (97.9 F). What is the most appropriate next step to confirm the suspected diagnosis?

Correct answer: Computed tomography of the brain Explanation The signs and symptoms described in this clinical scenario are consistent with an acute epidural hematoma. This lesion's pathophysiology is a dural tear with laceration of an artery or, less commonly, a vein. The most commonly injured vessel is the middle meningeal artery (66% of cases), but lesions of the anterior ethmoidal artery and the dural sinuses may also occur. About 85-95% of these cases are associated with a fracture of the skull. However, there is little injury to the underlying intracranial components; because of this, the prognosis is good. Other findings, such as intracranial air or vascular malformations, may also be present. Due to the progressiveness of this pathology if not treated, immediate diagnosis and surgical intervention is required. The most commonly used method to make this diagnosis to date is a non-contrast CT of the head. Not only will this help locate the hematoma, this will also help visualize any other abnormalities that may be present, such as skull fractures. A CT scan would also enable the patient to be observed during the entirety of the procedure to ensure his stability and safety. MRI of the brain may be considered appropriate, however should not be used in any patients who are unstable. Due to the patient being comatosed with no response to painful stimuli, this would not be an acceptable choice. Burr hole craniotomy is used primarily in the treatment approach in a rapidly deteriorating patients with signs of imminent herniation. Although the patient is in a comatose state, his vitals are acceptable. Knowing if there is indeed an epidural hematoma as well as where it is located will both be accomplished via a CT of the head.

Case A 60-year-old woman presents with short-term memory loss, an inability to care for herself, and an inability to handle finances. According to her neighbor, she recently left something on a burning stove and almost burned her house down. There is no family with her. Question Ico-delete Highlights What is the next step in patient management?

Correct answer: Contact adult protective services Explanation Psychiatric emergencies take on many different types. When a patient is dangerous to himself/herself or other people, it requires immediate attention. It is crucial to call the police if they are homicidal or suicidal or threatening to hurt a child. It is also important to do so if they are potentially dangerous to you or other healthcare professionals. Other forms of emergencies may be abuse or neglect of an older person. In most states, it is mandated that, at some point in the assessment process, you are to contact the nearby adult protective services agency for evaluation and follow up. They are designated by the state to have legal authority to complete emergency guardianships, emergency certificates for emergency hospitalizations, and other legal actions. If a client is suicidal and is willing to be hospitalized, it is probably important to do so. If they will not voluntarily go, which is often the case, you will have to involve adult protective services and the police. Also, if you have an older adult who is suffering from self-neglect or abuse, it is mandated that you refer him/her to adult protective services for case management or follow up. Many physicians have a social worker do this for them, which is appropriate, but it is very important to collaborate together and work as a team. Also, it is very important that office physicians refer suspected self-neglect or abuse cases to adult protective services in order to complete the medical testing and work-up that may be needed. Self-neglected older adults often do not show up for their medical testing, forget about the appointments, or refuse to go along with the process. If an older adult is assaulted and is a victim of a crime such as robbery, assault, or rape, it is very important to contact the victim of crime unit for case management and follow up. Most hospital emergency rooms have social workers that can consult and follow up on necessary referrals. Often, depressed or demented older adults do not seek diagnosis or treatment because they either forget or do not want to have the stigma of having a mental health problem.

Case A 16-year-old boy sustained a head injury during wrestling practice. During a practice match, the boy's head struck the concrete floor, and he then received a secondary blow to the head from a nearby wrestler. He briefly lost consciousness and then was dazed, confused, and slow to respond to questions for a few minutes. His vision became briefly blurry and double. His balance was initially unstable but improved with time; however, he developed lethargy and a headache. He did not have any nausea or vomiting. His mother was instructed to have him checked for a head injury. On exam about an hour later, he appears tired but alert and oriented. He's able to recount the events leading to his injury, but he's somewhat unsure of what happened right after. His vitals are stable, and visual acuity is 20/40 in both eyes; head and neurologic exam are normal, including equal and reactive pupils and cranial nerves. His very anxious mother says he suffered a concussion 2 seasons ago while playing football and had to sit out of practice for 2 weeks. Question What is the preferred management of this patient?

Correct answer: Cranial CT scan Explanation In a patient with neurological symptoms after a head injury, head imaging is advised. Cranial CT is considered the imaging modality of choice for traumatic brain injury after a thorough history, physical, and neurologic examination. Compared to MRI scans, CT scans are more sensitive for hyperacute and acute intracranial hemorrhage and are more quickly, easily, and cheaply performed. In the absence of available CT scanning, MRI scans more subtle contusions or hematomas, but it is rarely the first imaging modality. The consensus is that observation alone (after a thorough history and physical plus normal neurologic exam are performed) can be used as the primary management strategy in patients who present with no loss of consciousness, amnesia, vomiting, headache, or mental status changes. However, this would not be the recommended approach in a patient such as the one in this scenario who had mental status changes. Skull radiographs have a very limited role in the head injury evaluation of a child with loss of consciousness. There is a substantial rate of false-positive results where a fracture is detected without intracranial injury. Furthermore, intracranial injury can occur in the absence of a skull fracture. If imaging is desired, CT scanning is the modality of choice based on increased sensitivity and specificity. The use of ultrasound in traumatic brain injury is being evaluated to measure the optic nerve distensible sheath diameter. This method can predict raised intracranial pressure. However, it is not the standard technique for evaluating the presence of intracranial injury.

Case A 50-year-old woman with Hodgkin's disease has been treated with cytotoxic drugs and steroids. She is admitted to the hospital with a 5-week history of headache, low-grade fever, visual disturbances, and confusion. The patient also has a 2-month history of progressive memory loss. Physical examination reveals a stiff neck. After lumbar puncture, initial studies reveal a WBC of 100 cells/mm3, glucose of 20 mg/dL, and protein of 100 mg/dL. CSF testing with India ink is positive. Question What is the cause of the CNS disease?

Correct answer: Cryptococcal meningitis Explanation Cryptococcal meningeal infection should be always excluded by a lumbar puncture in an immunocompromised patient. CSF findings show: high cell count (>20 leukocytes/mm3), elevated protein, low glucose, positive India ink preparation, and positive culture. Cryptococcus neoformans is an encapsulated yeast found in the environment, particularly soil contaminated with bird droppings. Humans are infected by inhaling the spores. This type of meningitis is more sub-acute or chronic. Cryptococcal meningitis is often seen in patients with conditions that cause a compromised immune system, such as AIDS, steroid therapy, immunosuppression induced after organ transplantation, lymphoproliferative disorders, diabetes, tuberculosis, and sarcoidosis. Typical treatment includes amphotericin B and flucytosine. Bacterial meningitis is often caused by organisms such as H. influenzae, N. meningitidis, and S. pneumoniae, usually with preceding respiratory infections. Bacterial meningitis is more acute in presentation. The CSF will not be positive for India ink in cases of tuberculous, aseptic, bacterial meningitis, or HSV encephalitis.

Case Ico-delete Highlights A 76-year-old woman presents because her children are concerned that that she might have dementia. She states that she is doing reasonably well, except that she sometimes sleeps less deeply and wakes up more often than she did several years ago. According to her children, she is slower than before, and her memory has been getting worse over the last 3 years; she has difficulties recalling the specific date of an event (although she can describe the event itself). She also has a great deal of trouble with names, but she can easily recognize people. She always says: "It is on the tip of my tongue, but..." Aside from hypertension that is under control, she does not have any other health problems. She has been a widow for about 10 years. Her older brother was diagnosed with dementia. Physical examination today is within normal limits for the age, and neurological examination is nonfocal. Question What should be your next diagnostic step?

Correct answer: Mini-mental status examination Explanation The mini-mental state examination (MMSE) is a brief 30-point screening test for cognitive impairment. In persons older than 70, the ability to memorize, acquire, and retain new information and recall names is diminished. Memory function may be disturbed in this way despite the relative retention of other intellectual abilities. It particularly applies to episodic memory, and it is manifested by difficulties with recalling names and/or specific dates. This disturbance is known as benign senescent forgetfulness or age-associated memory impairment. It progress very little or not at all over a period of many years, and it does not interfere significantly with the person's work performance or activities of daily living. Your patient most probably has this sort of age related cognitive decline. MMSE can be used also as follow-up of her condition. Electroencephalography (EEG) is recording of spontaneous electrical activity of the brain; it is primarily used in the diagnosis of epilepsy, and it is sometimes used in the diagnosis of coma, encephalopathies, sleep disorders, and brain death. Your patient's sleep complaints are normal in advanced age. Stanford-Binet Intelligence Scales cannot be used to score adults. MRI of the brain is indicated when structural changes are suspected; however, they are not likely in this patient due to her normal neurological examination. You should perform a thyroid function test if you find out that a patient has dementia - slowness is a part of normal aging.

Case Ico-delete Highlights A 48-year-old HIV-positive man starts to develop headaches. At first, he attributes the headaches to stress; however, they persist and become worse over the next few weeks. He develops nausea and vomiting, and he thinks he has a fever. He starts to become confused, so he seeks medical attention. On physical examination, his temperature is 100°F. He has signs of meningeal irritation. A lumbar puncture is performed, and there is an elevation of his CSF pressure. Cerebrospinal fluid is sent to the lab. The CSF is centrifuged. A drop of India ink is placed on the slide along with a drop of the spun CSF. The India ink test is positive. Question What organism is likely causing the patient's symptoms?

Correct answer: Cryptococcus neoformans Explanation This patient has signs and symptoms of cryptococcal meningitis. Cryptococcus neoformans is a yeast with a capsule made of polysaccharides. India ink exam is positive for Cryptococcus neoformans. When visualized on high dry magnification, encapsulated organisms (Cryptococcus neoformans) have capsules that look like halos because they exclude the India ink. Cryptococcus neoformans can cause meningitis, and there is an increased risk of Cryptococcus neoformans meningitis in immunosuppressed individuals. India ink exam would not be positive with Pneumocystis jiroveci, Cryptosporidium, Coccidioides immitis, or Histoplasma capsulatum.

Case A 23-year-old woman was diagnosed with multiple sclerosis approximately 2 years ago; she presents to request information regarding her risk of exacerbation should she become pregnant. Question What does current information on the subject indicate?

Correct answer: Decreased risk during pregnancy with increased risk during the post partum state Explanation Confavreux et al have reported the exacerbation rate per year to be 0.7 in non-pregnant state, decreased during pregnancy (0.5 during first trimester, 0.6 during the 2nd trimester, and 0.2 during the 3rd trimester), and increased during the 3-month postpartum state (1.2). However, when calculated over the whole pregnancy and post partum state, the risk of exacerbations is increased very little, if at all.

Case Ico-delete Highlights A 70-year-old woman presents with a 1-year history of short-term memory loss. She did not seek medical attention earlier because she believed the memory loss to be age-related. Her husband has also noticed that she is unable to perform her regular daily activities and is often misplacing things. Recently, she was unable to get back home after her evening walk; neighbors brought her home. The husband also feels she has been acting strangely, and she is suspicious of her own son. Her past medical history is significant for hypertension, and she has been on medication for the past 3 years. On clinical examination, she appears conscious and alert; PR is 70/min and BP is 120/74 mm Hg. She is afebrile. Question What is the provisional diagnosis?

Correct answer: Dementia of Alzheimer's disease Explanation Dementia of Alzheimer's disease involves multiple cognitive deficits, including 1 or more of the following: aphasia (language disturbance), apraxia (the inability to carry out motor activities despite intact motor function), agnosia (the inability to recognize familiar objects), and the disturbance of executive activities (e.g., planning, organizing, sequencing, or abstracting). There can be changes in personality and behavior, in addition to the inability to learn and recall new information. Long-term memory is affected. The disease has a gradual onset and continuing decline. Focal neurological signs and symptoms are usually absent. If the disease occurs before age 65, it is considered early-onset. If it occurs after age 65, the disease is late-onset. Dementia due to medical conditions may present with similar (or the same) symptoms as other dementias, but it is due to medical or neurological diseases (e.g., head trauma, Parkinson's disease, Pick's disease, Huntington's disease, HIV, multiple sclerosis, and vitamin deficiencies). Vascular dementia has the same general symptoms and signs as Alzheimer's disease, except the onset is usually more sudden and acute. In addition, it is picked up easily by clinical or laboratory evidence of a vascular cause, a history of cerebrovascular disease, or multiple infarctions. Many elderly suffer from major depression. Symptoms include short-term or recent memory loss, fatigue, insomnia or hypersomnia, low appetite, weight loss, overeating, and an inability to handle activities of daily living. In major depression, deficits in various cognitive domains may be present. Temporal lobe magnetic resonance imaging (MRI) is helpful in differentiating dementia of Alzheimer's disease from depression and other disorders that may cause diagnostic difficulties in clinical practice. Substance-induced dementia includes cognitive defects caused by inhalants, sedative drugs, hypnotic drugs, prescription side effects, overdoses of prescription drugs, or other substances. This is common in the elderly.

Question A 35-year-old woman with Down syndrome (trisomy 21) is brought in by her family. Previously, she had learned to read street signs and write her name. Until a year ago, she attended a sheltered workshop where she sorted color coded components for appliance repair. In the past year, she has become more impaired and can no longer reliably recognize people she does not see every day. She is no longer able to dress herself, and she is unable to work. On mental status examination, she is pleasant but distracted, and she is oriented to person only. Her speech is grammatically fragmented and dysarthric. She cannot remember what she ate for lunch an hour before. She does not recognize a picture of a stop sign. What is the most likely diagnosis?

Correct answer: Dementia of the Alzheimer type Explanation This patient is an example of a person with trisomy 21 who has survived into middle age; as is almost universal for such patients, she has now developed Alzheimer disease. Although this patient was suffering from intellectual disability at baseline, she now warrants a diagnosis of dementia because her functional level has markedly decreased. She has classic symptoms of the disease, including decreased language capacity, agnosia, apraxia, and loss of short-term memory. She is not depressed, ruling out pseudodementia. Oppositional defiant disorder and dependent personality disorder do not begin in adulthood. In any case, she is not oppositional, just incapacitated.

Case An 80-year-old man presents with a gradual decline in memory. He does not recognize himself, and he has difficulty performing activities of daily living. On exam, the patient demonstrates no evidence of a serious medical illness, but a CT scan shows some cortical atrophy. The patient scored poorly on the mini-mental status exam. Question What is the most likely diagnosis?

Correct answer: Dementia of the Alzheimer's type Explanation Dementia of the Alzheimer's type usually consists of a gradual decline in memory. Agnosia, aphasia, or apraxia will also be found. Agnosia hinders patients from recognizing themselves, other familiar objects, or people. Aphasia is the development of a speech problem. Patients become dependent on familiar words, talk around the issues, or may even become mute. Apraxia causes the inability to perform motor acts. Afflicted individuals are unable to draw objects, copy designs, or write complete sentences. Along with the other symptoms of Apraxia, patients may also experience loss of executive functioning, making them unable to organize themselves in order to perform activities of daily living. Treatable diseases have been ruled out, and there is no evidence of delusions or an affective disorder. There is usually an impairment of activities of daily living. Vascular dementia presents in a similar manner, but in addition to impaired memory, there is usually impairment in motor acts and focal neurological deficits; this type of dementia usually occurs after 1 or more infarctions. The patient may also score poorly on mini-status exams. Dementia due to HIV disease presents with impairment in memory, usually aphasia or apraxia. It may also present with weight loss and immune system impairment; afflicted individuals may score poorly or mid-range on the mini-status exam. Dementia related to substance abuse presents with impairment in memory, and it may or may not present with aphasia, apraxia, or agnosia, but the patient is usually unable to care for themselves. This will also include a history of drug or alcohol abuse. Dementia due to multiple etiology usually presents with substance abuse and more than one serious medical condition, such as Parkinson's disease or head trauma. It may also include diseases such as multiple sclerosis, Pick's disease, or Huntington's disease.

Case You have completed comprehensive testing on an 80-year-old woman, and the diagnosis is dementia of the Alzheimer's type. You believe she is in the very early stages of the disease and want to try a medication to possibly slow disease progression. Question Which drug would you be most likely to try first in this patient?

Correct answer: Donepezil (Aricept) Explanation Donepezil (Aricept) is the correct response. Alzheimer's dementia accounts for over 60% of dementia in older adults. It develops with multiple cognitive deficits, including memory impairment and 1 or more of the following: aphasia (language disturbance), apraxia (difficulty in carrying out motor activities), agnosia (the inability to recognize familiar objects or people), and/or disturbance in organizing and planning. These symptoms usually develop over a gradual period of time and represent a distinct decline from previous functioning. Another mental illness or serious medical problem does not account for the symptoms. To make an accurate differential diagnosis, the physician must first complete a history and physical exam in order to rule out physical causes of the dementia. A complete mental status exam must then be done, along with initial lab testing. All other additional testing, such as neurological or psychological testing, should be done at a later stage after all other physical causes have been ruled out. Also, it is very important to refer the patient and family to a medical social worker at some point in order to assist the family in community resources and emotional support, especially if the diagnosis becomes dementia of the Alzheimer's type. There are no current cures for Alzheimer's disease, and no one really knows what causes it. There are 2 major drugs being used to treat some of the symptoms: Namenda (memantine) and donepezil (Aricept). Donepezil has become the treatment of choice because it has once-a-day dosing; there is also no need for close monitoring, and it causes fewer side effects. Memantine is used primarily in moderate to severely affected dementia patients. Vitamin E is being used to reduce the effects of drugs, but it is not a definitive treatment of choice. Risperidone (Risperdal) and haloperidol (Haldol) are being used as later choice drugs to treat later-stage patients who have developed psychotic or agitated reactions. Both have side effects, have to be more closely monitored, and are usually prescribed by a psychiatrist/liaison. Normal pressure hydrocephalus is characterized by gradual onset of dementia, gait disturbance, and usually urinary incontinence. Pick's disease is very rare and presents itself with changes in personality, deterioration in social skills, emotional flatness, and significant language problems. Vascular dementia usually has a more sudden onset, and there is evidence of cerebrovascular disease with focal neurological signs and symptoms and usually a history of a previous stroke. Pseudo dementia is the term used to describe false dementia or major depression in the elderly; it usually consists of depressed mood, crying episodes, hopelessness, helplessness, and fatigue. Patients may also present with insomnia or hypersomnia.

Case Ico-delete Highlights A 45-year-old woman presents due to intermittent burning pain for 6 weeks in her left wrist. She has a recent history of a left wrist Colles fracture 4 months ago which was successfully treated with cast immobilization. Following, the pain is severe with minimal provocation, and her pain interferes with joint activity. The patient states that the wrist joint feels "foreign", and that she must both look at her wrist and think about moving her wrist in order to move the joint. She denies any other trauma, fever, chills, other joint pains, bowel or urinary changes, numbness, tingling, neck pain or rashes. On physical exam, the left wrist is warm and locally tender to palpation, with hypoesthesia and hyperhidrosis noted. Cervical ranges of motion and upper extremity reflexes were normal; Spurling maneuver was negative. Question What is a correct expectation regarding the diagnostic work up of this patient?

Correct answer: Electromyography and nerve conduction studies aid in establishing etiologies Explanation This patient's most likely diagnosis is complex regional pain syndrome (CRPS) due to a fracture. Electromyography (EMG) and nerve conduction studies are helpful in determining the neuroanatomy behind the symptoms and therefore in identifying the etiological process. For example, they confirm the presence of nerve injury, can differentiate between CRPS types I and II, nerve root avulsion, or a painful neuropathy due to diabetes, which may present with autonomic dysfunction. No specific diagnostic tests confirm the presence of CRPS. However, the differential diagnosis includes other neuropathic conditions, as well as a host of metabolic, systemic, vascular, and rheumatological disorders. Excluding other treatable but serious causes is indicated even in cases that present with the classic signs and symptoms. This patient's manifestations do not suggest a herniated cervical pulposus or a vitamin B12 or folate deficiency. Blood work for inflammatory arthropathy and vasculitis is indicated, which, in its basic form, includes complete blood count (CBC), erythrocyte sedimentation rate, C-reactive protein, antinuclear antibody, rheumatoid factor, complement fixation panel, serum immunoelectrophoresis, and a bone scan. Workup for diabetes should also include a test for hemoglobin A1c. Compared with radiography, 3-phase bone scintigraphy has higher sensitivity (97% vs 73%) and specificity (86% vs 57%) in early postfracture CRPS.

Case A 62-year-old man presents with vision problems and difficulty swallowing. Over the last week, he has had a constellation of symptoms; they began with numbness and tingling in his feet and progressed to weakness that now affects both lower and upper extremities. Within the last day, he has started to notice difficulty swallowing and double vision. He also feels it is difficult for him to take a big breath. His past medical history is noncontributory, and he takes no medications. Exam reveals bilateral absence of patellar and ulnar reflexes. A lumbar puncture is performed to confirm the diagnosis. Question What cerebrospinal fluid (CSF) finding is most likely?

Correct answer: Elevated CSF protein content Explanation The correct response is elevated CSF protein content. Symmetrical ascending paralysis as described in this patient is indicative of Guillain-Barré syndrome (GBS). The cause of GBS is unknown, but it is generally thought to be an inflammatory autoimmune process. More than half of patients with GBS report an antecedent illness. The antibodies produced in response to antigens present in the infectious agent are thought to cross-react with components of human neurons, prompting an acute postinfectious demyelinating process. Lumbar puncture characteristically reveals elevated CSF protein content. Other results are normal, although the white blood cell count may be somewhat elevated. Decreased CSF glucose and increased polymorphonuclear cell counts are seen in acute bacterial meningitis. Decreased CSF glucose and elevated CSF lymphocyte counts are commonly seen with meningitis caused by fungi. Viral meningitides usually presents with elevated lymphocyte counts, normal CSF glucose, and normal or slightly elevated CSF protein levels.

Case A 3-year-old girl presents with terrible head pain, increasing fever, and irritability. On examination, you find the child febrile, 103°F, and she cries when you lift her head from the examining table. The neck appears to be quite stiff when manipulated. You perform a lumbar puncture. Question What findings would you expect if this child had bacterial meningitis?

Correct answer: Elevated protein Explanation The correct response is elevated protein. Bacterial meningitis is usually an acute infection of the subarachnoid space and meninges caused by a variety of organisms. In the neonatal period, infection with group B. streptococcus, E. coli, Listeria monocytogenes, and other gram-negative enteric bacteria are the most likely. In children, the most important pathogens are Haemophilus influenzae (most common), Neisseria meningitidis (meningococcus), and Streptococcus pneumoniae (pneumococcus). In adults, pneumococcus and meningococcus are the most important causes of meningitis. Older individuals are at increased risk for infection with Listeria and gram-negative enteric bacteria. The classic triad of symptoms of meningitis includes fever, headache, and stiff neck. Respiratory distress, increased response to auditory stimulus, and increased Babinski reflex are not characteristic of presenting symptoms of this disease. Pallor is not characteristic, either. Once the diagnosis of meningitis is suspected, immediate examination of the spinal fluid is indicated with lumbar puncture. Spinal fluid abnormalities usually include elevated protein, low glucose (hypoglycorrhachia), elevated opening pressure, and elevated white cells in the 100-10,000 cells/mm3 range with a predominance of polymorphonuclear cells. The Gram stain is usually positive in more than 90% of patients.

Case A 37-year-old man fell from a ladder as he finished hanging the Christmas lights on his house. The right side of his head hit the alley cement, and he lost consciousness for about 1 minute; he woke up with a headache, but he had no other complaints. A few hours later, the patient is brought to the emergency room by his neighbor because of an intense headache, confusion, and left hand hemiparesis. On examination, the patient has a bruise located over the right temporal region, mydriasis, and right deviation of the right eye, papilledema, and left extensor plantar response. An emergency CT scan of the head without contrast reveals a lens-shaped hyper-density under the right temporal bone with mass effect and edema. Question What is the most likely diagnosis?

Correct answer: Epidural hematoma Explanation Epidural hematoma most often results from a traumatic tear of the middle meningeal artery. Although a lucid interval ranging from minutes to hours followed by altered mental status and focal deficits is typical for epidural hematoma, this clinical picture is only encountered in up to 1/3 of the patients. The collection of blood between the skull and dura mater causes an evident mass effect with ophthalmic nerve palsy and the contralateral hemiparesis. Surgical evacuation of the clot via burr holes is the treatment of choice. Subdural hematoma results from a traumatic rupture of the bridging veins that connect the cerebrum to the venous sinuses within the dura. This venous hemorrhage will result in a gradual increase of the hematoma, with a progressive clinical picture over days or weeks. The CT scan will show a concave, crescent-shaped hyper-density compared to the convex, lens-shaped hyper-density in epidural hematoma. Subarachnoid hemorrhage is the result of an aneurysm rupture; the most common is the congenital berry aneurysm. The clinical picture is of a sudden, severe headache with meningeal irritation. A CT scan will show blood in the subarachnoid space, and a lumbar puncture will reveal xanthochromia CSF. Intracerebral parenchymal hemorrhage is most likely caused by hypertension complicated with Charcot-Bouchard aneurysms. The blood accumulates into the brain substance and most commonly involves the basal ganglia.

Case A 12-month-old girl presents with her parents after a 3-day history of intermittent episodes of strange behavior. A neurologist is consulted because the parents are concerned she is having some form of seizure activity. Her health history includes 2 episodes of otitis media, but she is otherwise healthy. Her initial vital signs and physical exam by the emergency room staff are all normal. Question What description of the infant's strange behavior would lead the neurologist to suspect a diagnosis of simple partial seizures?

Correct answer: Eye deviation with facial twitching that lasts 1-2 minutes Explanation 'Eye deviation with facial twitching that lasts 1-2 minutes' is the correct answer. Simple partial seizures can vary depending on what portion of the cortex has been effected. Patients may display isolated tonic or clonic activity or may have transient altered sensory perception. Clonic movements are jerks that may involve a specific body part or may spread to adjacent body parts, known as "Jacksonian march." Tonic activity is stiffening or sustained posturing, such as eye deviation in this case. Sensory seizures can involve illusions or hallucinations, but this would be difficult to diagnose in a young child. One important hallmark of a simple partial seizure is that the patient does NOT lose consciousness. 'Facial and right arm twitching lasting about a minute then loss of consciousness' is not the correct answer. While a patient having a simple partial seizure might experience facial and right arm twitching for that period of time, they are not going to have loss of consciousness. The type of seizure involved in this answer choice best describes a complex partial seizure. 'Blank stare and impaired awareness for 10-20 seconds' is not the correct answer. This description is best used to describe a patient who is having a nonconvulsive generalized seizure, otherwise known as an absence seizure. Patients having an absence seizure will rarely have any movement during the seizure. At most, they may have some minor motor activity, such as blinking. In an older child or adult, these episodes will leave the patient feeling as though they have short periods of time that they do not remember. 'Loss of consciousness followed by stiffening of the body and extremities' is not the correct answer. The fact that this description includes loss of consciousness is what excludes it from being a description of a patient having a simple partial seizure. This scenario better describes a person who is having a generalized tonic-clonic seizure. 'Very brief periods of whole body going limp followed by brief loss of consciousness' is not the correct answer. The fact that this description includes loss of consciousness is what excludes it from being a description of a patient having a simple partial seizure. This scenario better describes a person who is having an atonic seizure, which is characterized as an abrupt, brief loss of body tone, possibly followed by brief loss of consciousness.

Question Ico-delete Highlights A 72-year-old man exhibits progressive tremors, bradykinesia, and muscular rigidity. He is slow to initiate movements and shows other motor abnormalities. The most likely diagnosis of his disease is Parkinson disease. What drug mimics the effects of this disease and should be avoided in patients with this disease?

Correct answer: Haloperidol Explanation This patient suffers from Parkinson disease. In individuals over 65 years old, the incidence of the disease is approximately 1:100. Although the root cause of Parkinsonism is unknown, most patients show a reduced function in the dopaminergic neurons in the substantia nigra and corpus striatum, which are parts of the basal ganglia that are important in the control of motor function. The substantia nigra contains dopaminergic neurons that project to the corpus striatum where they make numerous synaptic contacts with many cells and exert an inhibitory function via release of dopamine. Since these neurons fire tonically, they appear to exert a constant influence on motor functions. The neurons of the corpus striatum project to the substantia nigra. The neurotransmitter for these neurons is GABA. Furthermore, axons from the cerebral hemispheres and thalamus terminate in the corpus striatum where they secrete the excitatory neurotransmitter glutamate, which results in voluntary initiation of contraction of skeletal muscle and intentional movements of the body. Striatal interneurons secrete acetylcholine. In Parkinson disease, neurons in the substantia nigra atrophy; consequently, the levels of dopamine decline. Levodopa (L-dopa) (but not dopa itself) crosses the blood-brain barrier rapidly and can be converted directly into dopamine. Increases in dopamine levels ameliorate the effects of death of dopaminergic neurons in the substantia nigra. Large doses of L-dopa must be administered because it is rapidly decarboxylated in peripheral tissues, leading to increases in peripheral dopamine with unwanted side effects (arrhythmias, hypotension, nausea and vomiting). Co-administration of a dopamine decarboxylase inhibitor such as carbidopa, a drug which does not cross the blood-brain barrier, decreases peripheral production of dopamine from levodopa, thus decreasing unwanted peripheral side effects; this allows a 4-fold reduction in the dose of levodopa required to obtain relief of parkinsonian symptoms. Haloperidol is the correct answer. It blocks the dopamine receptors in the brain, releasing inhibitory neurons in the corpus striatum. Thus, one of the major side effects of this and other neuroleptics is secondary Parkinsonism. These drugs are contraindicated in parkinsonian patients.

Case A 10-year-old boy is referred to you for being fidgety at school, though he makes good grades. Prior history is unremarkable, and there has been no recent illness. The boy's father had a similar history as a child. While speaking with the patient, you notice that he clears his throat several times per minute. Examination is otherwise normal, except for rapid, nonrhythmic jerking movements of the face, neck, and shoulders while at rest. Question What regarding this patient's condition is true?

Correct answer: He has a condition that clusters in families Explanation The combination of vocal tics (in this case, throat clearing) and motor tics should suggest Gilles de la Tourette (Tourette) syndrome, which is a common tic disorder that clusters in families. Once thought to be a single gene, autosomal dominant condition, it appears that multiple genes and nongenetic factors may be at play. Coprolalia is but one of many vocal tics seen in Tourette syndrome (throat clearing, humming, whistling); although a notorious one, it is not a necessary feature. The positive family history makes it unlikely that the condition described is factitious. Pimozide, SSRIs, and other agents have been used in the treatment of Tourette syndrome, but levodopa is not used. The movement disorder of Sydenham chorea, as seen after a streptococcal infection, is choreiform; that is, it tends to be slower and rhythmic, as opposed to the rapid, nonrhythmic jerks seen in Tourette syndrome. Also, vocal tics are not typically seen with Sydenham chorea

Case A 48-year-old Caucasian woman with a past medical history of hypertension and hypercholesterolemia was diagnosed recently with a cerebral aneurysm. The treatment plan for the aneurysm is endovascular coiling. Among other complications, this patient has an increased risk of thromboembolism postoperatively. Question What is the initial preventive medication that will be used to help minimize this risk in this patient?

Correct answer: Heparin Explanation Complications of endovascular coiling can include thromboembolism and intra-procedural aneurysm rupture. A study found the incidence of thromboembolism in up to 12.5 % of patients in their patient population group. For this reason, many providers will choose to administer some type of combination of heparin and/or antiplatelet therapy to help minimize this potential complication. Coumadin is not indicated as an appropriate choice for this acute procedure. The medications abciximab, eptifibatide, and tirofiban are all anti-platelet/glycoprotein IIb/IIIa inhibitors. These all have been shown to be safe, but they are currently only recommended for use in percutaneous coronary intervention and the prevention of complications in patients with unstable angina/non-ST-elevation myocardial infarction.

Case A 22-year-old engineering student presents after a major motor seizure. The patient was well until 3 days prior to presentation, when he developed generalized headache and a fever of 101.2 F (by mouth), for which he took aspirin. He later complained that he could not concentrate on his studies, and he added that all his teachers wanted to flunk him anyway. On more than 1 occasion, he accused his roommate of stealing his food from their shared refrigerator. He complained that he could not smell his food. Several times, he was noted to be having staring spells and lip smacking; during these episodes, he did not respond to his roommate. The spells lasted less than 1 minute. On the day of presentation, the roommate returned to the dorm to find the patient standing naked in the hallway outside their room; he was screaming "We're going to settle this now!" He then fell to the floor and sustained a major motor seizure. The patient's mother denies a history of major illness, but reveals he had measles and chicken pox as a child. The roommate states that the patient drinks a lot of beer at weekend parties and smokes marijuana occasionally, but he denies knowledge of other drug use. There is no history of seizures. Exam shows a well-developed, lean, muscular young man. He is lying on a gurney and fighting his restraints. He is alert but mute, and he is unresponsive to the spoken word. Temperature is 101.2 F (taken rectally). Heart rate is 110 and regular; respirations are 22 and unlabored. Blood pressure is 140/85 mm Hg. Neurological exam is limited by patient agitation and lack of cooperation, but it reveals normal pupils and eye movements, full symmetric facial movements, a mild right hemiparesis, and right extensor plantar response. Labs include complete blood count with a white blood cell count of 8,500; there is a normal differential. The chemistry profile, chest X-ray, and electrocardiogram are normal. Stat computed tomography of the head with contrast reveals left inferomedial temporal and frontal confluent areas of hypointensity with surrounding edema as well as scattered areas of hemorrhage in the same regions. These areas enhance densely. Cerebrospinal fluid exam reveals moderately elevated pressure, mildly elevated protein, and 267 white blood cells (80% lymphs, 20% polymorphs). Glucose is normal. Question What is the most likely cause of this patient's illness?

Correct answer: Herpes simplex encephalitis Explanation This is a rather classic presentation for focal encephalitic form of herpes simplex encephalitis. A prodrome of fever and headache is common. Anosmia, apparent temporal lobe seizures, and personality changes point to the inferomedial temporal and frontal regions. A convulsion is frequently the event that precipitates admission. The computed tomogram is classic. The cerebrospinal fluid is typical, but it can be normal early in the clinical course. This is the most common and serious of the acute encephalitides. This is a very grave illness, and it must be recognized and treated with antiviral agents (acyclovir) as soon as possible. Acute hemorrhagic leukoencephalitis is seen mostly in children, but it can also be seen in young adults. However, it is almost always preceded by symptoms of respiratory infection of 1 - 14 days duration. These patients may also develop headache and confusion, but neck pain and signs of meningeal irritation are more common and pronounced. Anosmia and temporal lobe seizures would be less common or pronounced. In this condition, white blood cell counts as high as 20,000 - 30.000 can be an important point of differential. Cerebrospinal fluid may be identical, but pleocytosis of up to 3,000 can help to differentiate it from herpes simplex encephalitis. The most helpful information at this point in differentiating this condition is the computed tomogram and magnetic resonance imaging scan, which would be expected to show diffuse asymmetric, large, and confluent edematous lesions in the white matter, with multiple small hemorrhages in both white and gray matter. Subdural empyema is a suppurative condition between the inner surface of the dura and the outer surface of the arachnoid. Since it occurs in a preformed space, it is properly termed an empyema, although it is frequently and erroneously called subdural abscess. It most commonly follows a worsening of chronic sinusitis, in which cases it most commonly involves adolescent and young men. There is usually a history of worsening of brow or mastoid pain and nose or outer ear canal purulent discharge. Fever, malaise, and headache appear. The headache pain worsens, and then becomes generalized; vomiting is frequent. Focal neurological signs may appear, and stupor and coma may ensue. Fever, leukocytosis, and stiff neck are almost universal. Cerebrospinal fluid is rather similar to that seen in herpes simplex encephalopathy. However, computed tomography scan of the head shows ear or sinus lesions and possible bone erosion. Enhanced meninges around the pus may be seen. Magnetic resonance imaging is more likely to visualize the empyema. About 40% of cerebral abscesses are related to diseases of the paranasal sinuses, middle ear, and mastoid air cells. Other conditions include bronchiectasis, lung abscess, acute bacterial endocarditis, congenital heart disease, pulmonary arterial venous malformation (AVM), abscessed teeth, osteomyelitis, or other sites of bacterial infection. The clinical course may be as seen in this patient, but onset and progression are much more variable depending on where the abscess is and the stage of development it has reached. The main concern here would be in differentiating herpes simplex encephalopathy from temporal or inferior frontal abscess. Spinal fluid is usually quite similar, and while finding bacteria might resolve the issue, this is rarely the case. Here, contrast studies are most helpful. Computed tomography would show a central hypodensity, a surrounding ring of enhancement, and hypodensity beyond the enhancing ring. MRI T1-weighted images show capsular enhancement with central hypodensity. T2-weighted images show capsular hypodensity, and surrounding edema is obvious. A septic embolism usually produces sudden neurological deficit, with the clinical picture depending on the site of ischemia. It might be mild and brief, its history being obtained only later when the patient presents with a clinical picture of expanding intracranial lesion (abscess), or it may present as a major acute typical appearing ischemic cerebrovascular accident. If it formed a temporal or frontal abscess, magnetic resonance imaging and computed tomography would probably be definitive. They may also form mycotic aneurysms of the more distal cerebral arteries; in turn, they may rupture, producing a picture of acute subarachnoid hemorrhage with or without intracerebral hemorrhage. Elevated sedimentation rate, leukocytosis, fever, malaise, fatigue, and known heart disease should prompt a thorough search for a septic embolic source. Ruptured saccular aneurysms are not usually a problem. There should be no prodrome (preceding headache, fever, stiff neck, malaise, abnormal behavior, etc.). The patient would be expected to have sudden severe headache, nausea, vomiting, and possibly loss of consciousness. There would be no fever or leukocytosis (except for a possible stress pattern). Focal neurological deficits are the exception, at least initially. Optic disc hemorrhages may be seen. Computed tomography generally shows blood (90%) in the subarachnoid spaces, which can be dramatic, or show only a subtle shadow along the tentorium or in the Sylvian or adjacent fissures. Rarely, the bleed may extend into brain tissue, producing an intracerebral hematoma. In the 5 - 10% of cases where no blood is seen on computed tomography or magnetic resonance imaging, a lumbar puncture generally reveals red blood cells in the thousands to a million, frequently with xanthochromia after several hours.

Case A 40-year-old woman presents with a 7-day history of pain in her right arm. The patient denies any trauma or injury to this extremity just prior to the pain starting, but she does admit to having a Colles' fracture in this arm about 2 months ago. She denies any injury to her back, neck, or other components of the musculoskeletal system prior to the event of pain. She describes the pain burning and throbbing with an extremely diffuse, uncomfortable ache accompanying it. She further states that this limb has become extremely sensitive to touch and cold; she adds that it appears somewhat more swollen than her left arm. The patient is very upset because she does not know why her arm is so painful when she has not done anything to it. She is a nonsmoker. She does not drink alcohol, and she exercises 3 times a week. Physical examination of the extremity reveals a slightly cyanotic hue and generalized pain of the entire right upper extremity. Pulses are strong (2+), but due to the pain, the patient states ROM is limited. Question Considering the most likely diagnosis, what is the initial pharmaceutical regimen that should be initiated?

Correct answer: Naprosyn 500 mg 2 tablets daily Explanation The correct response is Naprosyn 500 mg 2 tablets BID. The patient being described is most likely in stage I of complex regional pain syndrome (CRPS). This disorder is rare, causing instability of the vasomotor and autonomic components. CRPS is a disorder of a body region, most commonly the extremities, that is characterized by pain, swelling, limited range of motion, vasomotor instability, skin changes, and even patchy bone demineralization. Frequently symptoms will begin after the patient has sustained an injury, had surgery, or experienced a vascular event, such as a stroke. 35% of patients with CRPS report not having a distinct precipitating event. Stage 1 of CRPS characteristically has findings such as pain in the limb (burning/throbbing), diffuse, uncomfortable aching, sensitivity to touch or cold, or even localizededema. The distribution of pain is not compatible with a single peripheral nerve, trunk, or root lesion. There will also be signs of variable vasomotor disturbances that result in altered color and temperature. At this stage if a radiograph was completed, most likely it will be normal but could show signs of patchy demineralization. Early treatment gives the best prognosis for this patient. The accepted first line treatment would be NSAIDs (Naprosyn 250 - 500 mg 2 times daily). More severe cases that have significant edema (our patient above states there is swelling, but pulses are strong on PE) will be started on daily prednisone, 30 - 60 mg/day for a 4 week taper. Other options that may be considered for treatment include (but are not limited to) gabapentin, topical capsaicin, topical dimethylsulfoxide, IV bisphosphonates, IV immunoglobulin, and intrathecal baclofen. Acetaminophen, acyclovir, colchicine, and cephalexin are not indicated for treatment of CRPS.

Case Ico-delete Highlights A 32-year-old man is admitted to the hospital following loss of consciousness. The patient had been ill with fever and headache for several days, then developed double vision, confusion, and loss of consciousness. He has been otherwise healthy with no past medical conditions. Imaging reveals edema of the frontal lobe. Question Which virus most commonly causes the patient's infection?

Correct answer: Herpes simplex virus Explanation Herpes simplex virus is correct. The patient is suffering from encephalitis, which is typically viral in nature. Herpes simplex virus is the most common viral cause of encephalitis. Adenovirus is a rare cause of encephalitis. Adenovirus more commonly causes respiratory and eye infections. Encephalitis is a rare complication of these conditions. Influenza A is a rare cause of encephalitis. Influenza A typically causes respiratory infection, which is infrequently complicated by encephalitis. West Nile Virus is a rare cause of encephalitis. While about half of all patients who are diagnosed with West Nile Virus have meningitis and encephalitis, the overall rate of West Nile Virus is low. Epstein-Barr Virus is incorrect. Epstein-Barr virus causes infectious mononucleosis. It is estimated that only 1-5% of cases of infectious mononucleosis have nervous system involvement.

Case A 15-year-old girl presents with loss of consciousness. She is accompanied by her mother, who states that the patient initially fell ill several days ago with a headache, muscle aches, and fever. The patient developed a severe headache today, accompanied by double vision, difficulty speaking, confusion, and eventual loss of consciousness. She has not taken any medications aside from acetaminophen (Tylenol) for her fever. Her mother states that her daughter is usually active and had been playing soccer regularly until she became ill; the patient has been fairly healthy aside from occasional cold sores. Past medical history is significant for frequent ear infections as a toddler that were treated with tympanostomy tube placement at age 2. Brain imaging reveals edema of the temporal lobe. Question What component of the patient's history is most likely contributing to her current illness?

Correct answer: History of cold sores Explanation The patient is exhibiting neurological symptoms and imaging results consistent with encephalitis. Encephalitis is typically viral in nature, with the herpes simplex virus being the most common cause. Cold sores are associated with herpes simplex virus. Recent acetaminophen (Tylenol) use would not contribute to neurological symptoms unless the patient had taken an overdose of the medication. Given her symptoms of fever and myalgia, it is unlikely that acetaminophen caused the loss of consciousness. Playing soccer regularly might place the patient at risk for a concussion, but her history is not consistent with concussion. History of frequent ear infections as a child and history of tympanostomy tube placement would not place the patient at risk for encephalitis as a teenager.

Case A 45-year-old man presents with concerns of uncontrollable movements that he has noticed for the past 2 months. He feels he cannot control these involuntary movements of his upper body. His wife reports that he appears irritable and impulsive. She feels that his personality has changed but is really more concerned about the sudden jerking in his body. She shared that her father-in-law had passed away in his 50s with similar symptoms. Upon physical exam, the patient appears to have tics that are sudden and appear depressed. Additionally, Hoffmann's sign and Babinski's sign are normal. A CT scan shows cerebral atrophy and genetic testing from the lab is pending. Question What is the most likely diagnosis?

Correct answer: Huntington Disease Explanation The clinical picture is suggestive of Huntington's Disease due to the description of jerky involuntary movements (chorea). Additionally, the description of depression, irritability, and family history helps confirm the diagnosis. Amyotrophic lateral sclerosis is not the correct answer because the patient does not report weight loss, slurring of speech, or gait disorder. Additionally, Hoffmann's sign and Babinski's sign were normal. Guillain-Barré Syndrome is not correct because symptoms of weakness present with a progressive and rapid onset that can reach its worst within 1 month are not described. In the stem of the problem, the patient describes onset over the past 2 months and not as rapidly as Guillain-Barré would present. Multiple sclerosis is not correct, as the patient does not describe vision loss, facial palsy, bowel or bladder incontinence, or vertigo. Additionally, CT scan did not demonstrate the presence of oligoclonal bands. Myasthenia gravis is incorrect because myasthenia gravis is associated with muscle weakness, most commonly starting with the eye; furthermore, myasthenia gravis does not present with chorea or have a family history. Etiology is commonly due to reduction in the function of acetylcholine receptors.

Case You are seeing a 38-year-old man with uncontrolled facial movements. He states that in the last few months, he has noticed himself making expressions without even realizing it or being able to control it. Further questioning reveals that he also has noted an inability to intentionally move his eyes quickly without blinking. Very recently he noted an inability to sustain physical movements, such as grasping objects with his hands. Physical examination reveals a puppet-like gait and obvious chorea. The patient admits that he does not know anything about his family history due to the fact that he was adopted when he was 4 years old. Question Based on the history and physical examination findings, what is the likely diagnosis in this patient?

Correct answer: Huntington disease Explanation The patient above is most likely showing signs and symptoms of Huntington disease. This is an inherited, autosomal dominant genetic disorder that occurs in all ethnic groups. Clinical onset is usually between the ages of 30-50 years with the disease process itself being progressive; it is fatal within a time period of 15-20 years. Symptoms may consist of movement and intellectual changes. Early mental signs and symptoms include irritability, moodiness, antisocial behavior, and psychiatric disturbances, as well as dementia. Movement signs and symptoms may initially be seen as fidgeting or restlessness, but eventually chorea movement and dystonic posturing occur, as well as progressive rigidity and akinesia. Benign essential (familiar) tremor is not the correct answer.This tremor may begin at any age and can be enhanced by emotional stress. Usually one or both hands are involved, and possibly the head or even the voice. This is specifically a postural tremor that may notably improve somewhat with alcohol consumption. The tremor itself is the only symptom. Multiple sclerosis (MS) is also an incorrect choice. Symptoms found in a MS patient during the initial diagnosis may commonly include a mixture of evidence: complaints of weakness, numbness, tingling, or unsteadiness in a limb; spastic paraparesis; retrobulbar optic neuritis; diplopia; disequilibrium; or even sphincter disturbance. These are not consistent with the patient description. Myasthenia gravis (MG) is also the incorrect choice. Patients with MG typically present with ptosis, diplopia, difficulty chewing or swallowing, respiratory difficulty, limb weakness, or any combination of the above. Muscle weakness of a specific or even eventually general component of the body is the key feature of MG; it follows a slowly progressive course, and potentially may be fatal if the respiratory dysfunction is not corrected. This is not consistent with the patient described in the above scenario. Parkinson's disease, and specifically idiopathic Parkinson disease, is a somewhat common disorder that also occurs in all ethnic groups, shows equal sex distribution, and begins symptomatically between the ages of 45-65 years old. Cardinal motor features of Parkinson's disease are a tremor (this is most conspicuous at rest and enhanced by emotional stress, as well as being less severe during voluntary activity) and an increase in resistance to passive movement. Rigidity is also found in these patients, and this is responsible for the flexed posture seen in many individuals with this disease state. Only a mild intellectual decline may be seen. This description is not consistent with the patient above.

Case A 25-year-old woman is seeking medical attention because she is concerned about a genetic disease that is running in her family. Both her father and her paternal grandfather are affected. Her 50-year-old father has suffered for several years with coordination difficulties, chorea, decline in cognitive ability, and aggressive outbursts. His symptoms are getting progressively worse. Question This patient has a 50% chance of developing what disorder?

Correct answer: Huntington's disease Explanation The correct answer is Huntington's disease. Huntington's disease is a progressive disorder that affects motor skills and cognition. It is caused by a CAG trinucleotide repeat expansion in the HD gene, located on chromosome 4p16. It is inherited in an autosomal dominant manner. The majority of affected individuals present with neurological manifestations; symptoms include coordination problems and involuntary movements. Oculomotor disturbances, dysarthria, and hyperreflexia occur early in the disease. It is characterized by a progressive deterioration of both voluntary and involuntary muscle control, cognitive decline, and psychiatric disturbances. The average age of onset is between 35 and 44 years. Median survival time is 15 to 18 years. Eventually, the patient becomes severely disabled due to a complete loss of motor control, muteness, and incontinence. Molecular genetic diagnosis is available. Creutzfeldt-Jakob disease is a subacute spongiform encephalopathy caused by a prion, which is a proteinaceous infectious particle. These infectious agents are thought to act by transforming normal protein molecules into deadly ones. This disorder can be inherited, transmitted through infection, or occur sporadically. Inherited cases (15% of CJD cases) show a mutation in the gene that codes for the normal prion protein. The disease is characterized by rapid and progressive dementia and neuromuscular symptoms. Initial symptoms include insomnia, personality or behavioral changes, problems with memory, incoordination, and vision disturbances. This is followed by dementia and myoclonus. Eventually, the patient becomes bedridden and enters a coma. Duration is usually 1 year or less. Average age of onset is between 50 and 75 years of age. Presently, there is no treatment. Early-onset familial Alzheimer disease is an adult onset progressive dementia. It is associated with cerebral cortical atrophy, β-amyloid plaques and neurofibrillary tangles. As the name implies, patients with this disorder have a positive family history. Mutations have been found in 2 genes: the amyloid precursor protein gene (APP), presenilin 1 (PSEN1), and presenilin 2 (PSEN2). Inheritance occurs in an autosomal dominant manner. Mean age of onset is less than 65 years. Initial symptoms are vague and include memory failure, confusion, language disturbance, agitation, and withdrawal. Progression of the disorder is slow, taking place over the course of years (mean range is 8-10 years). Alzheimer disease is the most common cause of dementia in North America and Europe. Eventually, the disease worsens, and death occurs due to malnutrition and pneumonia. Diagnosis is based upon the progressive dementia with the presence of β-amyloid plaques and neurofibrillary tangles. Molecular genetic testing is available for all 3 genes involved. Hallervorden-Spatz disease is a rare neurodegenerative disorder inherited in an autosomal recessive manner. It is characterized by a progressive rigidity that starts in the lower extremities and moves to the upper extremities. Rigidity may be accompanied by involuntary movements. Muscle tone and voluntary movement deteriorate progressively, leading to feeding difficulties and emaciation. During the late stages of the disorder, mental deterioration and progressive dementia are present. Average age of onset is the 1st or 2nd decade; mean survival time is approximately 10 years. Although the responsible gene has not been identified, it has been mapped to chromosome 20p13-p12.3. Diagnosis has generally been made postmortem. Lesch-Nyhan syndrome is a neurological disorder caused by overproduction of uric acid. It is caused by mutations in the hypoxanthine-guanine phosphoribosyl transferase (HPRT) gene located on chromosome Xq26.1. It is inherited in an X-linked recessive manner. Only men are affected (with exception). The syndrome is characterized by neurologic dysfunction, cognitive disturbances, behavioral disturbances, and the overproduction of uric acid. Initially, affected individuals show developmental delay and hypotonia, usually within the 1st year of life. Most never walk. Over the next few years, spasticity and hyperreflexia emerge, as does the hallmark sign of self-injurious behavior. This includes biting of the hands, lips, and cheeks, as well as head or limb banging. Survival into the 2nd or 3rd decade is common. The disorder can be diagnosed by molecular genetic testing.

Case Ico-delete Highlights A 40-year-old man presents with clumsiness; he frequently drops things. Over a period of time, he has developed strange behaviors and has become quite withdrawn. He has also noticed a decrease in his mental capacity. He has developed an accompanying movement disorder; the movements can be involuntary, coarse, and irregular. The abnormal movements are exhibited by his limbs in particular. His father had the same condition. An MRI is done and shows cortical atrophy and striatal atrophy. Question What is the most likely diagnosis?

Correct answer: Huntington's disease Explanation This patient has Huntington's disease (also called Huntington's chorea). A decline in mental functioning, dementia, emotional changes, and chorea are all symptoms of Huntington's disease. Chorea refers to involuntary, coarse, irregular movements. Huntington's disease is inherited in an autosomal dominant manner. An MRI showing striatal atrophy (particularly atrophy of the caudate and putamen) and cerebral atrophy is a finding seen with Huntington's disease. On chromosome 4, there is a trinucleotide sequence that is extensively repeated; it is thought to be the underlying cause of the disease. The symptoms of Gilles de la Tourette syndrome are tics and vocalizations. The symptoms of Parkinson's disease include tremor and rigidity, but not chorea. The symptoms of progressive supranuclear palsy include dystonia and eye abnormalities, but not chorea. A symptom of Sydenham chorea is chorea, but it is seen in children after rheumatic fever (or occasionally in pregnant women).

Question A 4-month-old infant presents for a well child check up. She was a spontaneous vaginal delivery at 39.5 weeks without complications. The mother notes the baby has been more irritable and has not been eating well. No cough or fever is described. There are no smokers at their home. On exam, you note impaired extraocular movements, especially in the upward gaze, and a bulging anterior fontanel. There is increased tone of the legs. Skin exam is normal. Similar to her last visit, the length and weight are at the 50th percentile. The head circumference was formerly at the 75th percentile and is now at the +99th percentile. What clinical diagnosis do you make for this child?

Correct answer: Hydrocephalus Explanation The clinical diagnosis of hydrocephalus can be made by macrocephaly, impaired upward gaze, and hypertonia of the extremities. These signs would warrant further workup for definitive diagnosis. Craniosynostosis results when there is premature closure of the cranial sutures, usually resulting in elongation of the head. Positional plagiocephaly is occipital flattening, most often from infants lying on their back to sleep. The skin exam is normal in this baby, while neurofibromatosis will often display café au lait spots. Catch-up growth can be seen in babies born prematurely, which is not the case with this patient. Also, only the head percentile is changing, not the length and weight.

Case Ico-delete Highlights A 36-year-old woman presents with fatigue and diplopia; symptoms started in the morning and worsened during her time at work. On examination, the patient has ptosis bilaterally and decreased ocular muscle power. A CT scan of the chest shows a thymoma. Question What is the pathophysiology of this patient's neuromuscular disorder?

Correct answer: Immune-mediated destruction of the acetylcholine receptor Explanation Myasthenia gravis is a neuromuscular disorder caused by immune-mediated loss of acetylcholine receptors. It is associated with thymic hyperplasia in 65% of cases and with thymoma in 15% of cases. Myasthenia gravis (MG) is an autoimmune disorder affecting the neurochemical transmission of information between motor nerve fibers and skeletal muscle fibers. In this disorder, nerve impulses are transmitted in a normal fashion to the axon terminals, and skeletal muscle is capable of contracting normally. The dysfunction occurs at the interface between the 2 components of the neuromuscular junction (NMJ). In cases of MG, an autoimmune response causes antibodies of the IgG (not IgA) class to be produced against the nicotinic receptors of the NMJ. The antibodies fix onto the nicotinic receptors, block acetylcholine (ACh) from complexing with the receptors, and ultimately bring about destruction of the receptors. The patient may present with ophthalmic or non-ophthalmic manifestations. The ocular disturbances mainly include ptosis and diplopia; the non-ophthalmic symptoms include difficulty in talking, swallowing, or chewing and upper limb and trunk weakness. Anti-acetylcholine receptor antibodies assay is positive in 90% of patients with generalized MG, whereas it is positive in 50-70% in ocular MG. Other tests include the Tensilon test, imaging studies such as CT or MRI scanning, repetitive nerve stimulation, and single fiber electromyography. Medical treatment includes neostigmine therapy, plasmapheresis, or intravenous immunoglobulin therapy. Thymectomy may be considered in cases of thymoma. The reduction in receptor population decreases neurotransmission, and the result is that muscle depolarization is not produced. The quantity and quality of cholinesterase enzymes is unaffected by the disorder, and so is any alteration in the composition of motor units at the neuromuscular junctions. A genetic disorder associated with deficiency of the structural protein dystrophin in muscles occurs in Duchenne muscular dystrophy, and it is not seen in myasthenia gravis

Case Ico-delete Highlights A 42-year-old female teacher presents with a hand tremor; the tremor has been present for several years, but it is getting more bothersome. The patient would like treatment for her tremor. It appears to be present only in her right hand, and she tells you that it "comes and goes." It is especially embarrassing for her because she writes on the board in front of her students. She denies pain, motor weakness, and abnormal sensation in the right hand. She reports that her father's hand used to "shake" when he drank his coffee. She otherwise feels well, and she denies other symptoms. She has not noticed memory problems, incoordination, or balance problems. Her past medical history is unremarkable. She had 3 uncomplicated pregnancies and deliveries; she has no chronic medical conditions. Except for a mild tremor on the patient's right hand when held extended, the physical exam is unremarkable. Her neurological exam is otherwise normal. Question What is the most appropriate intervention for this patient's condition?

Correct answer: Initiate propranolol. Explanation Based on her history and exam, this patient has a benign essential (familial) tremor, for which it is appropriate to initiate propranolol. Propranolol is a non-selective beta blocker. This patient has no contraindications to the therapy; she is troubled by the worsening tremor, which interferes with her daily activities. Essential tremor is indeed a benign condition. When the tremor is mild and does not cause embarrassment or interfere with the patient's activities, it would be reasonable to avoid medications, but it is inappropriate to tell the patient that no treatments exist. Especially since this patient desires treatment. Carbidopa/levodopa would be an appropriate medication for an individual with tremor due to Parkinson's disease, but this patient has no other neurological complaints and no history to suggest she currently has Parkinson's disease. Methylphenidate is a stimulant typically used for the treatment of attention deficit hyperactivity disorder. Not only would it be ineffective in this patient's case, it would also likely make her tremor worse. Physical therapy has not been proven to have a role in treating essential tremor. It would not be helpful in this case.

Case Ico-delete Highlights A 35-year-old man presents with a 2-week history of constant burning and throbbing pain in his left hand. The pain also seems to affect his distant forearm. Contact with normal clothing and bed sheets worsen his hand pain. He also reports his hand is swollen. He denies trauma to his hand, but he does report an uncomplicated left wrist fracture 3 months ago that resulted from a sports accident. By the time of cast removal at 7 weeks post-injury, the patient denied any pain or edema in the affected limb. He was instructed to return to normal activities, as tolerated. He was initially treated with ibuprofen and opioid pain pills. He discontinued all pain medicines within 2 weeks of the fracture. He now describes his hand pain as an 8 out of 10, on a 1-10 pain scale. On physical exam, the left hand and forearm are noted to have some localized edema, warmer temperature, and increased hair growth, compared to his right hand and forearm. No rash or skin lesions are noted. With even light palpation of the affected region, the patient cries out in pain. Range of motion is decreased, and reflexes are increased in the left upper extremity (in comparison to the right). The rest of his exam is normal. X-ray of the left wrist and hand are normal, with good fracture resolution. Question What actions might have best prevented this patient's complication?

Correct answer: Initiation of vitamin C following fracture Explanation This patient most likely has complex regional pain syndrome (CRPS). CRPS is indeed a complex syndrome often occurring in a limb; patients exhibit vasomotor and neurologic symptoms out of proportion with objective findings. It often occurs following some type of event, such as a surgery, fracture, stroke, or myocardial infarction. When fracture is the inciting event, initiation of vitamin C following the fracture is associated with lower risk of CRPS.The reason is unknown. CRPS is classified into three stages, and symptoms can vary depending on the stage. It is characterized by persistent burning and/or throbbing pain in an extremity. Swelling, redness, and increased hair/nail growth occur in stage I; it may progress to cool, pale skin and even osteoporosis in the latter two stages. Allodynia, the phenomenon in which a normally painless stimulus produces significant pain response, is a common finding. No single exam finding or test is diagnostic of CRPS, which is a clinical diagnosis. This patient's painful erythematous hand could be mistaken for cellulitis, but cellulitis is not typically associated with closed wrist fracture. Initiation of broad-spectrum antibiotics at the time of an uncomplicated fracture is not recommended. One could consider a venous thrombosis as the cause of this patient's symptoms (although unusual to present in the hand and not associated with wrist fractures). Initiation of clopidogrel (Plavix) immediately after treatment of this fracture would not be recommended, and no current data suggest anticoagulants could have prevented this patient's CRPS. In contrast, anticoagulation should be considered following hip fractures, especially in vulnerable geriatric populations. No current data support that more aggressive pain management following fracture reduces the likelihood of CRPS. In fact, this patient reports that his pain resolved appropriately quite soon after the fracture. Prolonged immobilization with a hard cast would have likely contributed to further problems; there would have been more muscle atrophy, and there would have been no reduction of CRPS incidence following the fracture.

Question What condition is characterized by fine movements that are absent at rest, occur with activity, and worsen as the target is neared?

Correct answer: Intention tremor Explanation Intention tremors are fine movements of the hands that are absent at rest, occur with activity, and worsen as the target is neared. They occur with multiple sclerosis and cerebellar disease. Choreiform movements are brief, rapid, jerky, unpredictable motions of the hands. They occur both at rest and during normal actions. They are associated with both Huntington's and Sydenham choreas. Asterixis is non-rhythmic flapping movements of the hands, especially if the wrists are dorsiflexed. It is associated with liver failure, renal failure, and pulmonary insufficiency. Static, or resting, tremors are coarse movements of the hands that are present at rest, disappear with movement, and may involve alternation of the fingers in a "pill-rolling" manner. They are associated with Parkinson's syndrome and extrapyramidal dysfunction. Postural tremors are fine movements of the hands that occur when the hands are held in an active position, usually against gravity. Examples of postural tremors are those resulting from anxiety, fatigue, and hyperthyroidism, as well as familial postural tremors.

Case Ico-delete Highlights A 19-year-old man presents for evaluation of excessive daytime sleepiness. Despite getting a full night's sleep, his coworkers have repeatedly noticed him suddenly dozing off at his desk and during meetings; sometimes, he falls asleep mid-conversation. Upon further questioning, the patient admits that he occasionally experiences nighttime episodes during which he has the sensation of being unable to speak or move. The remainder of the history and physical exam are unremarkable. Aside from the occasional ibuprofen for shoulder pain, he does not take any medications. Question The patient's symptoms are most consistent with what disorder?

Correct answer: Narcolepsy Explanation Based on the symptoms described, the patient is most likely to have narcolepsy. Narcolepsy is marked by 4 major symptoms: excessive daytime sleepiness cataplexy sleep-related hallucinations sleep paralysis It is a central sleep disorder that is attributed to abnormal neurotransmission in the brain's sleep centers. Daytime sleepiness can be so severe that it results in involuntary somnolence during activities such as driving or talking. Cataplexy describes the brief loss of muscle tone that can occur sporadically during waking hours. Patients also may suffer from visual, auditory, or tactile hallucinations at the onset of sleep or upon waking. In addition, they may experience inability to move on falling asleep or emerging from sleep (sleep paralysis). Central sleep apnea is an uncommon neurological condition that involves impairment of a patient's normal spontaneous cue to breathe based on changes in blood oxygen and carbon dioxide levels. The symptoms are similar to those of obstructive sleep apnea. Insomnia is a generic condition marked by difficulty falling asleep or the inability to remain sleeping for extended periods of time. It is a symptom, not a diagnosis. Sleep derangement is a common medication side effect; it is especially observed with psychotropic medications. There is no reason to suspect medication-related sleep impairment in this patient because he reports no medication use. Obstructive sleep apnea is defined as partial or complete upper airway obstruction caused by anatomic collapse of pharyngeal structures. Patency of the upper airway is partially (hypopnea) or completely (apnea) lost during sleep. This results in the development of progressively increasing inspiratory effort against an obstructed glottic opening, leading to the build-up of increasingly negative intrathoracic pressure. At some point, brain stem reflexes result in an altered sleep state that prompts reopening of the airway. This may awaken the patient or be observed as a "gasping" respiratory effort by an onlooker. Snoring is indicative of obstructed airflow, and daytime somnolence is a function of disrupted sleep.

Case A 48-year-old HIV-positive man starts to develop headaches. At first, he attributes the headaches to stress; however, they persist and become worse over the next few weeks. He develops nausea and vomiting, and he thinks he has a fever. He starts to become confused, so he seeks medical attention. On physical examination, his temperature is 100°F. He has signs of meningeal irritation. A lumbar puncture is performed, and there is an elevation of his CSF pressure. Cerebrospinal fluid is sent to the lab. The CSF is centrifuged. A drop of India ink is placed on the slide along with a drop of the spun CSF. The India ink test is positive. Question What diagnostic sign is most likely to be found in this patient?

Correct answer: Kernig's sign Explanation Kernig's sign is a sign of meningeal irritation. The patient lies on his back, and his leg is raised and knee bent at a 90-degree angle. If there is pain or resistance to further extension when the examiner straightens the knee, Kernig's sign is said to be positive. This patient has signs and symptoms of cryptococcal meningitis. Cryptococcus neoformans is a yeast with a capsule made of polysaccharides. India ink exam is positive for Cryptococcus neoformans. When visualized on high dry magnification, encapsulated organisms (Cryptococcus neoformans) have capsules that look like halos because they exclude the India ink. Cryptococcus neoformans can cause meningitis, and there is an increased risk of Cryptococcus neoformans meningitis in immunosuppressed individuals. Babinski's sign is an abnormal response to stimulation on the sole of the foot that results in dorsiflexion of the big toe and fanning of the other toes. Babinski's sign is seen with pyramidal disease. Kussmaul's sign is an increase in venous pressure during inspiration; it can be seen with cardiac tamponade. Quincke's sign is also called Quincke's pulse. This sign is seen in the nails. It consists of whitening and reddening of the nail bed coinciding with each heartbeat. Quincke's sign can be seen with aortic insufficiency. Chvostek's sign can be seen with tetany. When tapping on the facial nerve produces contraction on that side of the face, it is called Chvostek's sign.

Case A 45-year-old African-American man with no significant past medical history presents with a 1-hour history of left retroorbital headache. The headache was of a sudden onset, and it began upon waking this morning. It is described as excruciating, stabbing, sharp, and lancinating; it is rated as severe in intensity. He denies any preceding infections, nausea, vomiting, photophobia, or osmophobia; he denies fever, chills, stiff neck, focal weakness, numbness, tingling, vision, hearing, gait, or speech changes. He recalls a similar episode several months ago; it lasted about 1 week and dissipated without complications. His physical exam is remarkable for painful distress, nasal congestion with rhinorrhea, left ocular miosis, and left forehead flushing diaphoresis. Question What is an expected manifestation in this patient?

Correct answer: Lacrimation and conjunctival injection Explanation The correct response is lacrimation and conjunctival injection. This patient's most likely diagnosis is most likely a cluster headache. These headaches are typically short in onset; they last 5 minutes to 3 hours, and they occur with a clear periodicity, particularly during sleep or early morning hours. They usually correspond with the onset of REM sleep. Its quality is excruciating, stabbing, sharp, and lancinating; it is distributed unilaterally, usually in the periorbital, retro-orbital, or temporal regions. Unlike migraines, cluster headaches are not preceded by aura, and they are not usually accompanied by symptoms such as nausea, vomiting, photophobia, and/or osmophobia. The headache is often accompanied by cranial parasympathetic symptoms, such as ipsilateral lacrimation or conjunctival injection, nasal stuffiness or rhinorrhea, ipsilateral eyelid edema, miosis or ptosis, and ipsilateral forehead and facial perspiration. Cervical muscle tenderness is suggestive of a tension headache, while the finding of temporal artery tenderness is found in temporal arteritis. Visual scintillations and scotomas are associated with an aura-related migraine headache. Papilledema indicates increased intracranial pressure, which is due to an underlying structural etiology.

Case Ico-delete Highlights A 40-year-old man presents with drooping of eyelids and difficulty getting up from a sitting position. He also gives a history of impotence and frequent dryness of his mouth. On examination, he is found to have ptosis, depressed reflexes, and loss of power in the proximal muscle groups of the lower extremity. On electrodiagnostic testing, he is found to have incremental response on repetitive nerve stimulation. Question What is the likely diagnosis in this patient?

Correct answer: Lambert Eaton syndrome Explanation The above history and findings are suggestive of Lambert-Eaton syndrome. Lambert-Eaton myasthenic syndrome (LEMS) is a presynaptic neuromuscular disorder that causes weakness of the muscles. LEMS is caused by antibodies directed against the calcium channel on the motor nerve terminal, leading to the decreased release of acetylcholine. Half of the patients have an autoimmune form, and the other half have a tumor-associated form (usually small-cell lung cancer) of the disease. Hence a thorough investigation has to be done to detect occult malignancy. Clinical features include: Weakness of proximal muscle groups initially (especially the lower limb) and later other muscles. Ptosis and diplopia. Depressed or absent reflexes. Autonomic changes such as dry mouth, impotence, etc. Worsening of symptoms in the morning and improvement with exercise, as opposed to myasthenia gravis Diagnostic data includes incremental response of the muscle groups on repetitive nerve stimulation, as opposed to decreased response in the case of myasthenia gravis. Treatment includes plasmapheresis and immunosuppression. A significant number of patients have clinically benefited from 3, 4-diaminopyridine (DAP). Immunotherapy should be considered if severe weakness persists despite DAP. Plasma exchange and high-dose immunoglobulin induce a transient improvement in many patients, but function rarely becomes normal. Multiple sclerosis is a demyelinating disease that may present with impaired vision and loss of proprioception. Ptosis is not a common presenting feature. Repetitive nerve stimulation would not increase strength in patients with this disease. Botulism is a paralytic disease involving the cranial nerves. It progressively involves the extremities in a rostrocaudal manner. It is caused by a neurotoxin produced by the bacteria Clostridium botulinum (anaerobic Gram-positive organism) and is primarily associated with home-canned foods. Cranial nerve involvement marks the onset of the disease with diplopia, dysarthria, dysphagia, and muscular weakness/descending paralysis. Ptosis is frequently seen. Diagnostic features include the absence of fever, ptosis, depressed pupillary reflexes, suppressed gag reflex, and normal or depressed deep tendon reflexes. Paralytic ileus, severe constipation, and urinary retention may also be seen. Progressive external ophthalmoplegia is a rare condition usually involving the extraocular muscles and proximal muscle groups. This is usually caused by some mitochondrial disorder, and it is usually diagnosed by muscle biopsy. Myasthenia gravis is a neuromuscular disorder characterized by weakness and fatigability of the skeletal muscles. The pathophysiological cause is the decrease in the number of acetylcholine receptors in the postsynaptic membrane. Due to the decreased receptors, the amount of acetylcholine bound to the postsynaptic receptors are decreased, leading to inadequate depolarization and ultimately resulting in weakness of the muscles. Clinical features include muscular weakness and generalized fatigue. The weakness increases on repetitive use of a group of muscles, and it reduces on rest.

Case Ico-delete Highlights A 64-year-old man presents with difficulty speaking, trouble swallowing, and right arm weakness. Physical examination reveals dysarthria, tongue deviation to the right, paralysis of the left soft palate, left facial droop, left facial sensory loss, and right arm weakness. Question These findings suggest involvement in what area?

Correct answer: Left brain stem Explanation Although precise localization of neurologic deficits is often difficult, certain patterns are important to recognize. Brain stem involvement results in 'crossed findings' (e.g., right facial weakness and left arm weakness) because the lesions affect the brain stem nuclei directly (uncrossed) and the corticospinal tract as it is crossing to the opposite side of the body. Because of the unique anatomic arrangement of the brainstem, a unilateral lesion within the structure can cause 'crossed findings' in which ipsilateral dysfunction of 1 or more cranial nerves is associated with hemiplegia and hemisensory loss on the contralateral side of the body, as described in this case. Medullary syndromes, such as lateral medullary syndrome, are associated with vertigo, nausea, vomiting, nystagmus as well as hiccups, and diplopia; they were not observed in this patient. Medial medullary syndrome is associated with tongue deviation toward the lesion, which is opposite to what is described in this case. Cerebral (cortical) lesions produce contralateral motor and sensory findings in the limbs and contralateral cranial nerve deficits (e.g., right arm and right facial paralysis). Pontine lesions result in coma, miosis, gaze paresis, and altered respiratory patterns. Cerebellar lesions produce nystagmus, dizziness, nausea and vomiting, and the inability to stand or walk if midline cerebellar areas are involved.

Question A 74-year-old woman presents for management of an ischemic stroke. She reports difficulty seeing objects on her right side. You perform confrontational visual field testing as part of your neurological examination, and you discover she has a right inferior homonymous quadrantanopsia. This lesion localizes to what part of the body?

Correct answer: Left parietal lobe Explanation Quadrantanopsia (visual loss in a quadrant of the visual field) may result from several causes, of which stroke is the most common. Accurate visual field testing can often localize the region of ischemia. An inferior homonymous quadrantanopsia localizes to the contralateral parietal lobe where the superior optic radiation fibers run. Other symptoms such as hemiplegia or hemisensory loss may also be seen. An optic tract lesion would cause a contralateral homonymous hemianopsia. An optic chiasmal lesion would cause a temporal non-homonymous hemianopsia. A left optic nerve lesion would cause unilateral, not bilateral visual field loss.

Case Ico-delete Highlights A 54-year-old man presents after having a generalized seizure. The patient is HIV-positive, but he has been unable to afford antiretroviral therapy since losing his job 2 years ago. Other than cachexia, the physical exam is unremarkable. Upon further inquiry, the patient also notes that he has become short-tempered and hypercritical; at times he seems confused. An MRI of the brain is performed, and it reveals several cortical ring-enhancing lesions. Question What is the most likely diagnosis?

Correct answer: Toxoplasma encephalitis Explanation The patient's symptoms and MRI findings are most consistent with the diagnosis of toxoplasmi encephalitis. Toxoplasmosis is the most common cerebral mass lesion among HIV-positive patients. Infection with the Toxoplasma gondii parasite is relatively common and usually asymptomatic. Reactivation occurs in HIV-positive patients due to failing cellular immunity, and it causes a multifocal necrotizing encephalitis. Seizures may be the initial manifestation of central nervous system (CNS) infection; other common clinical manifestations include focal neurologic deficits (e.g., impaired speech and hemiparesis). Personality change, lethargy, headache, and confusion are also observed. The MRI in patients with toxoplasma encephalitis characteristically reveals multiple, ring-enhancing lesions with surrounding edema; these lesions usually occur bilaterally in the frontal and parietal cortices. AIDS dementia complex describes a constellation of cognitive symptoms seen among HIV-positive patients. The condition occurs when the HIV virus disseminates to the CNS. Within the CNS, the virus tends to concentrate in the basal ganglia and subcortical regions. Symptoms include a constellation of cognitive, behavioral, and motor disturbances that cause varying degrees of functional impairment. Characteristic MRI findings include non-enhancing white matter, cerebral atrophy, and ventricular enlargement. The diagnosis requires that other central nervous system infections, carcinoma, general medical conditions, and substance abuse have been excluded. Cryptococcal meningitis is caused by the encapsulated fungus Cryptococcus neoformans. Among HIV-positive patients, the illness may be the result of new infection or reactivation of latent infection. Presenting signs are often nonspecific; they include headache, fever, change in mental status, and nausea or vomiting. Nuchal rigidity and photophobia may also be present, and elevated intracranial pressure is not uncommon. MRI findings vary, but they include lesions in the basal ganglia; meningeal enhancement, cerebral edema, and shrunken ventricles may also be seen. Cytomegalovirus (CMV) infection causing encephalitis is usually observed in patients with evidence of CMV infection at other sites. MRI findings vary, but they often show areas of focal necrosis within the brain parenchyma, meninges, or periventricular regions. Symptoms typically reflect progressive dementia, with episodes of confusion, apathy, and focal neurologic deficits. Progressive multifocal leukoencephalopathy is often a fatal disorder. It is caused by reactivation of a latent JC viral infection. Focal neurologic deficits (e.g., hemiparesis and gait disturbance) are often the initial presenting symptoms; they are followed by progressive cognitive decline, coma, and death. The MRI commonly reveals multiple, non-contrast-enhancing foci in the cerebral white matter.

Question A 31-year-old man with a known generalized seizure disorder is brought to your emergency department. His friends tell you that the patient had a seizure and did not wake up. When he did not wake up after 30 minutes, his friends called 911. On examination, he is breathing and his heart is beating. He is warm, dry, and pink. His basic laboratory values are within normal limits, and the computerized axial tomography (C.A.T.) scan of his brain is unremarkable. An emergency electroencephalogram (E.E.G.) is not available. His only medications are phenytoin and phenobarbital. What is the best initial treatment in this case?

Correct answer: Lorazepam (Ativan) Explanation According to consulting neurologist Wade S. Smith, Audio-Digest Family Practice, Vol. 49, 2001, "Consciousness is usually regained in a few minutes following a generalized seizure. When consciousness is not promptly regained following a seizure, then the diagnosis of status epilepticus becomes a reasonable consideration." When the emergency electroencephalogram (E.E.G.) is not readily available on a 24-hour basis, Professor Smith recommends that empiric pharmacotherapy for a working diagnosis of status epilepticus be provided. The first pharmacotherapy recommended for status epilepticus is lorazepam (Ativan). The second line pharmacotherapy for status epilepticus is phenytoin (Dilantin). The third line pharmacotherapy for status epilepticus is phenobarbital. Note that the absence of tonic-clonic movements does not exclude the diagnosis of status epilepticus, as it is possible that it will present with the solitary manifestation of persistent loss of consciousness. With his normal laboratory values, glucose would not be of value for this patient. Additionally, with the normal computerized axial tomography (C.A.T.) scan, the neurosurgical placement of burr holes by craniotomy would be nonfunctional.

Case A 5-month-old male infant presents after a seizure involving all four limbs. His mother tells you that he was born full term without any complications, and he was well until 2 days ago when he developed a fever. He vomited multiple times yesterday and was irritable. He has not had diarrhea or a cough. He was given antipyretic medication for his fever. He has no known allergies. His immunizations are up to date. His developmental milestones have been in accordance with his age. On physical exam, his temperature is 102.7°F, and his pulse is 154/min; BP is 90/50 mm Hg, and RR is 20/min. He is lethargic, pale; there are no focal neurological deficits. You suspect that he has bacterial meningitis. Question After drawing blood samples for investigations, what is the most appropriate next step?

Correct answer: Lumbar puncture Explanation Antimicrobial therapy along with dexamethasone should be initiated as quickly as possible but after the performance of the lumbar puncture (LP) unless the infant is acutely unstable and lumbar puncture cannot be performed expeditiously or there are focal neurologic deficits or signs of increased intracranial pressure. Intravenous phenytoin may be needed to control ongoing seizures but is not a first-line treatment for suspected meningitis. CT head is usually only indicated for patients with pre-existing immune deficiency, focal neurologic deficits, papilledema, or history of CNS tumor or surgery. Intravenous sodium bicarbonate is only used in the treatment of severe acidosis. Intravenous glucose is necessary if the patient is found to be hypoglycemic; bedside serum glucose is mandatory in any patient that presents with a seizure. Glucose should not be initiated empirically.

Case A 31-year-old female nurse who works at a local hospital presents with a purpuric rash covering her arms, legs, and abdomen, as well as fever, chills, nausea, abdominal tenderness, tachycardia, and generalized myalgias. Prior to the development of the rash, the patient noted that she had a headache, cough, and sore throat. Laboratory studies were positive for gram-negative diplococci in the blood, along with thrombocytopenia and an elevation in PMNs. Urinalysis showed blood, protein, and casts. Vital signs are as follows: BP 92/66, P 96, RR 14, T 39. The patient denies any foreign travel and does not have any sick contacts. She is admitted to the hospital and placed in respiratory isolation. Question What is the most likely diagnosis?

Correct answer: Meningococcemia Explanation Meningococcemia is an acute onset disease caused by the organism Neisseria meningitidis, which is an encapsulated, Gram-negative, aerobic, nonmotile diplococcus. When the organism enters the bloodstream and is disseminated, the result is meningococcemia. Patients with acute meningococcal infection may present with meningitis, meningitis with meningococcemia, or meningococcemia. Humans transmit the bacteria via aerosols or secretions from the nasopharynx. Infection is preceded by colonization of the nasopharynx. The bacteria may then enter the bloodstream and spread to areas, such as the meninges or joints, or disseminate throughout the body. The infection may begin with sore throat, cough, and headache. Later, the patient experiences a rapid onset of fever with chills, myalgias, and arthralgias. The progression can be quite rapid, which makes early diagnosis extremely important. Fulminant meningococcemia may develop within a few hours, with rapid enlargement of petechiae, hypotension, cardiac depression, and purpuric lesions. In patients with meningitis, headache, neck stiffness, lethargy, altered mental status, seizures, and drowsiness may develop. On physical exam, patient may have tachycardia, hypotension, and moderate fever. The fever increases with fulminant meningococcemia. A petechial/purpuric rash is present in approximately 55-80% of patients. It frequently involves the axillae, trunk, flanks, and ankles. Petechiae are often located in the center of lighter-colored macules. Serious complications include congestive heart failure, pulmonary edema, renal failure, and other end-organ damage. Laboratory analysis shows the presence of meningococci from blood, spinal fluid, or joint fluid. About 50-80% of skin scrapings yield N.meningitidis if the skin chosen is over petechiae or a pustule. More research is being done on this to find a rapid method for diagnosis. Polymorphonuclear leukocyte levels are usually elevated, and thrombocytopenia may be evident. Thrombotic Thrombocytopenic Purpura can affect any organ system, including the blood, the CNS, and kidneys. It is diagnosed by bland thrombi in the microvasculature of the affected organs. These thrombi are made mainly of platelets with fibrin and red blood cells mixed in. The cause is unknown at present. Rocky Mountain spotted fever is a tick-borne disease caused by the organism Rickettsia rickettsii. It is the most fatal tick-borne disease in the United States. It is most common in rural and suburban areas of the Southeastern U.S. Patients may present with fever, myalgias, headache, and a maculopapular rash that usually appears about 2 to 6 days after infection. It begins on the wrists and ankles and then spreads to the rest of the body. It also spreads to the palms and soles of the feet. The rash is usually diagnostic. Henoch-Schönlein purpura is an inflammatory disorder characterized by a generalized vasculitis involving the small vessels of the skin, kidneys, GI tract, joints, lungs, and CNS. It is the most common vasculitis in children. It is thought to be an immunoglobulin A (IgA)-mediated autoimmune disease. An antigenic stimulant is thought to cause a rise in IgA, and the resulting antigen-antibody complexes deposit locally throughout the body and activate pathways leading to necrotizing vasculitis. Physical exam shows a rash, migratory polyarthritis, and renal and GI involvement. There will be antigen-antibody complexes depositing throughout the body, which can cause migratory arthralgias, abdominal cramping, the petechial and/or vasculitic rash, and hematuria. Lab tests would not show bacteria within the blood.

Case Ico-delete Highlights A 22-year-old woman presents with an 8-hour history of headache and vomiting. She resides in a college dormitory; she has no remarkable travel history, and reports eating food from a street vendor 24 hours before becoming ill. The patient is lethargic and disoriented. On examination, she is found to have a temperature of 38.5° C and a petechial rash on her legs. Gram stain of exudate from a skin lesion shows Gram-negative diplococci. A lumbar puncture is performed; the cerebral spinal fluid (CSF) reveals a leukocyte count of 5/μL consisting of primarily neutrophils. A Gram stain of CSF is negative for bacteria. Question Ico-delete Highlights What is the most likely diagnosis?

Correct answer: Meningococcemia Explanation Meningococcemia may evolve rapidly, with death ensuing before the development of meningitis. The presence of Gram-negative diplococci and the clinical picture are consistent with a diagnosis of bacteremia caused by Neisseria meningitidis. While also a Gram-negative diplococcus, N. gonorrhoeae is an unlikely cause of this clinical presentation. Listeria monocytogenes, transmitted by contaminated food, is also a common cause of meningitis. This organism is not typically associated with a petechial rash; it appears as a Gram-positive coccobacillus in the CSF, where mononuclear cells predominate. Rocky-Mountain spotted fever is caused by the Gram-negative obligate intracellular pathogen Rickettsia rickettsii, which is not detectable by Gram stain. Infection with R. rickettsii results in a macropapular rash. Viral meningitis is usually accompanied by monocytes in the CSF, although neutrophils may appear early in the disease.

Case A 34-year-old woman, an immigrant from South America, presents with an acute onset of chest palpitations and shortness of breath. The patient also gives a 1-week history of fever, fatigue, and weakness. Echocardiography reveals that her heart is dramatically increased in size. Electrocardiogram (EKG) shows low voltage QRS with atrioventricular (AV) conduction abnormality. It is determined that she has Chagas disease. Question What is a complication that occurs during the acute phase of this patient's condition?

Correct answer: Meningoencephalitis Explanation Chagas disease is also referred to as American trypanosomiasis; it is seen in the Western Hemisphere, primarily in South America, Central America, and Mexico. It is caused by the protozoa Trypanosoma cruzi. It can be acute or chronic; the manifestations in the acute phase include prolonged fever, tachycardia, fatigue, anemia, weakness, mild hepatosplenomegaly, and lymphadenopathy. Meningoencephalitis is the complication occurring during the acute phase; this stage resolves spontaneously by 4 - 8 weeks in 90% of the individuals who are infected. The persistence of the disease leads to development of the intermittent, or chronic, phase. Chronic infection can lead to cardiomyopathy. The cardiac changes seen with chronic Chagas disease include an enlarged heart and EKG findings of right bundle branch block and premature ventricular contractions. Chronic infection can also lead to megaesophagus, megacolon, and thromboembolism. Gastric dilatation and megaureter may occur, but they are rare.

Case Ico-delete Highlights An 18-year-old woman presents with a history of recurrent headaches; they mainly occur above her right eye. She describes them as "throbbing" in quality. During these headaches, she gets bouts of nausea and vomiting. After a nap, she typically feels better. The headaches only last a few hours, and she experiences them roughly 2 times a month. Question What is the most likely diagnosis?

Correct answer: Migraine Explanation The clinical picture is suggestive of a migraine headache. The pain can be described as pulsatile in nature and is commonly unilateral. Other common manifestations include nausea, vomiting, and photophobia. Sinusitis commonly presents with unilateral or bilateral feeling of facial fullness, pressure, and tenderness over the cheeks. Cluster headaches are typically seen in middle-aged men. Unilateral periorbital pain may occur daily for several weeks. They are often associated with the following symptoms: ipsilateral nasal congestion, rhinorrhea, lacrimation, and redness of the eyes. Episodes often occur at night and last 15 minutes to 3 hours. Tension headaches are caused by muscle contractions that usually elicit a "band-like" tightness rather than throbbing pain. Often brought on by stress, they are usually not associated with nausea or vomiting, and they more commonly cause bilateral rather than unilateral pain. Intracranial masses cause an increase in intracranial pressure, which can cause headaches with vomiting. Headaches related to intracranial masses are typically worst in the morning, as recumbency increases intracranial pressure. Headaches due to intracranial pathology are likely to worsen over a short period and to be progressive in nature, rather than presenting as a cyclical course.

Case A 28-year-old woman presents with a 6-hour history of severe pulsatile hemicranial headache. She had numbness of the right upper limbs and lower limbs prior to the onset of the headache. The numbness lasted for 45 minutes. The headache worsens with exertion. She had a similar episode of headache, with nausea and vomiting, 2 weeks prior to presentation. She does not have a history of head injury. She has been married for 2 months, and she is on oral contraceptives. She gives a history of febrile convulsions in her childhood, and she has a sister with epilepsy. On examination, her blood pressure is 150/84 mm Hg, and neurological examination is normal. Her serum cholesterol level is 384 mg/dl. Question What is the most likely diagnosis?

Correct answer: Migraine Explanation The correct answer is migraine. The patient presents with a headache which is hemicranial and pulsatile, which are characteristics commonly seen with migraines. In addition, the association of nausea/vomiting and thepresence of a precipitating factor (i.e., being on oral contraceptives) increases the likelihood of it being a migraine. The numbness lasted for less than 60 minutes, which is probably an aura of sensory symptoms. Sensory epilepsy is an incorrect response. The numbness lasted 45 minutes and disappeared; however, in cases of sensory epilepsy, there is usually a distinct progression of symptoms. Cluster headache is an incorrect response. Cluster headaches are usually accompanied by at least 1 of the following ipsilateral autonomic symptoms: conjunctival injection or lacrimation, nasal congestion or rhinorrhea, eyelid edema, forehead and facial sweating, miosis or ptosis, and restlessness or agitation. Thromboembolic transient ischemic attack is an incorrect response. In cases of transient ischemic attacks, the maximum deficit is present immediately, and headache is unusual. Hypertension is an incorrect response. Systolic blood pressure of 150 mm Hg may probably be due to pain (headache). Mild hypertension {(140 - 159)/ (90 - 99) mm Hg} is usually asymptomatic.

Case A 22-year-old woman presents with headache. The pain is located on the right side of her head and is described as throbbing. The headache has lasted for approximately 10 hours and has been unrelieved by acetaminophen and ibuprofen. The pain worsens with movement, exposure to light, and loud noises. Her neurological examination is within normal limits. Question What is the most likely diagnosis?

Correct answer: Migraine headache Explanation Migraine headache is correct. The typical presentation of migraine is moderate to severe pain that is often unilateral and described as throbbing. Pain is often accompanied by nausea, vomiting, photophobia, and phonophobia. A visual, auditory, sensory, or motor aura may occur with migraine but does not need to be present for diagnosis. Migraine headaches typically last 4 to 72 hours. Cluster headache is incorrect. Cluster headaches typically present with severe unilateral pain lasting anywhere from 15 minutes to 3 hours. The "clusters" may occur multiple times daily and usually occur daily for weeks to months. Associated symptoms include nasal congestion, lacrimation, rhinorrhea, conjunctival erythema, miosis, ptosis, or anhidrosis on the affected side. Following a series of attacks, patients typically have remission of several weeks or even months. Tension headache is incorrect. Tension headaches are usually bilateral and described more as a tightness sensation than throbbing. Tension headaches usually last only 30 minutes. Brain tumor is incorrect. Symptoms of brain tumor vary by tumor location, but the headache tends to be worst in the morning and gradually increases in severity and frequency over time. Associated symptoms include vision changes, nausea, vomiting, sensory changes, confusion, and seizures. Pseudotumor cerebri is incorrect. Pseudotumor cerebri is a condition in which the intracranial pressure increases. The headache associated with pseudotumor cerebri typically is located behind the eyes and worsens with eye movement. Other associated symptoms include tinnitus, nausea, vomiting, dizziness, and changes in vision.

Case A 74-year-old man presents to discuss difficulty with orientation, memory, and word-finding. The orientation problems are worse at night. His problems began with mood and behavioral changes 7-8 years ago, after his brother passed away. He reports no trouble with activities of daily living. The patient has only an elementary school education, stating that he was "not smart enough to go to college". There is no family history of dementia. Question What is the most likely diagnosis?

Correct answer: Mild cognitive impairment Explanation Mild cognitive Impairment (MCI) is your first choice. It is a condition in which people face memory problems more often than that of the average person their age. However, these symptoms do not prevent them from carrying out normal activities and are not as severe as the symptoms for Alzheimer's disease. Symptoms often include misplacing items, forgetting events or appointments, and having trouble finding desired words. Intellectual disability is a generalized disorder appearing before 18 years and characterized by significantly impaired cognitive functioning and deficits in 2 or more adaptive behaviors. It has historically been defined as an Intelligence Quotient score (IQ) under 70.Besides cognition, the deficits may include other related mental functions and functional skills in the environment so that a person with a below-average intelligence quotient may not be considered to have intellectual disability. By definition, delirium is an acute confusional state, meaning that it has been present from hours to days, but not months. The first symptoms in your patient coincide with a traumatic event, but do not meet the criteria of post traumatic stress disorder (PTSD). PTSD requires that the symptoms last more than one month, causing significant impairment in social, occupational, or other important areas of functioning. Nyctalopia refers to the impaired vision in dim light and in the dark due to impaired function of the rods. It can interfere with some activities, but it is not the cause of cognitive decline.

Case A 77-year-old woman presents for a routine examination. She is worried about her memory; she has been forgetting where she puts things, such as keys, utility bills, or bank statements. She has no troubles with the activities of daily living. On examination, you find that there was no deterioration in her chronic heart disease due to high blood pressure (high blood pressure is effectively managed), but her cholesterol is still high and you recommend that she continue taking statins. Her family history is noncontributing. Both Mini-Mental State Examination and Clinical Dementia Rating scale suggests that her global cognitive function is satisfactory. Geriatric Depression Scale score excludes depression. Question What is the most likely diagnosis?

Correct answer: Mild cognitive impairment Explanation Regarding memory loss and other age-related declines in cognition, it is important to make a distinction between dementia (defined as severe deterioration of mental functioning due to underlying pathologies) and the condition called mild cognitive impairment (MCI). MCI is a decline in cognitive function greater than expected for age. Older individuals affected by MCI have obvious and measurable deficiencies in memory, language, and/or other cognitive functions. However, MCI does not significantly interfere with the individual's performance of daily activities. MCI can evolve in dementia, but at this point, your patient does not satisfy criteria for dementia. Dementia is a decline in previously acquired cognitive and behavioral abilities severe enough to interfere with work, social activities, activities of daily living, etc., that is not due to delirium or encephalopathy. Alzheimer's disease, which accounts for approximately 60% of all dementia cases, is characterized by its gradual onset and progressive cognitive declines. The key symptoms include early memory loss, confusion, apathy, depression, and anxiety. A combination of genetic and environmental factors leads to the aggregation of β-amyloid plaques in cerebral cortex and to the formation of neurofibrillary tangles, which is the underlying pathology of neuronal dysfunction. Pseudodementia includes a wide variety of underlying disorders, such as depression, schizophrenia, mania, dissociative disorders, Ganser syndrome, conversion reaction, and psychoactive drugs. Among them, depression is the most common; patients may complain of having memory problems and being upset about them; however, they will perform well on objective neuropsychological tests of memory. People with dementia often deny having any problems with memory or minimize their importance. They display impairment on neuropsychological tests. Both dementia and depression are excluded in your patient. The importance of pseudodementia is that it is often reversible. Vascular dementia refers to a group of diseases with distinct underlying mechanisms. The symptoms depend on the area affected by vessel lesions; it could be a major stroke caused by large-vessel disease, multiple small-vessel infarcts, ischemia-related leukoaraiosis, other vascular conditions, or the combination with Alzheimer's pathological changes. Your patient has several risk factors for vascular dementia, but, by definition, she does not have dementia.

Case A 32-year-old man presents with muscle weakness, lack of coordination, and tingling in his fingertips; he states that he has not been able to control his urinary function for the past few weeks. He also states that he has double vision sometimes. Question What is the most likely diagnosis?

Correct answer: Multiple sclerosis Explanation The clinical picture is suggestive of multiple sclerosis. The common clinical presentation is weakness, numbness, tingling, or unsteadiness in a limb; spastic paraparesis; retrobulbar neuritis; diplopia; disequilibrium; or a sphincter disturbance, such as urinary urgency or hesitancy. Common findings in Parkinson's disease are tremor, rigidity, bradykinesia, and postural instability. Parkinson's disease is commonly seen in older patients. Multiple myeloma is a malignancy of plasma cells characterized by replacement of bone marrow, bone destruction, and paraprotein formation with a median age-of-onset of 65 years old. Most patients present with lower back pain. Transient ischemic attacks (TIA) are characterized by focal ischemic cerebral neurological deficits that last for less than 24 hours. Symptoms vary from patient to patient, but they include abrupt or sudden onset; recovery usually occurs within minutes. Some common symptoms include weakness and heaviness in the contralateral arm, leg, or face. Strokes are similar to TIA, but symptoms last longer than 24 hours, and most patients have a history of hypertension, diabetes mellitus, valvular heart disease, or atherosclerosis.

Case A 36-year-old woman presents with a 2-day history of severe headache. The headache has kept her from going to work. When she shakes her head, the headache improves a little, but it does not resolve. It is not any worse in the morning upon waking, and she denies having had an aura at the beginning of the headache. Question What type of headache does she have?

Correct answer: Muscle tension Explanation Muscle tension headaches are not intensified by coughing or straining at stool, and they are ameliorated by shaking the head. Characteristics of vascular headaches include that they are intensified by coughing or straining at stool and not ameliorated by shaking the head. Vascular headaches include common migraines, classic migraines, cluster headaches, hypertensive headaches, fever headaches, and temporal arteritis headaches.

Case A 23-year-old woman presents with increasing fatigability and weakness. On further questioning, she reveals that, in the past month, she has experienced difficulty swallowing; there have also been episodes of double vision as the day progresses. Several of her professors have expressed concern because she looks sleepy during the day. A CT of the chest is advised. It reveals a mass that, once removed surgically, significantly lessens symptoms. The pathology slide made from the removed tissue can be found below. Question Based on the combined aspects of her history, findings upon investigation, and her recovery, what did the patient likely have?

Correct answer: Myasthenia gravis Explanation Myasthenia gravis is an autoimmune disorder in which patients present with the cardinal symptoms of weakness and fatigue.This condition is more common among young women. Clinical features of this condition include: Ptosis and diplopia Chewing and swallowing difficulties Respiratory difficulties Asymmetrical proximal limb weakness with normal deep tendon reflexes The initial symptoms are often related to muscles that are controlled by cranial nerves, as these are affected first. Confirmation of this condition and the elimination of differential diagnosis can be done using the edrophonium test. The administration of edrophonium, a short-acting anticholinesterase, results in a marked increase in the muscle strength for a short duration in patients with myasthenia gravis. Investigations done in patients with myasthenia gravis are: The acetylcholine receptor antibody test CT scan of the thorax to detect a thymoma if present Electromyogram (EMG) The patient in this question was found to have a mediastinal mass, which was found to be a mixed thymoma. (See image) Treatment of myasthenia gravis involves: Neostigmine and other anticholinesterase drugs, improving symptoms. Avoidance of drugs such aminoglycosides. Thymectomy(removal of the thymus), which produces an improvement in over 85% of the patients between puberty and 55 years of age. It is usually performed in cases of moderate-to-marked generalized myasthenia gravis, and in some, it can produce complete remission of the disease. A thymoma is an indication for surgery. Steroids provide for symptom management as well; improvement is the most dramatic with high-dose corticosteroids. Plasmapheresis and IV immunoglobulins. Amyotrophic lateral sclerosis is a type of motor neuron disease. This is a mixed upper and lower motor neuron deficit, which is found in the limbs and is often associated with Parkinsonism and dementia. This condition does not present with ptosis, and it does not have the characteristic variation of muscle power seen in myasthenia gravis. Myasthenic syndrome, otherwise referred to as the Lambert-Eaton Syndrome, is the result of the defective release of acetylcholine in response to a nerve impulse. The resulting weakness is more prominent in the proximal muscles of the limbs. Muscle contraction strength increases with sustained contraction. Diagnosis of the disease is by electrophysiological studies. It is usually associated with small cell carcinoma of the lung. Treatment is aimed primarily at the tumor, but plasmapheresis and immunosuppression have been successful. Botulism is due to consumption of the toxin of Clostridium botulinum, which prevents the release of acetylcholine at the neuromuscular junctions and the autonomic synapses. This is an acute condition. Symptoms appear within 72 hours of ingestion, and they require immediate medical attention. Inclusion body myositis is a disorder of middle age. It results in weakness of the proximal muscles of the lower and then upper limbs. The diagnosis of this condition is also by means of a muscle biopsy, and it does not respond to corticosteroids. This patient does not show any of the other clinical signs of hyperthyroidism, such as weight loss, increased appetite, preference for cold weather, nervousness, increased bowel movements, an enlarged thyroid, etc.

Case A 65-year-old woman presents with general weakness for the past few days, and it gets worse as the day progresses. She says she has been having double vision, and you notice she has poor posture. A blood test reveals the presence of antibodies to acetylcholine receptor. A repetitive nerve stimulation test showed a decremental response. Question What is the most likely diagnosis?

Correct answer: Myasthenia gravis Explanation Myasthenia gravis, a disorder more common in women, is characterized by chronic weakness of voluntary muscles. This weakness improves with rest and worsens with activity. The body produces an immune response (autoantibodies) against the nicotinic acetylcholine receptors at the neuromuscular junction. The most common presentation is muscle weakness that progressively worsens as the day progresses. The clinical features include ocular manifestations such as ptosis and diplopia, extra-ocular manifestations including limb and trunk weakness, dysphagia, generalized weakness, depressed gag reflex, limp body, slackening of the jaw, and respiratory depression. The diagnostic tests are anti-ACh receptor antibody assay, repetitive nerve stimulation (shows a decremental response), and electromyography. Treatment consists of the administration of acetylcholinesterase inhibitors, immunosuppressive agents like corticosteroids, cyclosporine, azathioprine, plasmapheresis, and intravenous gamma globulin. Thymectomy can also be done. Marfan syndrome occurs due to a mutation in the fibrillin-1 gene, located on chromosome 15. The cardinal features occur in 3 systems: skeletal, ocular, and cardiovascular. Skeletal manifestations include an increase in height, disproportionately long limbs and digits, joint laxity, anterior chest deformity, vertebral column deformity such as scoliosis and thoracic lordosis, and high-arched palate with crowding of the teeth. Ocular manifestations include myopia and subluxation of the lenses (ectopia lentis). Cardiovascular features are mitral regurgitation, mitral valve prolapse, dilatation of the aortic root, and aortic regurgitation. Molecular studies of fibrillin-1 gene and imaging studies should be carried out in patients suspected of Marfan syndrome. Duchenne muscular dystrophy is an X-linked recessive disorder. It is characterized by a delay in gross motor milestones, symmetrical proximal progressive muscle weakness, pseudohypertrophy of calf muscles, and lordotic posture. Positive Gower sign and waddling gait are diagnostic of Duchenne muscular dystrophy. Confirmation of diagnosis is by elevated levels of creatine kinase, DNA testing, and muscle biopsy. Rheumatoid arthritis is a chronic inflammatory systemic disease that primarily involves the articular surfaces and synovial membranes of multiple joints. It is characterized by the presence of rheumatoid nodules and rheumatoid factor. Assay for rheumatoid factor and imaging studies should be carried out in patients suspected of having rheumatoid arthritis. Systemic lupus erythematosus (SLE) is a chronic, inflammatory, multisystem disorder. SLE is an autoimmune disorder associated with the production of antinuclear antibodies (ANA). The manifestations include polyarthralgias, polyarthritis, malar rash, discoid lupus erythematosus, alopecia, aphthous stomatitis, anemia, and lupus nephritis.

Question A 32-year-old woman starts to notice that she is having difficulty brushing her hair. She also notices that her eyes are extremely tired after reading the evening newspaper. One night, her husband comments that she has droopy eyelids. She is relieved the next morning when her eyelids appear normal. Her condition deteriorates, and she has weakness and fatigability to the point that she finally sees her family doctor. On history, she denies pain. Her physical exam demonstrates weakness in her limbs. Her deep tendon reflexes are within normal limits. Her doctor wants to run a diagnostic test by injecting a drug. What is the most likely diagnosis?

Correct answer: Myasthenia gravis Explanation This patient's symptoms indicate myasthenia gravis. Ocular muscle weakness and ptosis are common. Muscular weakness and fatigue are consistent with this condition. Myasthenia gravis occurs due to the presence of antibodies to the acetylcholine receptor. A diagnostic aid is the Tensilon test. The Tensilon test entails the administration of a rapid acting anticholinesterase, edrophonium. By administering an anticholinesterase, any acetylcholine released will be present longer and will have a longer period of time to act at the neuromuscular junction. Duchenne's dystrophy would present at a very young age. It is X-linked. Polymyositis can present with weakness. However, the weakness is proximal; it is usually of the hips and thigh, but sometimes it is of the shoulder girdles. Polymyositis would spare the ocular muscles. Familial periodic paralysis can present with weakness. However, the weakness would be in the extremities and the trunk. It would also spare the ocular muscles. Botulism can present with weakness. In addition, the ocular muscles can be involved. However, botulism would present with a progressive weakness, rather than an intermittent weakness. There would also be a loss of deep tendon reflexes. Furthermore, there is nothing in her history to suggest ingestion of botulism toxin.

Case A 40-year-old woman presents with a 7-day history of pain in her right arm. The patient denies any trauma or injury to this extremity just prior to the pain starting, but she does admit to having a Colles' fracture in this arm around 2 months ago. She denies any injury to her back, neck, or any other musculoskeletal system prior to the event of pain. She describes the pain as burning and throbbing; there is an extremely diffuse, uncomfortable aching accompanying it. She further states that this limb has become extremely sensitive to touch and cold, and it appears somewhat more swollen than her left arm. The patient is very upset because she does not know why her arm is so painful when she has not done anything to it. She is a nonsmoker, does not drink, and exercises 3 times a week. Physical examination of the extremity reveals a slightly cyanotic, mottled right arm with generalized pain of the entire extremity. Pulses are faint (1+), and ROM is limited. Question Keeping in mind the most likely diagnosis, what is the most appropriate first-line therapy for the patient's signs and symptoms?

Correct answer: NSAIDs Explanation The correct response is NSAIDs. The patient in this case is most likely in stage I of complex regional pain syndrome (CRPS). This disorder is rare; it causes instability of the vasomotor and autonomic components. CRPS is a disorder of a body region, most commonly the extremities, that is characterized by pain, swelling, limited range of motion, vasomotor instability, skin changes, and even patchy bone demineralization. Frequently, symptoms will begin after the patient has sustained an injury, had surgery, or experienced a vascular event, such as a stroke. 35% of patients with CRPS report not having a distinct precipitating event. Stage 1 of CRPS characteristically has findings such as pain in the limb (burning/throbbing), diffuse and uncomfortable aching, sensitivity to touch or cold, or even localized edema. The distribution of pain is not compatible with a single peripheral nerve, trunk, or root lesion. There will also be signs of variable vasomotor disturbances that result in altered color and temperature. At this stage, if a radiograph is completed, it will most likely be normal, but it could show signs of patchy demineralization. Early treatment intervention provides the best prognosis for CRPS, and considering the patient is in an early stage, this is also a promising component. First-line treatment for mild cases is nonsteroidal anti-inflammatory drugs, which are commonly referred to as NSAIDs. Naproxen is commonly considered in this scenario. Calcitonin may be considered in patients who still have mild or moderate symptoms despite the use of NSAIDs. A meta-analysis of the results of 5 clinical trials showed that it is effective in treating pain caused by CRPS; however, calcitonin is not considered first-line treatment in patients. Sulfasalazine, TNF inhibitors, and etanercept are not indicated in the signs and symptoms caused by CRPS.

Case A 40-year-old woman presents because of a 7-day history of pain in her right arm. The patient denies any trauma or injury to this extremity just prior to the pain starting but does admit to having a Colles' fracture in this arm around 2 months ago. She denies any injury to her back, neck, or other musculoskeletal system prior to the event of pain. She describes the pain as burning and throbbing with an extremely diffuse, uncomfortable aching accompanying it. She further states that this limb has become extremely sensitive to touch and to cold, and it does appear somewhat more swollen than her left arm. The patient is very upset because she does not know why her arm is so painful when she has not done anything to it. She is a non-smoker, does not drink, and exercises 3 times a week. Physical examination conducted of the extremity reveals a slightly cyanotic, mottled right arm with generalized pain of the entire extremity. Pulses are faint (1+) and ROM is limited. You diagnose the patient with Stage I complex regional pain syndrome. Question A multidisciplinary approach is recommended in the treatment of this patient. What pharmaceutical agents is recommended as part of the initial treatment plan?

Correct answer: NSAIDs Explanation The patient being described is most likely in Stage I of complex regional pain syndrome (CRPS). This disorder is rare, causing instability of the vasomotor and autonomic components. CRPS is a disorder of a body region, most commonly the extremities, that is characterized by pain, swelling, limited range of motion, vasomotor instability, skin changes, and even patchy bone demineralization. Frequently, symptoms will begin after the patient has sustained an injury, had surgery, or experienced a vascular event, such as a stroke. 35% of patients with CRPS report not having a distinct precipitating event. Stage I of CRPS characteristically has such findings such as pain in the limb (burning/throbbing), diffuse, uncomfortable aching, sensitivity to touch or cold, or even localized edema. The distribution of pain is not compatible with a single peripheral nerve, trunk, or root lesion. There will also be signs of variable vasomotor disturbances that result in altered color and temperature. At this stage, if a radiograph was completed, it will most likely be normal but could show signs of patchy demineralization. All treatment options are found to be much more effective if they are begun as soon as the diagnosis is made or if the patient is diagnosed as Stage I. When considering treatment for this patient, one must take a multidisciplinary approach, including initiation of oral/topical pharmacologic agents, as well as physical and occupational therapy. Less invasive actions should be taken initially, which should include those to help tolerate the pain: NSAIDs, a tricyclic antidepressant, an anticonvulsant, or even an oral opioid. Patients have even been found to have pain relief from bisphosphonates, oral glucocorticoids, or even intranasal calcitonin (the response being more variable, however). Topical treatments, such as capsaicin cream, should also be initiated. The key to success is to use whatever works to reduce the pain to aid the patient's participation in weekly physical therapy. Those who are in Stage I and have no improvement after 2 weeks of therapy or those who are considered Stage 2 or 3 will receive more invasive therapies to help alleviate pain. This will include such actions as epidural clonidine. This treatment may result in significant side effects; hypotension or sedation may be substantial. There are many investigations for treating longstanding or refractory forms of CRPS. These include ketamine infusion, intravenous immunoglobulin administrations, or even mirror visual feedback therapy. These should not be considered first-line treatment regimens.

Case A 27-year-old woman presents for the evaluation of hypersomnia; her symptoms began approximately 5 years prior to presentation. The hypersomnia persisted despite the fact that she was sleeping 6-8 hours per night during the week and up to 10 hours on the weekends. She has a difficult time maintaining wakefulness while at work. It has been difficult developing a social life since she often goes to bed early on weekends to catch up on her sleep. She often has vivid dreams and frequent hypnagogic hallucinations. She has long-standing sleep paralysis; it typically occurs 1-2 times a month. She has had episodes of cataplexy, with several episodes within a 1-week period. An episode would occur if she got excited or stressed. Her neck muscles would weaken, and she would have an irresistible urge to sleep. During these episodes, she was able to hear everything that was going on around her, but she could not respond. The episodes typically lasted a few minutes. She denies a history of head trauma or depression. She undergoes an overnight sleep study that demonstrates an apnea-hypopnea; she slept for 7.4 hours. Question What is the most likely cause of the patient's symptoms?

Correct answer: Narcolepsy Explanation Narcolepsy is characterized by daytime sleepiness, cataplexy, hallucinations, and sleep paralysis. It is a chronic sleep disorder caused by the brain's inability to regulate sleep-wake cycles normally. Narcolepsy is thought to result from genetic predisposition, abnormal neurotransmitter functioning and sensitivity, and abnormal immune modulation; human leukocyte antigen subtypes and abnormalities in monoamine synaptic transmission are thought to be involved. The cause of narcolepsy remains unknown. Men and women are affected equally, and children as young as 2 may also be affected. The age of onset distribution is biphasic; the highest peak occurs at the age of 15, and a smaller peak occurs at around age 36. Dysfunction and inappropriate regulation of rapid eye movement (REM) sleep is thought to cause narcolepsy. At various times throughout the day, patients with narcolepsy feel strong urges to sleep and may fall asleep anywhere within a few seconds to several minutes. In rare cases, some people may remain asleep for an hour or longer. Patients suffer from extreme daytime sleepiness, sudden loss of voluntary muscle tone (cataplexy), vivid hallucinations during sleep onset or upon awakening, and brief periods of total paralysis at the beginning or end of sleep. The condition may not be diagnosed until 10-15 years after the first symptoms appear. Absence seizures are brief episodes of staring. Awareness and responsiveness are impaired. Patients usually do not realize when they have had a seizure; there is no warning before, and the person is alert immediately afterward. Simple partial seizures are seizures that usually affect the temporal lobes and/or hippocampi. Simple partial seizures are usually followed by larger seizures later in life; the abnormal electrical activity spreads to a larger area of the brain (usually), resulting in a complex partial seizure or a tonic-clonic seizure. Frontal lobe epilepsy is either simple partial or complex partial, mostly with secondary generalization; it is characterized by recurrent seizures. The clinical presentation ranges from postural, behavioral, tonic, to motor manifestations. Status epilepticus is usually associated with frontal lobe seizures. Delayed sleep-phase syndrome (DSPS), which is also known as delayed sleep-phase disorder or delayed sleep-phase type, is a circadian rhythm sleep disorder. It is a chronic disorder of the timing of sleep, peak periods of alertness, hormonal, and other rhythms. People with DSPS tend to fall asleep well after midnight, and they also have difficulty waking up in the morning.

Case A 75-year-old African-American man has a past medical history that is significant for severe and uncontrolled hypertension; he is brought into the emergency room by his family due to a 30-minute history of change of mental status. The patient had just climbed stairs when he first developed a headache that has become progressive; it is associated with nausea, non-bilious vomiting, and unilateral upper and lower extremity numbness. His physical exam reveals an alert and oriented times zero patient. He is afebrile, with a blood pressure of 185/108 mm Hg, and there is nuchal rigidity. His neurological exam notes contralateral sensory loss, contralateral hemiparesis, gaze paresis, homonymous hemianopia, and miosis. A CT scan without contrast was performed, and it is shown in the image. Question Ico-delete Highlights What medication would be most appropriate in this patient?

Correct answer: Nicardipine Explanation The correct response is nicardipine. This patient is experiencing an intracerebral hemorrhage, with underlying hypertension as a major contributory etiologies. Antihypertensive agents reduce blood pressure to prevent exacerbation of intracerebral hemorrhage. Pressure should be cautiously lowered to a mean arterial pressure (MAP) less than 130 mm Hg. Excessive hypotension should be avoided. Early treatment in patients presenting with spontaneous intracerebral hemorrhage is important as it may decrease hematoma enlargement and lead to better neurologic outcome. Nicardipine is a calcium channel blocker with a potent rapid onset of action, ease of titration, and lack of toxic metabolites. It (as is Labetalol) is an agent that reduces blood pressure in order to prevent exacerbation of intracerebral hemorrhage. Patients with intracerebral bleeding should not be given blood-thinning agents. Instead, vitamin K and protamine may be used to restore normal coagulation parameters. Furthermore, recombinant factor VIIa (rFVIIa) within 4 hours after the onset of intracerebral hemorrhage limits the growth of the hematoma, reduces mortality, and improves functional outcomes at 90 days. Dobutamine should be avoided; it is a sympathomimetic agent. It has beta and alpha-1 adrenergic effects in the body, causing tachycardia and vasoconstriction. Pyrimethamine is the most effective agent in the treatment of toxoplasmosis, and it is included in most drug regimens. Methotrexate is the single most effective chemotherapeutic agent for primary central nervous system lymphoma.

Case A 74-year-old man presents after his wife witnessed him grab his head in pain and fall to the floor. He has not regained consciousness. His current blood pressure is 150/96 mm Hg and his heart rate is 65 bpm. Emergent head CT shows a subarachnoid hemorrhage. Question In addition to life-saving interventions, what prescription medication will most benefit this patient at this time?

Correct answer: Nimodipine (calcium channel blocker) Explanation Nimodipine (a calcium channel blocker) has been shown to improve outcomes in patients following aneurysmal SAH. The mechanism of action is thought to be prevention of ischemia. Glucocorticoids (e.g., prednisone) are often utilized in patients with SAH because they offer symptomatic relief of headache and neck pain, but they have not been proven to decrease cerebral edema. Antifibrinolytic agents (e.g., tranexamic acid) can be utilized in patients with a diagnosed aneurysm who cannot undergo directed treatment, but they are not routinely used following aneurysmal rupture. Labetalol (a beta blocker) may be utilized to treat elevated BP, but it must be used with caution because it can also decrease cerebral perfusion. Diuretics (e.g., furosemide) have no identified role in the treatment of aneurysmal SAH.

Case Ico-delete Highlights A 24-month-old boy is brought in by his mother. Although he has been fairly healthy, she is concerned about his speech and language development. The mother says his 4-year-old sister, when she was 2 years of age, was talking in 3-4 word sentences, asking and answering questions, and had an extensive vocabulary. Question What finding would warrant a referral for delay in speech and language?

Correct answer: No use of 2-word phrases Explanation Use of 2-word phrases should begin to be evident between 18 to 24 months of age, as well as the ability to learn words quickly and understand sentences. Between 24 and 36 months, the child is also beginning to follow 2-step commands, answer questions, ask what questions, and be about 50% intelligible. Another indication of speech delay or disorder at 24 months would be a vocabulary of less than 50 words. Flat or stilted intonation in speech indicates a delay or disorder at 36 months of age. Rote memorization of words or phrases that may be seen at 36 months of age (and sometimes of dialogues or scripts from videotapes) indicates the child can hear but does not understand what has been said, and may reflect a disorder of language processing and be a marker of possible autism. An inability to participate in conversation at 48 months of age is an indication of a speech delay or disorder. Having more than 75% of speech be unintelligible to strangers would warrant referral at 36 months of age

Case A 32-year-old woman presents several hours after a grand mal seizure. Her husband states that a few days before the seizure she had experienced headache, nausea, vomiting, fever, and "was not herself". She is on glucocorticoid therapy due to a kidney transplant 1 year ago. On examination, you find a lethargic, febrile (39 C), dysphasic patient; there is right-sided hemiparesis, and meningeal signs are present, but detailed examination cannot be performed because of lack of cooperation. EEG shows focal abnormalities over temporal lobes, and her CT is normal. A lumbar puncture is performed. Based on your suspected diagnosis, you initiate supportive care, anticonvulsants, and acyclovir. The patient improves over the following days. Question What additional finding would support your suspected diagnosis?

Correct answer: Normal CSF: serum glucose ratio Explanation The patient has signs and symptoms consistent with meningoencephalitis. The etiology of meningoencephalitis includes viral, bacterial, fungal, and tubercular infections. Subarachnoid hemorrhage and mass lesions may also cause meningeal irritation. In an immunocompromised patient with the clinical presentation, in this case, a viral meningoencephalitis such as herpes should be high on the differential. Its presentation is often atypical: fever, headache, and alteration of consciousness, focal neurological deficits, seizures, and other signs of acute encephalitis. Acyclovir is an antiviral drug with activity against the herpes virus. Lumbar puncture to obtain CSF for evaluation is a key part of determining the etiology of meningitis. CSF characteristics suggestive of viral meningoencephalitis include 1) normal to slightly elevated opening pressure; 2) predominance of lymphocytes; 3) normal protein level; 4) normal glucose level; and a 5) normal CSF: serum glucose ratio (0.6-0.7). Bacterial causes of meningitis are associated with 1) elevated opening pressure; 2) predominance of polymorphonuclear cells (PMNs); 3) elevated protein level; 4) reduced glucose level; and 5) reduced CSF:serum glucose ratio (0.4). Fungal and tubercular causes of meningitis are associated with 1) variable opening pressure; 2) predominance of lymphocytes; 3) elevated protein level; 4) reduced glucose level; and reduced CSF:serum glucose ratio.

Case A 32-year-old man presents with severe headaches. He reports that he had experienced 2 of these severe headaches about a month ago, and now has had 2 more, which have woken him from sleep. The headaches lasted anywhere from 15 minutes to an hour, then resolved entirely. His wife urged him to seek care for this one, as she was worried about a stroke. He complains of severe head pain, and his left eye appears droopy, slightly swollen, and it is watering. His nose is runny. He denies history of head trauma, memory loss, gait disturbances, muscle weakness, nausea, and vomiting. Over-the-counter acetaminophen and ibuprofen have not been helpful. He has no known medical conditions, takes no regular medications, and has no known allergies. He has never had surgery. He denies any family history of headaches or neurological conditions. He smokes 1 pack/day and drinks alcohol 3-4 times/week. He denies the use of drugs. On physical exam, the patient appears restless. Ptosis and pupillary constriction are observed in his left eye, and some clear nasal discharge is noted. The remainder of his exam is normal. Question What test results best support this patient's likely diagnosis?

Correct answer: Normal magnetic resonance imaging (MRI) Explanation This patient is most likely experiencing cluster headaches, in which he would be expected to have normal magnetic resonance imaging (MRI). Cluster headaches are intense, episodic headaches that occur more commonly in men than women. Symptoms are unilateral, often accompanied by rhinorrhea and lacrimation, miosis, and ptosis of the eye. Inflammation on temporal artery biopsy indicates temporal arteritis, which can cause severe unilateral headache. Temporal arteritis is more likely to occur in an older person and is not associated with rhinorrhea and these optic symptoms. (There may be amaurosis fugax, diplopia, and vision loss.) Polyspikes and slow wave pattern on electroencephalogram (EEG) indicates seizure activity. This patient's symptoms and headache pattern are not associated with any seizure disorder. If the patient had positive toxicology for cocaine, he could present with restlessness and mydriasis (dilation of the pupils, bilaterally) rather than unilateral pupillary constriction. Thickened sinus mucosa on computed tomography (CT) indicates chronic sinusitis, which can cause headaches. The headaches associated with chronic sinusitis are much less intense/acute and episodic than those of cluster headaches.

Case Ico-delete Highlights A 19-year-old woman presents with worsening headaches. She reports a multi-year history of episodic, throbbing headaches. They have intensified, and she now misses classes and work periodically due to her headaches. The headaches occur about 4-6 times per month recently, up from 1-2 per month when she first started experiencing headaches. Her headaches last 2-3 days and are accompanied by nausea, vomiting, and light sensitivity. After the headache resolves, she denies any residual symptoms. She denies neurologic symptoms, such as vision or taste changes, gait disturbances, and memory loss. She has tried multiple over-the-counter pain medications without relief. Her mother and maternal aunt experienced similar headaches. She reports some increased stressors and less sleep since recently starting college. Her past medical history is unremarkable, with no known medical conditions (except for the headaches), no surgeries, no chronic medications, and no drug allergies. She has never been sexually active and reports regular menses. She denies the use of tobacco, alcohol, and drugs. On physical exam, the patient appears comfortable and reports no headache at this time. Her entire exam, including neurological, is normal. Question If testing was performed on this patient for her headaches, which of the following test results best support this patient's likely diagnosis?

Correct answer: Normal magnetic resonance imaging (MRI) Explanation This patient is suffering from migraine headaches, common headaches classically described as throbbing in nature and accompanied by nausea (and possibly vomiting), photophobia (sensitivity to light), phonophobia (sensitivity to sounds), and possibly visual changes and aura. Migraines are more common in women than men. They are usually a chronic condition, beginning in adolescence or early adult years. Often, a family history is present. Stress, sleep deprivation, and various foods can be among many triggers for migraines. With a classic history and no alarm features of a more serious condition, no testing would be necessary. If testing was performed, this patient would be expected to have normal magnetic resonance imaging (MRI). Inflammation on temporal artery biopsy indicates temporal arteritis, which can cause a throbbing headache. Temporal arteritis is more likely to occur in an older person and is not associated with nausea and vomiting, nor does it occur episodically in nature like migraines. Patients with temporal arteritis may experience some visual symptoms, such as amaurosis fugax, diplopia, and vision loss. Polyspikes and slow wave pattern on electroencephalogram (EEG) indicate seizure activity. This patient's symptoms and headache pattern are not associated with any seizure disorder. If the patient had positive toxicology for cocaine, she could present with a nondescript headache and nausea or abdominal pain related to the vasoconstrictive properties of cocaine. In acute cocaine use, restlessness and mydriasis (dilation of the pupils bilaterally) may be observed. Nothing in this patient's history or physical suggests cocaine use as the cause of her headaches. Thickened sinus mucosa on computed tomography (CT) indicates chronic sinusitis, which can cause headaches. The headaches associated with chronic sinusitis can occur episodically but would likely be accompanied by nasal congestion, thick and/or discolored nasal discharge, sinus pressure, and possibly post-nasal drip with sore throat. Sinusitis headaches would not be expected to produce the episodic pattern described with this patient's migraines.

Question A 72-year-old man presents with the inability to comprehend spoken language. His speech is fluent and has a normal cadence and rhythm; however, when he talks, it is gibberish. He frequently substitutes one word for another. What is this phenomenon called?

Correct answer: Paraphasia Explanation Paraphasia is a type of aphasia. The substitution of a similar sounding word for another word is called paraphasia. With paraphasia, the words can also be jumbled. If a patient is able to hear things and repeat them, it is called echolalia. With echolalia, the patient does not understand what he has heard. This is also referred to as echophrasia. Alexia is a type of aphasia. An aphasia in which there is a problem with reading is called alexia. Alexia is word blindness or text blindness. Alexia is also called optical aphasia or visual aphasia. Apraxia refers to the condition in which a patient has difficulty performing motor acts, despite having the muscular capacity and coordination to do so. A patient with apraxia cannot execute the intended movement. A writing disturbance is called agraphia. There are various forms of agraphia. With absolute agraphia, even simple letters cannot be written. This is also referred to as literal agraphia.

Case Ico-delete Highlights A mother brings in her 6-year-old daughter because the girl's teacher is concerned about behaviors at school. The teacher has noticed the girl "staring off into space" frequently throughout the day; the teacher is able to get the girl's attention only occasionally. There are also periods when she appears to be talking to herself, but there is no sound coming from her mouth. The mother states that there are times when her daughter does not seem to be paying attention to what the mother is saying. Shortly after these episodes, the child engages in conversation without any problem, so the mother did not think the episodes were an issue. There is no concern about other abnormal behavior or discipline issues at home or at school. Question Ico-delete Highlights What is the most likely diagnosis?

Correct answer: Petit mal (absence) seizures Explanation The patient is probably experiencing petit mal (absence) seizures. Absence seizures are a form of generalized seizures seen in children; they usually cease by age 20. They are characterized by the abrupt onset of impaired consciousness and can be associated with enuresis or automatisms. Patients often appear as though they are staring off into space during the seizure, and the episodes often terminate as quickly as they began. These children are often categorized as "being in their own world," leading to a delay in diagnosis or a misdiagnosis of attention-deficit disorder. These types of seizures are diagnosed using an electroencephalogram (EEG), showing crests of bilaterally synchronous and symmetric 3 Hz spike-and-wave activity. Attention-deficit disorder (ADD) is a form of attention-deficit hyperactivity disorder (ADHD) that is characterized by inattentiveness, without the presence of restless or impulsive behavior. This is the most common emotional, cognitive, and behavioral disorder treated in children/adolescents. The diagnosis is made by careful clinical history, revealing a significant level of inattentiveness and distractibility, plus-or-minus impulsivity, and hyperactivity that is inappropriate for the developmental stage of the child. These patients often change activities frequently, have a difficult time with organizational skills, and are commonly caught daydreaming. The symptoms of ADD are usually pervasive, even though they may not occur in all settings. ADD/ADHD also does not involve any impairment of consciousness. Syncopal episodes are better known as fainting spells. These are episodes of diminished or complete loss of consciousness accompanied by flaccidity; they usually occur in relation to postural change, emotional stress, instrumentation, pain, straining, cardiac arrhythmias, or other physiologic disturbances. Associated signs/symptoms include pallor, sweating, nausea, and malaise. While this could be in the differential diagnosis for this patient, the cause of the syncopal episode would still need to be determined by conducting a battery of tests. In addition, this patient does not lose postural tone during her episodes; this can distinguish her presentation from true syncope. Narcolepsy is a condition that is associated with "sleep attacks" that are uncontrollable and occur in inappropriate and occasionally dangerous situations. Most of these attacks are brief. Associated symptoms can include cataplexy (a sudden loss of muscle tone), hypnagogic hallucinations (dream-like experiences while falling asleep), and sleep paralysis. Childhood disintegration disorder (CDD) is a rare syndrome on the autism spectrum. It is characterized by normal development until the age of 2 that is followed by a major loss of skills, both social and verbal. Motor skills are often preserved. The presentation is often very abrupt and very dramatic. This differs greatly from this patient's presentation; she has had a more gradual onset of symptoms that have almost gone unnoticed.

Question Ico-delete Highlights A 20-year-old woman presents with asthenia, difficulty in breathing, cyanosis of the extremities, and bilateral ptosis. On physical examination, the patient is in acute distress, but she is alert and oriented. She has difficulty holding her head upright and speaks in whispers. Vital signs are as follows: BP 110 / 80 mmHg, HR 94 bpm, RR 23 rpm temperature 36.6 °C (97.9 °F). On physical examination, there is bilateral ptosis and loss of ocular motility, tachypnea, distal cyanosis, and generalized weakness. A lab workup shows the following: Ht 44%, Hb 14 mg/dL, MCV 86 fl platelets 310,000 /mm3, SaO2 92%, pH 7.34, pO2 90 mmHg, pCO2 50 mmHg, HCO3- 29 mmol. Myasthenia gravis is suspected. An injection of edrophonium brings transient improvement in symptoms. What is most likely to induce a rapid response?

Correct answer: Plasmapheresis Explanation Myasthenia gravis is an autoimmune disorder of peripheral nerves in which autoantibodies to acetylcholine nicotine postsynaptic receptors at the myoneural junction are present. The management of myasthenia gravis revolves around 2 kinds of drugs: cholinesterase inhibitors, which enhance the action of acetylcholine on the motor endplates and thereby improve muscle strength, and medications that reduce the production of autoantibodies against the motor endplates. Intravenous immunoglobulin and plasmapheresis are useful in acute crises, during which a rapid reduction in circulating antibodies is sought. Immunoglobulin has similar effects to plasmapheresis and is easier to administer. However, it can take up to 2 weeks to act, it is expensive, and it is less effective than plasmapheresis. After stabilizing the patient's condition, more long-term management with prednisone and cholinesterase inhibitors is the treatment of choice. If the disease is refractory or the patient is intolerant to corticosteroids, oral immune suppressants such as azathioprine, cyclophosphamide, cyclosporine, or mycophenolate mofetil can be used. After an improvement in the patient's overall condition is obtained, the next step is to schedule a thymectomy. This procedure induces remission in about 60% of patients. However, because the thymus is already markedly involuted in this age group, thymectomy is not usually recommended in patients older than 60 years. The benefits of thymectomy usually take 6 to 12 months or longer to become apparent. Intravenous pulse prednisolone has no place in the treatment of myasthenia gravis. Anticholinesterase drugs are more useful in the chronic management of myasthenia. Use of very high doses can induce a myasthenic crisis, a complication characterized by cholinergic symptoms such as miosis, salivation, and abdominal cramping. This complication has become less common as other classes of drugs, particularly immune suppressants, have become available.

Case A 48-year-old man presents with a 5-year history of hand tremor. The tremor was initially mild and has progressed over the last year. It diminishes at rest, but it intensifies with intentional movements and upon emotional stress and fatigue. He is unable to write or drink from a cup during stressful situations. He never drinks alcohol, and he does not currently take any drug except metformin for non-insulin dependent diabetes mellitus. Family history is negative. Physical exam is unremarkable. His gait, speech, and posture are normal. Question What is the best first-line agent for treatment of this condition?

Correct answer: Propranolol Explanation Essential tremor is the most common idiopathic movement disorder, affecting 5-10 million people in the United States. Key features include a postural tremor that affects either the head or limbs. The tremor may improve with alcohol and be exacerbated by stress. It is usually bilateral, exacerbated by movement and absent at reset. Primidone or propranolol is the best initial agent for treatment of essential tremor. Relative contraindications for the use of propranolol are asthma, congestive heart failure, diabetes mellitus, and atrioventricular block. Primidone should be started at low doses because it is sedating. Second-line agents for the treatment of essential treatment include gabapentin or topiramate. Nimodipine, alprazolam, amitriptyline, and theophylline are not used in the management of essential tremor.

Case A 62-year-old woman presents with excruciating pain, burning, and swelling in her left forearm and wrist. She reports that symptoms initially began with a fracture 4 months ago. The fracture was casted, and the patient was told it had healed well with cast removal at 8 weeks. She is frustrated because her symptoms have persisted and worsened, rather than improving, as she was told they would. The patient has continued to use a sling and limit use of the left arm so she doesn't worsen her condition. She is unable to wear a jacket or long sleeves; even the fabric touching her skin causes pain. She denies fevers, pain in other areas, and new trauma. She denies polyuria and polydipsia. Her past medical history is unremarkable; she is a menopausal woman, with no known medical conditions, no history of surgeries, no regular medications and no allergies. She lives with her husband and is a homemaker. She denies drug, alcohol, and tobacco use. On physical exam, she is a small, thin, pleasant woman, and she is fully oriented. Vitals are normal. No gait or balance abnormalities are noted when she walks or gets onto the exam table. Her left forearm has some mild edema and erythema, as well as tenderness with even light touch. Distribution of findings includes the region from the elbow to wrist, both the anterior and posterior surfaces. Left wrist strength and range of motion are decreased compared to the right. Distal pulses, capillary refill, and reflexes are normal. The remainder of the exam, including mental status, is normal. Question What is an expected diagnostic test finding, given this patient's likely diagnosis?

Correct answer: Radiograph showing callus formation Explanation This patient's diagnosis is most likely a complex regional pain syndrome (CRPS). CRPS most often develops after a minor trauma and classic characteristics include pain out of proportion with findings and history, allodynia (pain sensation with normally non-painful stimuli), and motor and sensory disturbances in the affected extremity. The mechanism for CRPS development is not well understood. Prolonged immobilization following injury is a risk factor for development of CRPS. Diagnosis is clinical and testing is done to rule out other disorders. In this patient, we would expect a radiograph (X-ray) showing callus formation, but otherwise normal. Decreased perfusion on Doppler ultrasound is a finding that may be present with a deep venous thrombosis (DVT). DVT is rare in the upper extremities and is not associated with the hyperesthesia and allodynia shown in this patient. Elevated hemoglobin A1C suggests diabetes, which is associated with neuropathy in the extremities. However, the clinical course of diabetic neuropathy is typically gradual, symmetrical, and starts in the lower extremities. Elevated serum uric acid may be seen in erythema, swelling in the affected joint(s), and gout (a painful inflammatory condition which may cause acute pain). Gout tends to be more acute, and typically affect joints (not the forearm, as with this patient). Positive blood culture may indicate a severe infection, such as sepsis or possibly cellulitis. Without fevers and having only localized symptoms, this patient clearly is not septic. Cellulitis, which commonly presents with erythema, edema, and tenderness in an extremity, may be considered on the differential for this patient. Cellulitis tends to occur more acutely. This patient had a normal 8-week exam with symptoms of CRPS but no findings of cellulitis at that time.

Case A 42-year-old man presents following an episode of loss of consciousness. The patient was in excellent health until the evening of admission; his wife found him unconscious on the concrete deck surrounding their swimming pool. His swim trunks were fully wet, and there was vomitus on the deck near him. He regained consciousness in the ambulance, at which time he was confused and complained of severe generalized headache. Past medical history was positive for well-treated hypertension, 2 packs per day smoking history of 24 years duration, occasional social drinking, and a 2-month history of intermittent right ocular pain. Physical exam reveals a temperature of 98.5 F by mouth, blood pressure of 190/110 mm Hg, pulse rate of 116/min and regular, with respirations 22/min and unlabored. He is a well-developed, well-nourished, middle-aged man. He is awake, but drowsy, and he complains of severe headache. There is mild nuchal rigidity. Fundi are clear. The rest of the neurological exam is normal. There are no signs of trauma. Labs reveal a white blood cell count of 11,500/mm3, with a normal differential. The chemistry profile is normal. Electrocardiogram reveals prolonged QRS complexes, prolonged QT interval, and deeply inverted and symmetric T waves. Unenhanced CT of the head reveals a large collection of blood in the right Sylvian fissure and lesser amounts in the basal cisterns. There is no intracerebral hemorrhage or ventriculomegaly. The patient is identified as having a ruptured intracranial saccular (berry) aneurysm. Question What potential complication is the most problematic?

Correct answer: Rebleed or vasospasm at site of aneurysmal bleed Explanation In the International Cooperative Study of 1990, vasospasm and rebleed were noted as being the leading causes of morbidity and mortality in those who survived the initial bleed (1). Rebleed is the most feared complication, and a rebleed may occur at any time during the first 3 weeks; however, it is much more likely to occur during the earlier phases, especially during the first 24 hours. Vasospasm remains the main cause of morbidity and mortality following aneurysm rupture and subarachnoid hemorrhage (2). Seizures occur in 10-15% of cases; they are usually generalized and brief, and they do not seem to affect the prognosis (1). While seizures are relatively uncommon at the onset of aneurysmal rupture, phenytoin is often given as prophylactic therapy because a seizure may cause rebleeding (2) Hyponatremia and hypovolemia occur in over 50% of patients in the days following the initial bleed (1). While it is crucial to anticipate and treat this early, it is generally not that hard to correct. Hypovolemia increases the risk of ischemic infarction, especially when there is significant vasospasm. Tentorial herniation may occur from obstructive hydrocephalus, intracerebral bleeds, severe cerebral edema around infarcts due to ischemia from vasospasm, as well as multifocal or generalized brain edema; however, the incidence and consequences of tentorial herniation do not approach those of either rebleed or vasospasm (2). Hydrocephalus may develop acutely if a large amount of blood ruptures into the basal subarachnoid space and enters the foramina of Luschka and Magendie (1). Intraventricular blood is the major factor in the subsequent development of obstructive hydrocephalus. Acute hydrocephalus can cause stupor and coma. More often, a subacute hydrocephalus develops over a few days or a few weeks, and it causes a progressive drowsiness and slowed mentation with urinary incontinence (2). A chronic hydrocephalus may develop over weeks to months, and it may manifest as gait difficulty, urinary incontinence, mental fall away, or a combination of these. Hydrocephalus can usually be effectively treated with temporary ventriculostomies or more permanent ventriculoperitoneal shunts; how it is treated depends on the course and clinical situation.

Case A 62-year-old man presents with vision problems and difficulty swallowing. Over the past week, he has had a constellation of symptoms, beginning with numbness and tingling in his feet that progressed to weakness that now affects both lower and upper extremities. Within the past day, he has started to notice difficulty swallowing and double vision. He also feels it is difficult for him to take a big breath. His past medical history is noncontributory except that he had a brief respiratory illness about 2 weeks ago. Exam reveals bilateral absence of patellar and ulnar reflexes. Question Based on the most likely diagnosis, with proper treatment, what is the expected prognosis for this patient?

Correct answer: Recovery with little to no motor or sensory deficits Explanation The most likely diagnosis is Guillain-Barré syndrome (GBS). The vast majority of patients (>80%) with Guillain-Barré syndrome (GBS) experience either a full recovery or recovery with minor motor or sensory deficits. The most common residual difficulties are weakness of the lower leg muscles, weakness of facial muscles, or numbness of the feet and toes. In addition to supportive care, plasmapheresis and/or intravenous immune globulin are often used to reduce the duration and severity of symptoms once the diagnosis is recognized. Symmetrical ascending paralysis as described in this patient is indicative of Guillain-Barré syndrome (GBS). The cause of GBS is unknown, but it is generally thought to be an inflammatory autoimmune process. More than 50% of patients with GBS report an antecedent illness. The antibodies produced in response to antigens present in the infectious agent are thought to cross-react with components of human neurons, prompting an acute post-infectious demyelinating process. Lumbar puncture characteristically reveals elevated CSF protein content. Other results are normal, although the white blood cell count may be somewhat elevated. Decreased CSF glucose and increased polymorphonuclear cell counts are seen in acute bacterial meningitis. Decreased CSF glucose and elevated CSF lymphocyte counts are commonly seen with meningitis caused by fungi. Viral meningitis usually presents with elevated lymphocyte counts, normal CSF glucose, and normal or slightly elevated CSF protein levels. Recovery with significant motor or sensory deficits is rare, as is the development of severe chronic pain. Progressive paralysis and respiratory failure may occur if untreated but are unlikely if supportive care is provided. Waxing and waning polyarticular weakness and paresthesias are uncommon.

Case A 62-year-old right-handed man presents with symptoms and signs consistent with a stroke. His past medical history is significant for smoking a pack a day for 30 years, hypertension, and a brother who had a stroke at the age of 56 years. On physical exam, the patient is found to be alert and orientated to place and person only. His temperature is 98 Fahrenheit (36 Celsius), pulse is 100/minute, respirations of 20/minute, and blood pressure of 192/95mmHg. He has right-sided hemiparesis and is aphasic. There are no other significant signs. Question At this time, what should immediate management include?

Correct answer: Refer for emergent CT scan Explanation The correct answer is to refer for emergent head CT. The purpose of the head CT is to rule out intracranial bleeding, which would be a contraindication to thrombolytic therapy for stroke. Maintaining adequate tissue oxygenation is an important component of the emergency management of stroke. Hypoxia leads to anaerobic metabolism and depletion of energy stores, increasing brain insult. Although there is no reason to give supplemental oxygen, the potential need for oxygen should be assessed using pulse oximetry or an arterial blood gas measurement. Heparin and other antithrombotic drugs must be used with caution, and only after a baseline head CT scan rules out an intracerebral bleed. Corticosteroids are not an indicated in the management of cerebral edema and increased intracranial pressure after stroke. Optimizing cardiac output is a high priority in the hours immediately after a stroke. Cerebral hypoperfusion may occur if the blood pressure is dropped further; therefore, aggressive management with an antihypertensive is not indicated unless the systolic blood pressure is greater than 220mm Hg. Hypovolemia can exacerbate cerebral hypoperfusion, so there is no need to restrict oral fluid intake.

A 42-year-old woman presents with a 1-month history of severe, worsening pain in her right foot, ankle, and lower leg. The pain is constant and burning. She reports some initial swelling and warmth in the leg, which has lessened and now her right lower extremity is always cool to the touch. Hair growth has dramatically decreased on her right leg. She has tried multiple over-the-counter pain medications and topical analgesics with no relief. She denies trauma to the affected limb and reports her symptoms just "came out of the blue". Her sleep is poor, and she reports fatigue secondary to unrelenting pain, but she denies any other symptoms. Past medical history: Medical conditions: None Medications: No chronic medications; multiple over-the-counter pain treatments Allergies: None Surgeries: Right knee ACL repair 4 months ago; bilateral tubal ligation at age 32 Family history: Unremarkable Social history: Denies alcohol, tobacco, and drugs. She lives at home with her husband and children, and she works as a secretary. The patient is thin, with normal vital signs. On neurologic exam, her right lower calf, ankle and foot appear to be hypersensitive to light touch stimulus (cotton swab), compared to the left. The same region, which spans several dermatomes, exhibits decreased sensation to pinprick and decreased range of motion. The skin of the right lower extremity is cool; there are no lesions, rash, edema, or erythema. Ridges are noted on the right toenails, but not the left. The rest of her physical exam is normal. Blood count, glycohemoglobin, metabolic panel, D-dimer, as well as the x-ray and ultrasound of the entire right lower extremity are normal. Question What would be the most appropriate intervention in this case?

Correct answer: Refer to a pain specialist Explanation This patient most likely has complex regional pain syndrome (CRPS), and the best approach is to refer to a pain specialist for immediate treatment. CRPS is indeed a complex syndrome often occurring in a limb; patients exhibit vasomotor and neurologic symptoms out of proportion with objective findings. It often occurs following some type of event, such as a surgery (note the patient's recent ipsilateral knee surgery), fracture, stroke, or myocardial infarction. Most of the time, there is no nerve lesion. CRPS is classified into 3 stages, and symptoms can vary depending on the stage. It is characterized by persistent burning and/or throbbing pain in an extremity. Swelling, redness, and increased hair/nail growth occur in stage I; it may progress to cool, pale skin and even osteoporosis in the latter 2 stages. Allodynia, the phenomenon in which a normally painless stimulus produces significant pain response, is a common finding. No single exam finding or test is diagnostic of CRPS, which is a clinical diagnosis. Likewise, no single treatment approach is recommended. A variety of pain treatments, such as antidepressants, anticonvulsants, corticosteroids, alpha 2 agonists, topical analgesics, and nerve stimulators, may be used. If this patient were thought to have a deep venous thrombosis (DVT) as the cause of her unilateral leg and foot pain, it may be appropriate to initiate an anticoagulation regimen. However, the patient's physical exam and test results do not support DVT as her diagnosis. CRPS is a chronic, and potentially recurring, condition. Treatment recommendations caution again "medications that could cause dependency" (Bailey pg.513), so it would be inappropriate to prescribe long-acting opioid pain patch for daily use until symptoms subside. The side effect profile and dependency risk of opioids are not favorable for primary treatment of CRPS, but opioids do have a place as an adjunct, within the context of comprehensive pain management. Without intervention in the early stages, CRPS tends to worsen over time, leading to significant osteoporosis and decreased quality of life, and become more resistant to treatment. It would be inappropriate to reassure the patient that her tests are normal and employ "watchful waiting"; doing so would put her at risk for progression of the CRPS. A provider may consider conversion disorder, factitious disorder or drug seeking as a cause of the patient's reported pain seemingly out of proportion to her objective findings. However, this patient should not be referred to a psychiatric provider for drug-seeking behaviors. Her condition is a recognized, treatable disorder. Some consideration for psychiatric and counseling conditions is reasonable for patients with long-standing CRPS. Almost all sufferers of CRPS eventually develop insomnia, anxiety, and depression as a result of their condition. If the patient's painful limb symptoms were caused by compartment syndrome, it would be wise to refer her to urgent fasciotomy. Although compartment syndrome can produce severe pain out of proportion with findings, the skin is often taut on exam and there is a history of trauma to the limb.

Case Ico-delete Highlights A 34-year-old pregnant woman presents with what she describes as "tingling in her right arm"; she is at her 34th week of gestation. This is her 2nd pregnancy. It comes on along her palm, and it has been increasing in severity. She also notices it more at night and while attempting to carry her shopping bags. Of late, her pain has been so bad that she has been losing sleep. Examination reveals a positive Phalen's test; there is no weakness or atrophy of the thenar muscles. She is not a known diabetic or hypertensive, and she is otherwise healthy. Her thyroid function is normal. Her primary care physician advised treating with NSAIDs and vitamin supplements, neither of which have worked after 3 months of regular use. Question What is the next best step in management?

Correct answer: Rest and neutral splinting Explanation Rest and neutral splinting is the correct answer. Pregnancy predisposes women to carpal tunnel syndrome, especially during the 3rd trimester, probably due to changes in fluid composition and balance. Resolution is typically seen following delivery. Rest and placing the hand in a neutral splint are sufficient in the majority of patients, and doing so affords relief until delivery. Surgical release is incorrect. In the setting of pregnancy, carpal tunnel syndrome is reversible, making surgical release unnecessary. Furthermore, surgery is not indicated in any patient with carpal tunnel syndrome without a trial of conservative treatment, except when there is evidence of severe nerve entrapment, such as muscle atrophy, demonstrable muscle weakness, and/or nerve conduction studies showing impaired conduction. Oral corticosteroids are incorrect. Rest is sufficient, and corticosteroids are to be avoided in pregnancy except in very specific conditions for the benefit of the fetus. Splinting in flexion is incorrect. Splinting is done in a neutral wrist position. Flexion exacerbates carpal tunnel symptoms Corticosteroid injection is incorrect. It is a possible option if rest and splinting in neutral position fails.

Case A 42-year-old man presents to the emergency department with a severe headache. He has been getting several of these headaches recently and has tried all over-the-counter pain relievers and headache medicines with no relief. His current headache started 15 minutes ago. He describes the pain as located next to and behind his left eye and "stabbing/excruciating" in nature. He feels like his left eye tears up profusely with these headaches. He reports he has been healthy otherwise, with no chronic medical conditions, no history of surgery, no medications, and no drug allergies. He denies recent stressors that may have caused his headaches. On physical exam, the patient appears slightly agitated and appears uncomfortable. His left eye's conjunctiva is mildly injected, and lacrimation is noted. His right eye is normal. Cranial nerves II-VII are intact, although the patient expresses discomfort when the light is shown in his left eye. Speech, gait, coordination, and reflexes are all normal. The remainder of his exam is normal. Head MRI is performed and reported as normal. Question Which of the following aspects of patient history would be most consistent with this patient's suspected condition?

Correct answer: Rhinorrhea associated with headaches Explanation This patient is suffering from cluster headaches. Cluster headaches are classified as trigeminal autonomic cephalalgias and are most common in men. Cluster headaches may be triggered by alcohol, histamine, and nitroglycerin, but often no cause is identified. There may be a genetic component. Classic presentation of cluster headache includes episodic, unilateral, severe headache, usually located in the temporal and/or orbital region, along with associated lacrimation, injection, eyelid edema, miosis or ptosis of the ipsilateral eye, and nasal congestion and rhinorrhea associated with the headaches. Imaging studies, such as head MRI, should be performed to rule out other primary causes of headache, such as vascular malformation, neoplasm, and infection. A head injury prior to the onset of headaches is not associated with cluster headaches. Post-concussion headaches tend to have longer duration, but less severe intensity compared to cluster headaches. Other clues to a post-concussion headache may be memory problems, "fogginess," and nausea, and/or vomiting. The eye symptoms and signs this patient displayed are not consistent with a prior head injury. Periodic episodes of aphasia and gait disturbances would be concerning for a severe disorder, ranging from vascular, infectious, neurodegenerative, and neoplastic in etiology. Patients with cluster headaches would not be expected to have these types of neurologic symptoms. Recent exposure to outdoor pollens would be expected in a patient experiencing allergy-related headaches. A patient with allergy-related headaches would be expected to have much less intense severity and bilateral eye and nasal symptoms. 2 to 3-day duration of each headache can be seen with tension-type headaches and migraines. However, cluster headaches are noted to last 15-180 minutes of duration.

Case A 67-year-old man with a 10-year history of insulin-dependent diabetes mellitus awakes one morning with weakness of the right leg. The patient has difficulty walking and states that he tripped while walking up the stairs to his home. He is also experiencing a funny sensation along the lateral aspect of his right leg. He denies arm weakness, speech difficulty, back pain, bladder or bowel dysfunction, and other neurologic symptoms. The general examination is normal. Neurologic exam is significant for 0/5 weakness of the right peronei, tibialis anterior, and extensor hallucis longus muscles. The other muscles of the right leg (iliopsoas, quadriceps, hamstrings, tibialis posterior, and gastrocnemius) are 5/5. There is no weakness of the left leg or either arm. Sensation to pain, temperature, and light touch is diminished over the lateral aspect of the right leg and medial aspect of the right foot. Reflexes are 2+ in the arms, 1+ at the knees, and 0 at the ankles; the plantar responses are flexor. He walks with a right foot drop. The remainder of the neurologic exam, including palpation of the spine, is unremarkable. Question What is the most likely diagnosis?

Correct answer: Right common peroneal diabetic mononeuropathy Explanation This man presents with an acute right common peroneal neuropathy, which is a common presentation of diabetic mononeuropathy. There is weakness in all the muscles supplied by the common peroneal nerve (peroneal longus and brevis, tibialis anterior, extensor hallucis longus) and sensory impairment in the distribution of the superficial and deep peroneal nerves (anterolateral leg and dorsomedial surface of the foot). A lumbosacral plexopathy is unlikely because of the absence of sensorimotor signs in the distribution of multiple nerves (sciatic, tibial, femoral). An L5-radiculopathy would also affect the Hamstring muscles. Infarction of the anterior spinal cord would produce paraplegia and a dermatomal sensory level. Anterior cerebral stroke usually produces a monoparesis of the entire lower extremity with associated upper motor neuron findings (hyperreflexia, Babinski sign). Diabetic mononeuropathy is associated with ischemia in the vasa vasorum of affected peripheral nerves; recovery is dependent on adequate glucose control and occurs over several months.

Case A 28-year-old man presents with excessive daytime sleepiness and episodes of cataplexy. He has been having disturbed sleep at night, including dreams so lifelike that they seem like reality and fleeting inability to move when falling asleep. He does not snore. His uncle also has a similar problem. On examination, his weight is 190 lb and his height is 5'8"; his pulse is 72/min and his BP is 130/84 mm Hg. Question What is the most useful non-pharmacologic management of this condition?

Correct answer: Schedule routine napping Explanation The correct answer is schedule routine napping. The history suggests the diagnosis of narcolepsy. Besides the daytime sleepiness and cataplexy, the patient is experiencing other symptoms of narcolepsy: hypnagogic hallucinations and sleep paralysis. The scheduling of short naps (15-20 minutes) 2-3 times per day can help control excessive daytime sleepiness and improve alertness. This is the best non-pharmacological measure for this condition. Modafinil is the first-line medication for the treatment of narcolepsy. It has a low abuse potential, and, unlike other stimulants, it is not associated with rebound hypersomnolence. The exact mechanism of action of modafinil is unknown. Modafinil may indirectly increase wakefulness partly through inhibition of GABA release via serotonergic mechanisms, or indirectly on dopaminergic stimulation. It also stimulates norepinephrine inhibition of the sleep-promoting nucleus, the ventrolateral preoptic nucleus. Usual doses of modafinil are 200 mg/d or 400 mg/d, but higher doses may be required in some patients. Note that the question specifically asks for non-pharmacologic treatment. Weight loss and continuous positive airway pressure (CPAP) are non-pharmacologic measures for treatment of obstructive sleep apnea. The patient does not have symptoms of obstructed breathing, the most notable of which is nighttime snoring, making this diagnosis less likely. Avoiding intake of stimulants before going to bed may improve nighttime sleep, but it is not a specific treatment for the daytime sleepiness due to narcolepsy.

Case A patient presents with bilateral spastic paresis of his legs. His gait is noticeably stiff and his legs are advanced slowly. He uses short steps and crosses his thighs in front of the other as he advances. Question What type of gait does he have?

Correct answer: Scissors gait Explanation Scissors gait is apparent when each leg is advanced slowly and the thighs tend to cross each other. The gait is stiff and the steps are short. This is associated with bilateral spastic paresis of the legs. Steppage gait is associated with a foot drop, often secondary to lower neuron disease. The patient drags their foot, or lifts the knee high and slaps the foot against the ground. This gait may be unilateral or bilateral. Sensory ataxia gait is unsteady and wide-based. The feet are thrown forward, out, and then down. There is a distinct two-slap sound, secondary to the heel striking first, then the forefoot. This is associated with polyneuropathy or posterior column damage. The gait characterized by cerebellar ataxia is associated with a disease of the cerebellum or affiliated tracts. The gait is staggering, unsteady, and wide-based. The patient has difficulty with turns, and cannot stand steady when feet are together and eyes closed (Romberg's sign). A myopathic gait is where there is weakness on one side. This displays a drop in the pelvic area on the contralateral side of the pelvis while walking (Trendelenburg sign). When there is a bilateral weakness, it will display as a dropping of the pelvic girdle on both sides of the body while walking that results in a waddle. This is seen in patients who are afflicted with myopathies, such as muscular dystrophy.

Case A 47-year-old man presents with numbness, tingling, and pain anterior and to the left side of his left thigh. Pain is provoked even with light touch, and tingling continues for several minutes after the touch. Symptoms started a couple of months ago and worsen when he wears a belt and walks down slopes and stairs, as well as after prolonged standing. Symptoms are relieved when he puts a pillow between his thighs. Patient also assumes hunched posture while standing to avoid unpleasant sensory symptoms. His BMI is 40, and he has a protruding, pendulous abdomen. Question What else do you expect to find on examination?

Correct answer: Sensory loss in anterolateral thigh down to left upper knee Explanation Your patient most probably has entrapment syndrome caused by the compression of the lateral femoral cutaneous nerve and suffers from the condition called meralgia paresthetica. Pendulous abdomen and tight belt compress the inguinal ligament downward and onto the nerve. The lateral femoral cutaneous nerve is a pure sensory nerve, conducting sensory signals from the region of anterolateral thigh down to the upper knee. On examination, you can find decreased sensitivity to pinprick together with a hyperpathic reaction to touch. This angulation of the nerve is further exaggerated with extension of the thigh and relaxed with flexion. Foot drop is a sign of common peroneal nerve entrapment. Loss of foot inversion in plantar flexion is a sign of L5 radiculopathy, in which posterior tibialis function is disturbed. The quadriceps, patellar reflex, or knee-jerk is a deep tendon reflex that can be lost in the case of the lesion of L2, L3, and L4 root. This myotatic reflex tests motor function. Your patient has no signs and symptoms of radiculopathy or motor function disturbance.

Question The mechanism of action for sumatriptan in the treatment of migraine headaches involves which of the following neurotransmitters?

Correct answer: Serotonin (5HT1) Explanation The headache and associative symptoms of migraine are postulated to result from the spontaneous activation of the trigeminovascular system. The peripheral fibers of the trigeminal nerve (ophthalmic division) supply innervation to the supratentorial meninges (dura mater, pia mater) and the intracranial blood vessels (arteries, veins, and venous sinuses). When activated, the peripheral axons of these bipolar neurons release vasoactive peptides (e.g. substance P, calcitonin gene-related protein, prostaglandins) adjacent to craniovascular structures, resulting in sterile inflammation, vasodilation, and pain. Central stimulation of the trigeminovascular system disrupts activity in the brainstem and hypothalamus, which is likely responsible for associated nausea, vomiting, photophobia, and phonophobia of migraine. Treatment of migraine can be abortive or prophylactic. The trigeminovascular neurons are inhibited by serotonin 5HT1D- receptors, which are located both centrally and peripherally. Activation of these receptors aborts acute migraine headache and associative symptoms. Sumatriptan and dihydroergotamine (DHE) are specific 5HT1D- agonists used as abortive agents for migraine. Other medications used for acute migraine include acetaminophen, aspirin, naproxen sodium, ketorolac, caffeine, and butalbital. These drugs presumably act by reducing inflammation and promoting vasoconstriction within and around cranial blood vessels. Migraine prophylaxis is appropriate in patients who have frequent attacks (more than 3 per month) that interfere with social and occupational functioning. Prophylactic agents include tricyclic antidepressants (e.g. amitriptyline), β-blockers (e.g. propranolol, atenolol, nadolol), verapamil, and valproic acid.

Case An 11-year-old boy presents after waking with a new onset of left-sided weakness. He is also experiencing slurring of speech, double vision, dizziness, neck pain, and headache. His cognition is normal. He had been playing hockey on the evening prior to presentation and was struck by a puck on his neck. He has no significant past medical history. On physical examination, blood pressure is 110/60 mmHg, pulse 72/minute, and respirations 16/minute. His heart sounds and breath sounds are normal. The power in the left upper and lower limbs is 3/5, with sensory loss on the left side of the face and arms. Question What is the most likely cause of this boy's symptoms?

Correct answer: Stroke due to carotid dissection Explanation Dissection of the craniocervical arteries is the most common non-atherosclerotic cause of strokes in young people. He probably has experienced a stroke due to carotid dissection. Blunt trauma to the neck, such as being struck with a hockey puck, can cause clotting at the site of the dissection, resulting in an embolic stroke in the area perfused by the middle cerebral artery. Carotid angiography in such patients reveals internal carotid arterial occlusion. This type of stroke is not uncommon in childhood, and it may also occur following intraoral trauma, chiropractic manipulation, and cervical spinal injury following motor vehicle accidents. It usually presents with neurological deficits preceded by headache and neck pain. The patient may present with dizziness, dysarthria, diplopia, nystagmus, numbness, and features of Horner's syndrome. Diagnosis is by imaging methods such as Magnetic Resonance Imaging (MRI), Computed Tomography (CT), or Magnetic Resonance Angiography (MRA). Initial treatment is with anticoagulants. An injury of the cervical cord should cause bilateral symptoms and possibly respiratory compromise. Headache and neck pain would also not typically be present. A lesion of the brachial plexus should not cause weakness of the leg. Transverse myelitis usually causes bilateral symptoms, and a specific sensory level deficit is present. Conversion reaction is a psychopathological condition characterized by the presence of bodily symptoms that have no discernable physical cause. Conversion reaction is a diagnosis of exclusion and would not be the most likely explanation of these findings. Another risk factor for stroke in childhood is sickle cell disease. The incidence of stroke among children who have sickle cell disease is estimated to be as high as 6%. Transfusion to decrease the percentage of hemoglobin S has been found to lower that risk significantly. Hemorrhagic stroke in childhood may be due to arteriovenous malformations, arterial aneurysm, or cavernomas. Underlying hematologic conditions, including coagulopathy (e.g., hemophilia) or thrombocytopenia (e.g., idiopathic thrombocytopenic purpura), also may be a cause. Strokes have also been associated with cocaine use or trauma. Despite careful and complete evaluation, the etiology in some cases remains cryptogenic.

Case A 9-year-old boy presents to a neurologist's office accompanied by his mother. She reports that he has had 3 episodes of what she thinks are seizures. She describes these episodes in detail to the neurologist. Question What would lead the neurologist to consider the diagnosis of tonic-clonic seizures?

Correct answer: Sudden loss of consciousness with convulsions and confusion that lasts about 15 minutes each time, then resolves spontaneously Explanation The correct answer is the type of seizure that involves a sudden loss of consciousness with convulsions and confusion that lasts about 15 minutes each time, then resolves spontaneously. These types of seizures are also known as generalized convulsive seizures, or tonic-clonic seizures. They are associated with loss of consciousness, convulsions, and muscle rigidity lasting minutes to hours. Sudden loss of consciousness with convulsions that do not cease without medical attention and recur regularly is not the correct answer because this description is closest to status epilepticus given the recurrent and severe nature. Epilepsy is a condition during which a person has recurrent seizures that typically need medical attention. Any seizure that lasts longer than 5 minutes should be treated medically. Episodes of starting into space that last only a few seconds occur in patients with absence seizures. Absence seizures are a type of generalized seizure known as a generalized non-convulsive seizure. Tonic-clonic seizures involve loss of consciousness and some sort of convulsion, so this type of seizure is not a tonic-clonic seizure. Alternating jerking of each of his extremities while maintaining consciousness in episodes that last several minutes is not the correct answer, as this description closest describes simple partial seizures. Patients with simple partial seizures do not experience loss of consciousness. Episodes of blank staring that last a few minutes, during which he walks the same pattern in the room and does not remember it afterwards, is not the correct answer, as this describes a complex partial seizure. This type of seizure includes impairment of awareness and some form of automatism, such as lip smacking, chewing, walking, fidgeting, or another involuntary, repetitive coordinated movement.

Case A consulting physician asks for a follow-up on a 35-year-old woman being treated for an acute migraine headache. Upon interview, she states that the physician gave her an injection about 20 minutes ago; she now has significant chest pain. Question What medication did the patient most likely receive?

Correct answer: Sumatriptan Explanation The clinical picture is suggestive of a side effect of sumatriptan. Sumatriptan is commonly used for treatment of acute migraine attacks and can cause coronary vasospasms in 1-5% of patients. The vasospasms can be perceived as chest pain or discomfort. Promethazine has sedative and antiemetic properties; it does not cause chest discomfort as a side effect. Metoclopramide is commonly used to treat nausea and vomiting and does not cause chest discomfort as a side effect. Diclofenac is an NSAID and does not cause chest discomfort. Cyclizine is an antiemetic used for the treatment of nausea and vomiting; it does not cause chest discomfort as a side effect.

Case Ico-delete Highlights A 45-year-old African-American man with no significant past medical history presents with a 1-hour history of left retroorbital headache. The headache was of a sudden onset and began upon waking that morning. It is described as excruciating, stabbing, sharp, and lancinating; it is rated as severe in intensity. He denies any preceding infections, nausea, vomiting, photophobia, or osmophobia; he also denies fever, chills, stiff neck, focal weakness, numbness, tingling, vision, hearing, gait, or speech changes. He recalls a similar episode several months ago; it lasted about a week, and it dissipated without complications. His physical exam is remarkable for painful distress, lacrimation with conjunctival injection, nasal congestion, rhinorrhea, left ocular miosis, and left forehead diaphoretic flushing. Question Ico-delete Highlights In the acute setting, which pharmacologic agent may have the most potential benefit for this patient?

Correct answer: Sumatriptan Explanation The correct response is sumatriptan. This patient's most likely diagnosis is most likely a cluster headache. Pharmacologic management of cluster headache may be divided into abortive/symptomatic and preventive/prophylactic strategies. Abortive agents are used to stop or reduce the severity of an acute attack, and include oxygen, triptans, ergot alkaloids, and anesthetics. Inhalation of high-flow concentrated oxygen is extremely effective for aborting attacks. 5-Hydroxytryptamine-1 (5-HT1) receptor agonists, such as triptans or ergot alkaloids with metoclopramide, are often the first line of treatment. Stimulation of 5-HT1 receptors produces a direct vasoconstrictive effect and may abort the attack. Subcutaneous injection of sumatriptan can be effective, in large part because of the rapidity of onset. Studies have indicated that intranasal administration is more effective than placebo but not as effective as injections. Prophylactic agents are used to reduce the frequency and intensity of individual headache exacerbations. Preventive and prophylactic medications include calcium channel blockers, mood stabilizers, and anticonvulsants. Verpamil is the most effective calcium channel blocker for prophylaxis. It inhibits calcium ions from entering slow channels, select voltage-sensitive areas, or vascular smooth muscle, thereby producing vasodilation and preventing the initial vasoconstrictive phase of cluster headaches. It can be combined with ergotamine or lithium. Preliminary evidence suggests that prophylactic lithium may interfere with substance P and vasoactive intestinal peptide (VIP)-induced arterial relaxation. Anticonvulsants such as Divalproex and Topiramate are preventative medications whose mechanism of action may involve regulation of central sensitization. Prednisone is very effective for aborting the cluster headache cycle or providing intermediate prophylaxis as bridging therapy between acute and prophylactic agents. It is effective for treatment that does not respond to lithium.

Question What anticonvulsant is considered most effective for primary generalized seizures?

Correct answer: Valproic acid Explanation The correct response is valproic acid. A seizure is defined as a discrete neurologic disorder in which there is excessive and synchronous discharge of cortical neurons. Epilepsy is a syndrome characterized by recurrent, unprovoked seizures. A seizure may be classified as generalized or partial. In a generalized seizure, there is bilateral cerebral hemisphere disturbance; in partial seizures, the abnormal neuronal discharge is limited to a specific brain area (e.g., right anterior temporal lobe). Generalized seizures may be primary (bilateral brain involvement at seizure onset) or secondary generalized (seizure starts in a focal brain area, then spreads to encompasses both cerebral hemispheres). Valproic acid is usually the first-line medication for primary generalized seizures. It has a wide spectrum of action and is effective in most seizure types, including myoclonic seizures. It is not, however, recommended as the initial treatment in women of childbearing age because of its possible teratogenic nature. Phenytoin and phenobarbital may also have a role in the treatment of primary generalized seizures, but valproic acid is superior. Phenytoin, phenobarbital, carbamazepine, and gabapentin have efficacy in partial and secondary generalized seizures.

Case Ico-delete Highlights A 20-year-old female college student presents to the student health center with a 1-week history of daily headaches. She has no significant past medical history. Upon further questioning, she admits to headaches occasionally over the past 2 years, but it is lasting longer this time. She classifies the pain as a 4/10 that is generalized, but is worse in the back of the head. The headaches are not debilitating or throbbing, and she describes it as an annoying pain. Acetaminophen helps some, but does not make the headache go away completely. On physical exam, there are no neurologic deficits, vision is 20/20 uncorrected, and vital signs are within normal limits. She does note tenderness upon palpation of the muscles of the head, neck, and shoulders. Question What is the most likely diagnosis?

Correct answer: Tension headache Explanation Tension headache is the most common type of primary headache disorder. Common complaints include: pericranial tenderness, poor concentration, and daily headaches (often vice-like or tight). These can be exacerbated by emotional stress, fatigue, noise, or glare. Cluster headaches affects mostly middle-aged men and typically present with unilateral periorbital pain. Migraine headaches usually present with a lateralized throbbing pain that occurs episodically following its onset in adolescence or early adult life. These headaches can be associated with nausea, vomiting, anorexia, phonophobia, photophobia, and blurred vision. Giant cell arteritis affects mostly elderly patients and can cause headaches that are preceded by myalgia, malaise, anorexia, and weight loss. Loss of vision is a common manifestation. Intracranial masses can also cause headaches that can range from mild to severe. Often these types of headaches are accompanied by neurologic deficits

Case Ico-delete Highlights A 23-year-old woman presents due to a headache that appears to be bilateral, and she describes it as a "tight" feeling around her head. She started physician assistant school 2 months ago and has been really stressed out. Upon physical exam, no abnormal findings are found. Additionally, all laboratory findings are within normal limits. Question What is the most likely diagnosis?

Correct answer: Tension headache Explanation The clinical picture is suggestive of a tension headache, due to the description of bilateral headache and the "tight" feeling around the head. Additionally, tension headaches are precipitated by and associated with new onset of stress. Cluster headache is not correct, because the patient usually presents with ipsilateral nasal congestion, rhinorrhea, lacrimation, and/or redness of the eye, all of which are not present in this case. Migraine with aura is not correct, because the patient would describe an aura that can be visual or sensory that precedes head pain, all of which is not described in this case. Migraine without aura is not correct, because the patient usually presents with a headache that pulsates. Pain is usually unilateral; nausea, vomiting, and photophobia are also common, all of which are not present in the patient. Posttraumatic headache is usually present within 7 days following an injury. Additionally, the headache would worsen over weeks and eventually subside; however, an injury is not present in the problem and therefore this is not the correct answer.

Question Relaxation training has been shown to be as effective as biofeedback for what condition?

Correct answer: Tension headaches Explanation Biofeedback has been used to treat all of the conditions listed. However, relaxation training has been shown to be as effective as biofeedback for tension headaches, generalized anxiety, and insomnia. Patients suffering from the other listed conditions still seem to benefit more from the use of biofeedback

Case An 8-year-old African-American girl is admitted to the hospital for reassessment of her seizure disorder. She has suffered generalized tonic-clonic seizures since birth in spite of many different medications and consultations. The nature of her seizure disorder is unknown. Her prenatal course was uncomplicated. She was a full term, natural spontaneous vaginal delivery. Her Apgar scores were 8 and 9. She had no perinatal fever, sepsis, or meningitis. She was first observed to have seizure form activity at 10 days of life, when she was rushed back to the hospital. Since infancy, she has seen many specialists in the area without any definitive explanation for the seizures. The medications have tempered the frequency and duration of the seizures, but she did not meet her developmental milestones and has been diagnosed with mild intellectual disability. You review her lab reports and find that her electrolytes are normal, phenytoin level is therapeutic, and complete blood count is normal. The chart indicates that she has no fever and her blood pressure is normal. You leave to check on some other patients, but are called back at 2AM by the nurse who reports that the patient is seizing. You rush to the patient's bedside, and find that there has been seizure activity recorded on the EEG for the past 10 minutes. Question The best description of your assessment and management at this point would be:

Correct answer: The patient is in status epilepticus and lorazepam could be given to stop the seizure Explanation The correct answer is that the patient is in status epilepticus and lorazepam could be given to stop the seizure. Epilepsy is a relatively common disorder, affecting approximately 1% of the population. Some seizure types are summarized in the table. There have been many new anti-epileptic medications approved in recent years. The newer medicines may have significant side effects, yet offer hope to those whose seizures are not controlled on traditional agents such as phenytoin or carbamazepine. Refer to the table for a summarization of some of the newer agents. Newer Anticonvulsant Medications Felbamate - works for both partial and generalized seizures, can cause fatal aplastic anemia or liver failure Gabapentin - few side effects, no hepatic metabolism, renal clearance Lamotrigine - rarely causes sedation, broad spectrum activity, occasional rash or Stevens-Johnson Topiramate - broad activity, some weight loss, can cause paresthesias and kidney stones Tiagabine - narrow activity, must be titrated over weeks Levetiracetam - effective at the starting dose, not metabolized by liver Oxcarbazepine - reformulated carbamazepine, well tolerated but can cause hyponatremia Zonisamide Fosphenytoin - reformulated phenytoin for status epilepticus, administered IV The definition of status epilepticus is generally considered a seizure that lasts longer than 5 to 15 minutes, so this patient is in status epilepticus. Vagus nerve stimulation is a procedure where an electrode is placed over the vagus nerve surgically, and a "generator", similar to a pacemaker, is surgically inserted underneath the skin. Stimulation of the vagus nerve has been found to decrease seizure activity, although the exact mechanism is not clear. Implantation of such a device is not practical for the immediate treatment necessary to stop status epilepticus. A high fat, low carbohydrate diet, or "ketogenic" diet, has been used with much success in treating refractory seizure syndromes, particularly in children. The diet is manipulated until ketones appear in the urine, being careful not to induce hypoglycemia. Again, this would be a reasonable long-term treatment strategy but could not be used in the acute status epilepticus setting. This patient has been seizing for over 30 minutes, indicative of status epilepticus. Benzodiazepines have a rapid onset and can be titrated quickly to suppress convulsive activity. Tiagabine needs to be titrated over weeks. Fosphenytoin could also be used. Surgery for refractive cases of epilepsy is becoming a more accepted method of treatment, and this patient might be a candidate if medical therapies fail.

Case Ico-delete Highlights A 67-year-old man with an 80-pack/year smoking history and chronic bronchitis presents 30 minutes after suffering what was described as a "seizure" by his wife. According to his wife, this was his first seizure. During the seizure episode, the patient did not lose consciousness, but was observed to have developed an "involuntary jerking in his right arm and leg" that lasted about 5 minutes, with right arm weakness immediately following this involuntary activity. On physical exam, the patient appears drowsy, confused, and answers questions slowly. His right arm has objective weakness rated as 4/5; the remainder of the neurological exam is normal. Question What aspect of this patient's presentation most closely identifies this event as a simple seizure?

Correct answer: There was no impairment of consciousness Explanation Focal seizures without dyscognitive features (formerly termed simple partial seizures) can cause motor, sensory, autonomic, or psychic symptoms without impairment of cognition. Involuntary movements are typically clonic (repetitive, flexion/extension movements). Other possible manifestations include somatosensory symptoms (e.g., paresthesias or tingling) that spread (or "march") to different parts of the limb or body depending on their cortical representation. Generalized tonic-clonic seizures are attacks in which consciousness is lost. A focal seizure may progress to a generalized seizure, in which loss of consciousness occurs secondarily. The initial phase of generalized tonic-clonic seizures is usually tonic contraction of muscles throughout the body. Contraction of the jaw muscles may cause biting of the tongue. A marked enhancement of sympathetic tone leads to increases in heart rate, blood pressure, and pupillary size. After 10-20 seconds, the tonic phase of the seizure typically evolves into the clonic phase, produced by the superimposition of periods of muscle relaxation on the tonic muscle contraction. The postictal phase is characterized by unresponsiveness, muscular flaccidity, and excessive salivation that can cause stridorous breathing and partial airway obstruction. Bladder or bowel incontinence may occur at this point. Automatisms are coordinated involuntary orobuccolingual movements suggestive of focal seizures with dyscognitive features (formerly termed complex partial seizures). Automatisms may consist of very basic behaviors such as chewing, lip smacking, swallowing, or "picking" movements of the hands, or more elaborate behaviors such as a display of emotion or running. A transient focal deficit (usually unilateral) following a simple or complex focal seizure is referred to as Todd paralysis, and it should resolve within 48 hours.

Case A 19-year-old woman presents with worsening headaches. She reports a multi-year history of episodic, throbbing headaches. They have intensified, and she now misses classes and work periodically due to her headaches. Recently, the headaches occur about 4-6 times per month, up from 1-2 per month when she first started experiencing headaches. Her headaches last 2-3 days duration and are accompanied by nausea, vomiting, and light sensitivity. After the headache resolves, she denies any residual symptoms. She denies neurologic symptoms, such as vision or taste changes, gait disturbances, and memory loss. She has tried multiple over-the-counter pain medications without relief. Her mother and maternal aunt experienced similar headaches. She reports some increased stressors and less sleep since starting college. Her past medical history is unremarkable, with no known medical conditions (except for the headaches), no surgeries, no chronic medications, and no drug allergies. She has never been sexually active and reports regular menses. She denies the use of tobacco, alcohol and drugs. On physical exam, the patient appears comfortable and reports no headache at this time. Her entire exam, including neurological, is normal. Question In addition to treatment options for acute headache, this patient should be educated about which of the following daily medications to prevent headache episodes?

Correct answer: Topiramate Explanation This patient is suffering from migraine headaches, common headaches classically described as throbbing in nature and accompanied by nausea (and possibly vomiting), photophobia (sensitivity to light), phonophobia (sensitivity to sounds), and possibly visual changes and aura. Migraines are more common in women than men, but can occur in men. They are usually a chronic condition, beginning in adolescence or early adult years. Often a family history is present. Stress, sleep deprivation, and various foods can be among many triggers for migraines. With a classic history and no alarm features of a more serious condition, no testing would be necessary. Treatment approach includes offering treatment for acute headache and, with frequent migraine such as this patient's, prophylactic treatments, such as topiramate, propranolol, amitriptyline, valproic acid, and botulinum toxin type A. Topiramate augments GABA activity and has primarily been used for seizure disorders. It is approved and effective for prevention of migraine headaches. Cetirizine, an antihistamine, is useful for allergic rhinitis and urticarial. If this patient's headaches were attributed to allergic symptoms, cetirizine may be useful. However, cetirizine does not prevent migraines. Clopidogrel reduces platelet activation and aggregation and is used in acute coronary syndromes and prevention of thromboembolism. Although migraine headaches can be attributed to intracranial vascular changes, there is no role for clopidogrel in reducing migraine frequency. Hydrocodone, a semi-synthetic opioid, is often marketed in combination with other analgesics (such as acetaminophen) and is mostly used for acute pain. Especially with the frequency of this patient's headaches and risk for addiction/abuse, opioid use should be discouraged for this patient. Opiates are not especially effective for the acute migraine pain and are not at all effective for migraine prophylaxis. Sumatriptan works on 5-HT1 serotonin receptors to cause vasoconstriction. It is primarily used for acute migraine treatment, but is it also useful in acute treatment of cluster headaches. It has an oral, nasal, and subcutaneous route of delivery. Sumatriptan is not recommended for prophylaxis of any type of headache

Case A 44-year-old man presents after an episode he describes as "passing out." He has a past medical history of hypertension, diabetes mellitus type 2, and osteoarthritis. Earlier in the day, the patient visited his primary care physician for routine blood work. During his blood draw, the patient had an acute onset of syncope. The patient has never had such an episode before. Vital signs upon presentation are as follows: BP 124/82 mm Hg, P 88 beats/min, R14/min. EKG is within normal limits. Cardiovascular examination reveals a normal S1 and S2, with no rubs, murmurs, or gallops. The patient is being treated with atenolol and hydrochlorothiazide for his hypertension. His blood sugar is 96 mg/dL. Question What is the most likely cause of the patient's syncope?

Correct answer: Vasovagal episode Explanation This patient's syncope is most likely a vasovagal episode. This reaction is common in patients who are exposed to blood or are having their blood drawn. In fact, vasovagal episodes are the most common cause of syncope. These individuals typically present with loss of consciousness when exposed to a certain trigger. The trigger leads to an activation of the nucleus tractus solitarius of the brainstem, which results in a surge of parasympathetic response leading to the cardioinhibitory and vasodepressor effects. Although this patient has a history of diabetes mellitus, he is not currently hypoglycemic. Hypoglycemia, if severe, can potentially lead to a loss of consciousness; however, diabetics who are hypoglycemic typically have a sympathetic response. Arrhythmia is not the correct answer choice. This patient does not have any cardiac history of arrhythmia. As the patient had the syncopal episode during the blood draw, and there were no EKG changes, this answer choice is unlikely. Neurogenic shock occurs in the context of acute spinal cord injury. It is caused by autonomic dysfunction that causes widespread vasodilation. Orthostatic hypotension is a reasonable answer choice; however, this patient's acute syncopal episode in response to the blood draw makes a vasovagal reaction more likely. In addition, this patient has no history of dehydration or any other cause that would lead to this condition.

Case A 70-year-old woman is brought to your attention by her family because of the slowly progressive gait disorder, the impairment of mental function, and urinary incontinence. About 1 year ago, she started having weakness and tiredness in her legs, followed by unsteadiness; her steps became shorter and shorter, and she also experienced unexplained backward falls. She is becoming emotionally indifferent, inattentive, and her actions and thinking have became "dull". Over the past month, she has started having urinary urgency and involuntary leaking of urine. Besides multivitamins and local application of the Timolol for glaucoma, she takes no other medications; there are no other symptoms. Question What is most likely the best method of treating the patient's urinary problems?

Correct answer: Ventriculoperitoneal shunt Explanation Clinical triad of slowly progressive gait disorder, followed by impairment of mental function and then sphincteric incontinence strongly suggests the presence of normal-pressure hydrocephalus. Ventricular expansion is the cause of symptoms, and surgical CSF shunting is the main treatment modality. The potential benefit from surgery is usually evaluated by testing gait, cognition, and micturition before and after CSF drainage. Antimuscarinic Tolterodine is an antispasmodic that is used for symptomatic treatment of urinary incontinence in patients with an overactive bladder (urge incontinence). Antimuscarinic drugs are contraindicated in patients with glaucoma. A urinary tract infection will probably manifest with a strong, persistent urge to urinate, burning sensation when urinating, passing frequent, small amounts of urine that has unusual smell and the appearance. Your patient has no such signs and symptoms; therefore, in this case, antibiotics are not indicated. Donepezil is used to treat dementia, but in the case of normal-pressure hydrocephalus, the problem is anatomic (the distortion of the periventricular limbic system and frontal lobes), and the best treatment is probably surgical. Kegel exercises can prevent or control urinary incontinence and other pelvic floor problems in cases of pelvic sphincter weakness. However, pelvic sphincter weakness will probably manifest as stress incontinence.

Case A 42-year-old man presents to the emergency department with a severe headache. He has been getting several of these headaches recently and has tried all over-the-counter pain relievers and headache medicines with no relief. His current headache started 15 minutes ago. He describes the pain as located next to and behind his left eye and "stabbing/excruciating" in nature. He feels like his left eye tears up profusely with these headaches. He reports he has been healthy otherwise, with no chronic medical conditions, no history of surgery, no medications, and no drug allergies. He denies recent stressors that may have caused his headaches. On physical exam, the patient appears slightly agitated and appears uncomfortable. His left eye's conjunctiva is mildly injected, and lacrimation is noted. His right eye is normal. Cranial nerves II-VII are intact, although the patient expresses discomfort when the light is shown in his left eye. Speech, gait, coordination, and reflexes are all normal. The remainder of his exam is normal. Head MRI is performed and reported as normal. Question Which of the following medications is the most appropriate prevention for this patient's type of headaches?

Correct answer: Verapamil Explanation This patient is suffering from cluster headaches. Cluster headaches are classified as trigeminal autonomic cephalalgias and are most common in men. Cluster headaches may be triggered by alcohol, histamine, and nitroglycerin, but often no cause is identified. There may be a genetic component. Classic presentation of cluster headache includes episodic, unilateral, severe headache, usually located in the temporal and/or orbital region, along with associated lacrimation, injection, eyelid edema, miosis or ptosis of the ipsilateral eye, and nasal congestion and rhinorrhea. Imaging studies, such as head MRI, should be performed, to rule out other primary causes of headache, such as vascular malformation, neoplasm, and infection. Non-oral treatments are recommended due to the short duration (15-180 minutes). High-flow 100% oxygen, nasal or subcutaneous triptans, and nasal, intramuscular or intravenous dihydroergotamine are all considered appropriate acute treatments. For prevention, verapamil, a non-dihydropyridine calcium channel blocker, is the drug of choice and should be initiated at the beginning of a cluster. During a bout of cluster headaches, an afflicted patient may experience 1-8 of these excruciating headaches per day for several days to weeks. Amitriptyline, a tricyclic antidepressant, is often recommended (off-label) for migraine and tension-type headache prophylaxis. It has not been shown to be useful for prevention of cluster headaches. Fluoxetine, a selective serotonin reuptake inhibitor (SSRI), is an antidepressant that has shown some efficacy in reducing tension-type and migraine headaches. It may be appropriate for a trial in a patient with cluster headaches who seems to have excess anxiety or depressive symptoms with stress-related onset of headaches. However, fluoxetine is generally not considered a prophylactic treatment for cluster headaches. Propranolol, a non-selective beta-blocker, is primarily used for treatment of hypertension. It is also used for various cardiovascular rhythm problems, essential tremor, and migraine headache prophylaxis. It has not consistently been shown to be useful in the prevention of cluster headaches.

Case A 24-year-old woman presents with a 2-week history of persistent fatigue and decreased functional capacity. The woman has been consuming a strict total vegan diet (no dairy products or eggs) for the past 8 years. She also has a 3-week history of numbness and pins and needles sensation in her feet. The patient is having other problems, such as increased irritability and forgetfulness. Physical exam reveals pallor, peripheral neuropathy of her lower extremity, and memory deficits. CBC shows a macrocytic, normochromic anemia. There is also an elevation in the levels of methylmalonic acid. Question What therapy is indicated?

Correct answer: Vitamin B12 supplementation Explanation A strict vegan diet consists of fruits, vegetables, and grains. Meat (including fish, seafood, and poultry), dairy products, and eggs are excluded. This may result in vitamin B12 deficiency, presenting as macrocytic, normochromic anemia/megaloblastic anemia. The clinical manifestations associated with vitamin B12 deficiency include hematological, neurological, and psychiatric manifestations. The hematological manifestations include macrocytic (megaloblastic) anemia and pancytopenia in advanced cases. Paresthesias, peripheral neuropathy, and demyelination of the corticospinal tract and dorsal columns (subacute combined systems disease) are the neurological sequelae associated with vitamin B12 deficiency. Psychiatric disorders associated with vitamin B12 deficiency include impaired memory, irritability, depression, and dementia; rarely, psychosis is a result of the deficiency. Treatment of vitamin B12 deficiency includes administration of cobalamin through oral or parenteral route. Cobalamin is administered as 1000 mcg daily intramuscular (IM) 2 times per week for 2 weeks during the initial period, which is followed by 1000 mcg/week IM for 5 weeks; afterward, the patient must receive maintenance therapy of 100-1000 mcg IM every month for life. If iron studies indicate the need, iron supplementation may be considered in a hypochromic, microcytic anemia. Folate supplementation should be considered in macrocytic anemia with decreased serum folate levels. A normocytic, normochromic anemia may occur due to blood loss, aplastic anemia, sickle cell anemia, or end-stage renal disease. If severe, transfusion may be necessary. End-stage renal disease may result in decreased production of erythropoietin (EPO), and EPO therapy may be necessary.

Case Ico-delete Highlights A 35-year-old man presents with paralysis and confusion. His wife states he had been working in the yard about 3 days ago and suddenly began to not feel well. She states that it has progressed to the point where he appears quite confused, is extremely agitated, and does not seem to be able to close his right eye completely or smile on the right side of his face. The patient has a rash on the right arm that does not seem to be pruritic. Physical examination of the patient reveals findings of altered mental status, a cranial nerve palsy of the facial nerve (cranial nerve VII), exaggerated deep tendon reflexes, and the presence of a maculopapular rash on the right forearm. Question What is the most likely source of this patient's condition?

Correct answer: West Nile virus Explanation This patient has encephalitis secondary to exposure to the West Nile virus. Patients with encephalitis, regardless of the cause, will present with an altered mental status that has a huge variability (from subtle deficits to 100% unresponsiveness). Meningeal irritation may also be seen if involvement of the meninges is present (e.g., photophobia and nuchal rigidity). Seizures will also be seen, as well has hemiparesis, cranial nerve palsies, exaggerated deep tendon—or even pathologic—reflexes. Patients will appear confused, agitated, or obtunded. Many times, patients with West Nile virus encephalitis may be misdiagnosed as having Guillain-Barré syndrome because of the presence of the symptom of flaccid paralysis that is specific to this infection. Another key characteristic of this encephalitis is the presence of a nonpruritic maculopapular rash, estimated to be seen in up to 50% of patients with this type of infection. This patient most likely has encephalitis secondary to the West Nile virus. A patient who is unvaccinated and presents with parotitis and altered mental status will more likely have encephalitis secondary to mumps. If a patient has tremors of the eyelids, tongue, lips, and extremities, the most likely cause is the St. Louis virus. Findings such as hydrophobia, pharyngeal spasm, and hyperactivity would be seen in a patient with rabies-induced encephalitis. A patient who has encephalitis from the varicella-zoster virus will more commonly have the dermatomal pattern of grouped vesicles that is typically seen in a zoster infection. Herpes Simplex Encephalitis (HSE) is the most common form of encephalitis in the United States.

Question Ico-delete Highlights A 17-year-old boy develops progressively abnormal muscle fatigability. He is diagnosed as suffering from myasthenia gravis and is admitted into a hospital. In the course of his treatment with pyridostigmine, he develops increased weakness, nausea, vomiting, sweating, and bradycardia. What is the best management for this patient?

Correct answer: Withdrawal of pyridostigmine Explanation The patient is having a cholinergic crisis. The over-dosage of cholinesterase inhibitors, such as pyridostigmine and neostigmine, results in cholinergic crisis; it is characterized by salivation, lacrimation, urinary incontinence, emesis, and other symptoms like colic, diarrhea, and miosis. These symptoms are similar to myasthenic crisis, and because treatment differs in both, the 2 conditions need to be differentiated. After stabilizing the patient, edrophonium testing should be done in order to distinguish between a myasthenic and a cholinergic crisis. The management of patients with cholinergic crisis involves prompt withdrawal of cholinesterase inhibitors. The immediate administration of atropine is also recommended. Thymectomy is indicated in severe cases of myasthenia gravis. It is especially effective in young women. Steroids sometimes help if immunosuppression is needed for thymoma, which may be associated with myasthenia gravis. Plasmapheresis is a treatment for severe cases of myasthenia gravis with or without thymectomy. Neostigmine is an alternative cholinesterase inhibitor. There is no rationale in switching from pyridostigmine if a cholinergic crisis has already occurred.

Case You are evaluating a 14-year boy for a pre-participation sports physical. He has been conditionally accepted as a wide receiver on his high school's football team. He was diagnosed with generalized tonic-clonic seizures at age 6. He is well-controlled on valproic acid, having had only 2 seizures in the past 3 years, associated with an intercurrent illness. There is no history of status epilepticus, head trauma, or other neurologic abnormalities, and he maintains a B+ average in school. Past medical history is otherwise unremarkable. Growth and development have been normal and immunizations are current. Vital signs are normal. Examination is unremarkable. Question Would you clear him for football?

Correct answer: Yes; his seizure disorder is well-controlled. Explanation The purpose of a pre-participation sports evaluation is to maximize both the health of the athlete and the athlete's safe participation in sports. Generally, participation in athletics may have a favorable effect on seizure control and overall health and psychosocial benefits. When clearing patients with seizure disorders for athletic participation, a number of factors must be considered. These include: the degree of seizure control the sport involved seizure type precipitating factors, if any Here, seizures have been well-controlled, are of a common variety, and are not associated with other neurologic abnormalities. Football is not of the type of sport that presents an increased risk of fatality should a seizure occur during participation, as with activities such as skydiving, scuba diving, hang gliding or motor sports. A well-controlled seizure disorder is not a contraindication to participation in contact sports or competitive athletics in general. There is no evidence that increasing the anticonvulsant dose provides seizure prophylaxis during athletic participation.

Case A 43-year-old woman presents for the evaluation of possible seizures. The patient was well until the day of admission to the hospital. She works as a secretary at a law firm. She was having a heated argument about work assignments with her supervisor when she slumped back in her chair, became pale, and then had a 5-10 second spell of generalized tonic-clonic jerking. She was placed on the floor, and she began to rouse after 15-30 seconds. Past medical history reveals a history of 3 generalized tonic-clonic seizures at age 4. Electroencephalogram at that time revealed intermittent generalized spike and wave discharges. She was started on divalproex sodium (Depakote) and had no further seizures. At age 8, a repeat electroencephalogram was normal and divalproex was discontinued. Seizures did not reoccur. She was in a major automobile accident at age 27, was in the hospital for 3 months, and in rehabilitation for another 3 months. She made a full recovery. The patient describes her health now as 'excellent'. Physical examination reveals a well-developed, well-nourished woman; she is awake, alert, fully-oriented, and in no acute distress. Temperature is 98.8 F orally. Blood pressure is 129/85mmHg. Heart rate is 76/min and regular. Respirations are 14/min and unlabored. General neurological and medical exams are normal. Lab studies include a normal complete blood count and differential, blood chemistry panel, urinalysis, chest X-ray, and cardiogram. A portable electroencephalogram is done and reported as normal. Syncope and seizures are being considered, but pseudoseizures are too because of the relationship of this spell to emotional distress. Question What profile is best correlated with the likelihood of pseudoseizures (psychogenic non-epileptic seizures) in patients where pseudoseizures are a consideration?

Correct answer: Young (20 - 30 years old) women with history of sexual abuse Explanation While pseudoseizures (Psychogenic non-epileptic seizures) can occur at any age, the great bulk of them occur in the 20-30 year old age group. Women predominate (75%) in this age group, although this gender difference fades with years and nears 50% in the elderly. A history of sexual abuse can be obtained in 32% of women with pseudoseizures in this age group. The incidence of reported sexual abuse decreases steadily with age (4% in the over 55 age group). The middle years (20-40) are relatively quiet for the incidence of pseudoseizures. Although the percentage of men among patients with pseudoseizures increases with age, it never reaches 50%. While incidence of true epilepsy is frequent among patients with seizures (it is felt that 5-20% of those with a diagnosis of seizures have pseudoseizures), this correlation is seen mostly in the young and is not a serious factor over 55 years of age (2). Although the 20-30 year old age group contains the bulk of patients with pseudoseizures, the men in this age group are a distinct minority (25%). Among the elderly who have pseudoseizures, there is a relatively high incidence of major health problems (42%), especially serious cardiovascular issues (2), but this is not seen in the young age group. Again, the middle years are relatively quiet with respect to pseudoseizures. Men remain in the minority. However, a history of fibromyalgia does have a predictive value of 85% for pseudoseizures (3). Still again, the middle years are relatively quiet for pseudoseizures and men remain in the minority. There is, however, a strong relationship between drug abuse and pseudoseizures, but not as strong as among the young. While the elderly can have pseudoseizures, it is relatively uncommon. Again, men are in the minority. A history of physical abuse is uncommon in males.


Ensembles d'études connexes

NBSN 8004 - Biostatistics: Module 4 (RR/OR; Sensitivity, Specificity; Survival Analysis)

View Set

Pharmacology - Prep U - Chapter 38

View Set

Blood Bank Ch.8 (Compatibility Testing)

View Set

Managerial Accounting Chapter 6 Homework

View Set